Download as pdf or txt
Download as pdf or txt
You are on page 1of 149

CHUdNG2------------------

I. . _____ CAC DE ToAN ON TAP I)


1

"Beauty is the first test; there is no permanent place in


the world for ugly mathematics. "
- Godfrey Harold Hardy
, ?

1. Cac de thi tuyen sinh chfnh thlic


1.1. D€ thi tuy€n sinh THPT chuyen DHSP Ha NQi, 2014
Bai 1 (1.5 c1iim). Ghl sit a, b, c, x, y, Z Hl cac s6 thvc khac 0 thoa man d6ng
thdi cac di~u kien x + !!..
. !.!. y + £. a + l.b + ~
z = 0 va :!. c = 1. Chung minh r~ng
x2 y2 Z2
2a + b2 +"2 c = 1.
Bai 2 (1.5 c1iim). Tim t&t eft cac s6 thve x, y, z thoa man
xVI - y2 + y,J2 - Z2 + z,J3 - x 2 = 3.
Bai 3 (1.5 c1iim). Chung minh r~ng vdi s6 nguyen duong n ~ 6 thi s6
2·6·10 .. · (4n - 2)
an = 1 + ----~------..:.-
(n + 5)(n + 6) .. · (2n)
Ia m9t s6 chinh phuong.
Bai 4 (1.5 c1iim). Cho a, b, c > 0 thoa man abc = 1. Chung minh r~ng
1 113
------+ + ~-.
ab+a+2 bc+b+2 ca+c+2 4
Bai 5 (3.0 c1iim). Cho hinh vuong ABC D voi tam O. G<;>i M la trung digm
cua e<;mh A B. Cae di€m N, P thea thu tv thu9c cae qmh Be, CD sao cho
M N II A P. Chung minh r~ng
a) Tam giac B N 0 d6ng d~ng voi tam giae D 0 P va L Nap = 45 0

b) Tam dUdng tron ngo~i ti~p tam giae Nap thuge 0 C .


c) Ba dUdng th~ng BD, AN, PM d6ng quy.
Bai 6 (1.0 c1iim). Co bao nhieu t~p con A eua t~p h<;lp {I, 2, ... , 2014} thoa
man: A co it nhdt 2 phdn til va ntu x E A, yEA, x > y thi /~y EA.

Vo QU6e Ba C:1n - Nguy€n M~nh Linh 35


CHVdNG 2. cAc HE ToAN ON TAp
1.2. D~ thi tuy~n sinh THPT chuyen KHTN, 2014 (vong 1)
Bai 1 (3.0 c1iim).
a) Gild phu'dng trinh:

(.J1 +x + .JI-X) (2+2.JI-x 2 ) = 8.

b) Giai h~ phudng trinh:

Bai 2 (3.0 c1iim).


a) Gia sU' x, y, z la ba s6 thl;iC dudng thoa man di€u ki~n x + y + z = x y z.
Chung minh r~ng
x 2y 3z xyz(5x + 4y + 3z)
----::-+
1+ x2 1 + y2
+----=-
1 + Z2 (x+y)(y+z)(z+x)·

b) TIm t~t ca. cac nghi~m nguyen clla phudng trinh

x 2 y2(X + y) + x + y = 3 + xy.
Bai 3 (3.0 diim). Cho tam giac ABC nh9n yoi A B < Be. D Ul di€rn thuQC
qmh B C sao cho AD la phan giac clla L B A C. DUdng th~ng qua C song song
voi A D dt trung trl;ic clla A C t~i E . DUdng th~ng qua B song song yoi A D c~t
trung trl;ic clla A B t~i F.
a) Chung rninh r~ng tam giac A B F d6ng d~ng yoi tam giac ACE.

b) Chung rninh r~ng cac dUdng th~ng BE, C F, AD d6ng quy t~i mQt di€m,
g9i di€rn d6 la G.

c) DUdng th~ng qua G song song yoi A E dt dUdng th~ng B F t~i Q . DUdng
th~ng Q E dt dUdng tron ngo~i ti~p tam giac GEe t~i P khac E . Chung
minh ding cac di€m A, P, G, Q, F cung thuQc mQt dUdng tron.
Ba i 4 (1. 0 cJiim). Gia sit a, b, c la cac s6 thl;iC dudng thoa man d~ng thuc
a b + be + c a = 1. Chang minh r~ng
5
2abc(a + b + c) ~ -
9
+ a 4 b 2 + b 4 c 2 + c 4 a 2.

36 Vo Qu6c Ba Ca':n - Nguy€n M~nh Linh


1. CAC DE THI TUYEN SINH CHfNH THUC

1.3. D€ thi tuy€n sinh THPT chuyen KHTN 2014, (vong 2)


Bai 1 (3.5 dim).

a) Gilt sil' x, y lit cac s6 thvc duong phan bi~t thoa man

y 2y2 4y 4 8y 8
- - y+
x + x 2 + y2 + X4 + y4 + x8 _ y8 =4.

Chung minh ding 5 y = 4 x .

b) Gilti h~ phuong trlnh:

2x 2 -3 y2 +x y = 12,
1 6X+X2y = 12+6y+y2 x .

Bai 2 (2.5 diim).

a) Cho x, y Ut cac s6 nguyen ldn hon 1 sao cho 4 x 2 Y 2 - 7x + 7 y Ia s6 chinh


phuong. Chung minh r~ng x = y.

b) Gilt sil' x, y lit cac s6 thvc kh6ng am thoa man x 3 + Y3 + X Y = X2 + Y2 •


Tim gia tri ldn nhfit va nho nhfit cua bi€u thuc

1 + .Jx
P = + 2 + .Jx .
2 + .Jy 1 + .Jy

Bai 3 (3.0 diem). Cho tam giac ABC nQi ti6p duong tron ( 0) va di€m P n~m
trong tam giac thoa man P B = PC. D la di€m thuQc q.nh B C (D khac B
va D khac C) sao cho P n~m trong duong tron ngol;li ti6p tam giac DAB va
duong tron ngOl;li ti6p tam giac D A C. Duong th~ng P B dt duong tron ngol;li
ti~p tam giac DAB tl;li E khac B. Duong th~ng P C dt duong tron ngol;li ti~p
tamgiac DAC tai F khac C.

a) Chung minh r~ng b6n di€m A, E, P, F cung thuQc mQt duong tron.

b) Gilt sil' dUong th~ng A D c~t duong tron ( 0) tl;li Q khac A, duong thiing
A F dt duong th~ng Q C tl;li L. Chung minh r~ng tam giac ABE d6ng
dl;lng voi tam giac C L F.

c) G9i K la giao di€m cua duong th~ng A E va duong th~ng Q B. Chung minh
r~ng L.QKL + L.PAB = L.QLK + L.PAC.

Vo Qu6c Ba Can - Nguy6n Ml;lnh Linh 37


CHUONG 2. cAc D:E ToAN ON TAp

Bfli 4 (1.0 cliim). Cho t~p hQp A g6m 31 ph~n til va day g6m m t~p con cua A
thoa man d6ng thdi cac di€u ki~n sau:
i) M6i t~p thuQc day co it nhAt hai ph~n til.
ii) N~u hai t~p thuQc day co chung nhau it nhAt hai ph~n til thl s6 ph~n til eua
hai t~p nay khac nhau.
Chang minh r~ng m ~ 900.

1.4. O~ thi tuy€n sinh THPT chuyen, sa GD-DT HfI NQi,


2014 (d~mh cho chuyen Toan)
Bfli 1 (2.0 clie'm).
a) Gi:li phudng trlnh:

x (5x 3 + 2) - 2 ( y'2x + 1- 1) = O.

b) Giai h~ phudng trlnh:

X 2 (4 Y + 1) -2y = -3,

l X 2 (X 2 - 12y) + 4y2 = 9.

Bfli 2 (2.5 clie'm).

a) Chung minh r~ng n~u n la s6 nguyen dUdng thl25 n + 7 n - 4 n (3 n + sn)


chia h~t eho 65.

b) Tim cac e~p s6 nguyen (x, y) thea man


x 2y + xy - 2X2 - 3x +4= O.

c) Tim cac bQ s6 tv nhien (a l, a2, ... , a20l4) thea man


al + a2 + ... + a20l4 ~ 2014 2 ,
1 af + a~ + .,. + a~Ol4 ~ 2014 3 + 1.
Bfli 3 (1.5 cliim). Voi ba s6 dUdng x, y, z thay d6i thea man x +y +z = 1,
tIm gia tri IOn nhAt cua bi6u thuc
x y z
Q=
x+y'x+yz
+ y+y'y+zx
+ z+y'z+xy .

38 Vo Qu6c Ba Cffn - Nguyen M~nh Linh


1. cAc HE THI TUYEN SINH CHINH THUC

Bai 4 (3.0 diim). Cho tam giac d~u AB C nQi ti€p duong tron (0), H lit trung
di€m clla Be. M lit di€m bfit ky thuQC do;;tn th~ng B H (M kMc B). Lfiy di€m N
thuQc do;;tn th~ng C A sao cho eN = B M. GQi I la trung di€m Clla M N.
a) Chang minh r~ng b6n di€m 0, M, H, I cling thuQc mQt duong tron.

b) GQi P la giao di€m clla cac duong th~ng 0 I va A B. Chang minh r~ng
tam giac M N P la tam giac dSu.

c) Xac dinh vi tri Clla di€m M d€ tam giac I A B co chu vi nho nhk
Bai 5 (1.0 dim). Cho bang 0 vuong kich thuClc 3 x n (3 hang, n cQt, n la s6 tv
nhien lCln hon 1) dtt~c t;;tO bai cac 0 vuong nho kich thuClc 1 xI. M6i 0 vuong nho
du~c to bai mQt trong hai mau xanh ho~c do. TIm s6 n be nhfit d€ vCli mQi cach
to mau nhu th€ luon tIm du~c hlnh chif nh~t t;;to bai cac 0 vuong nho sao cho 4 0
vuong nho a 4 goc clla hlnh chit nh~t do cling mau.

1.5. D€ thi tuy~n sinh THPT chuyen, SO' GD-OT Ha NQi,


2014 (danh cho chuyen Toan-Tin)
Bai 1 (2.0 diim).
a) Giai phuong trlnh:
5x
4
+ 2x + 2 - 2,J2x + 1 = O.
b) Gicli h~ phuong trlnh:

Bai 2 (2.5 diim).


IX(2 Y +l)-Y=-3,
x 2 + y2 - 6xy = 9.

a) Chang minh r~ng n€u n Ia s6 nguyen dttong thl 5 n ( 5 n + 3 n ) - 2 n ( 9 n + 11 n)


chia h€t cho 2 1 .

b) TIm cac c~p s6 nguyen (x, y) thoa man


5x 2 + y2 - 2xy + 2x - 2y - 4 = O.

c) Chang minh r~ng trong 20 14 s6 nguyen dttong a 1, ... , a 2014 thoa man
111
-2+ -2+ .. ·+-->-4
2 c;/
a1 a2 a 2014
luon tIm dU<;lC it nhfit ba s6 b~ng nhau.

Vo QU6c Ba Cin - Nguy~n Mc,mh Linh 39


CHVdNG 2. cAc HE ToAN ON TAp

Bai 3 (1.5 diim). Voi ba s6 dudng x, y, z thoa man di€u ki~n x + y +z = 1,


chang minh r~ng
1- x2 1 - y2 1 - Z2
---+
x+yz y+zx
+ z+xy
;;::6.

Bai 4 (3.0 diem). Cho tam gi€ic d€u ABC nQi ti~p dUdng tron ( 0 ), H lii trung
di€m cua Be. M la di€m bfit ky thuQC do~n th~ng B H (M khac B). Lfiy di€m N
thuQc do~n th~ng C A sao cho eN = B M. GQi I lit trung di€m cua M N.
a) Chang minh r~ng b6n di€m 0, M, H, I cling thuQc mQt dudng tron.
b) Xac dinh vi tri cua di€m M d€ do~n th~ng M N co dQ dai nho nhk
c) Khi di€m M thay d6i va thoa man di€u ki~n d€ bai, chang minh ding di~n
tich tam giac I A B khong d6i.
Ba i 5 (1.0 diim). Cho t~p hQp A g6m 3 6 s6 tlj nhien lien ti~p tu 1 d~n 3 6. Chang
minh r~ng trong 2 5 ph~n tl1 bfit ky cua t~p hQp A luon tim dUQc 3 ph~n ti1 Hi 3 s6
doi mQt nguyen t6 cling nhau.

1.6. O~ thi tuy€n sinh THPT chuyen DHSP Ha NQi, 2015


Bai 1 (2.5 diem).
a) Cho a ~ 0, a =I- 1. Rut gQn bi€u thac

S = J 6 - 4../2· V 20 + 14 v'2
+ ;j (a + 3) """ - 3a - 1 -;- [ 2 (:;,;: ~ 1) - 1] .

b) Cho 0 < x, y < 1 thoa man l=X + G = 1. Tinh gia tri cua bilu thac

p = x + y + VX2 - xy + y2.
Bai 2 (2.0 diim). MQt xe tai co chi~u rQng U12.4 m va chi~u cao la 2.5 m mu6n
di qua mQt cai c6ng hinh parabol. Bi~t khoang each giUa hai chan c6ng Hi 4 m
va kholmg cach tu dinh c6ng (dinh parabol) Wi m6i chan c6ng la 2.J5 m (bo
qua dQ day cua c6ng).
a) Trong m~t ph~ng tQa dQ 0 x y, gQi parabol (P) : y = a x 2 voi a < 0 la
hinh bi€u di~n c6ng rna xe tai mu6n di qua. Chung minh r~ng a = - 1 .
b) Hoi xe tM co th€ di qua c6ng dUQc khong? T~i sao?

40 Vo Qu6c Ba C~n - Nguy~n Mc,mh Linh


1. cAc HE THI TUYEN SINH CHINH THUC

Bai 3 (1.5 dim). Cho hai s6 nguyen a, b thea man di~u ki~n

a2 + b2 + 1 = 2(ab + a + b).

Chung minh r~ng a va b la hai s6 chinh phudng lien ti~p.


Bai 4 (3.0 dim). ChotamghicnhQn ABC (AB < AC), M IatrungdiSm
cua e<;mh B C, 0 la Him cua duong tron ngo~i ti~p tam giac. Cac duong cao
AD, BE, C F cua tam giac ABC d6ng quy t~i H. Cac ti~p tuy~n v(ji ( 0) t~i
B va C dt nhau t~i S. GQi X, Y l~n lUQt la giao diSm cua duong th~ng E F v(ji
cac duong th~ng B S, A O. Chung minh r~ng
a) MX ..1 BF.
b) Hai tarn giac S M X va D H F d6ng d~ng.
EF BC
C) FY=CD'

Ba i 5 (1. 0 diim). Trong m~t ph~ng tQa dQ 0 x Y , cho tarn giac ABC co cac dinh
la cac diSm nguyen (m6i diSm dUQc gQi la diSm nguyen n~u hoanh dQ va tung d9
cua cac diSm do H1 s6 nguyen). Chung minh r~ng hai l~n di~n tich cua tam giac
ABC la mQt s6 nguyen.

1.7. D€ thi tuy~n sinh THPT chuyen KHTN 2015, (vong 1)


Bai 1 (3.0 diim).
a) Ghi sti' a, b la hai s6 thlJC phan bi~t thea man a 2 + 3a = b 2 + 3b = 2.
Chung minh r~ng a +b = -3 va a 3 + b3 = -45.
b) Gild h~ phudng trlnh:

j 2X+3 Y =5X Y ,
4x 2 +y2=5xy2.

Bai 2 (3.0 diim).


a) Tim t~t ca cac s6 nguyen x, y kh6ng nhe hdn 2 sao cho x y - 1 chia h~t
cho (x - 1)( Y - 1).

b) Vdi x, Y H1 cac s6 thlJC thay d6i thea man di~u ki~n x 2 y2 + 2y + 1 = 0,


tim gia tri Wn nh~t va nhe nhfit cua biSu thuc
p = xy
3y + 1

Vo Qu6c Ba Cffn - Nguy~n M~nh Linh 41


CHVdNG 2. cAc DE ToAN ON TAP
a
B i 3 (3.0 aim). Cho tam giac nh9n ABC khong cfm co tam dudng tron nQi
tiSp la di€m I. DUdng th~ng A I ciit B C t<;li D. G9i E, F lfin luQt la cac di€m
d6i xang cua D qua Ie, lB.

a) Chung minh r~ng E F song song v(ji Be.


b) G9i M, N, J lfin lUQt la trung diem cua cac dO<;ln th~ng DE, D F, E F.
DUdng tron ngo<;li tiSp tam giac A E M c~t dudng tron ngo<;li tiSp tam
giac A F N t<;li P khac A. Chang minh r~ng b6n diem M, P, N, J cung
thuQc mQt dUdng tron.

c) Chung minh r~ng ba diem A, J, P th~ng hang.


Bai 4 (1.0 aiim).

a) Cho biing 0 vuong 2015 x 2015. Ky hi~u 0 (i, j) la 0 d hang tha i, cQt
thu j. Ta viSt cac s6 nguyen dUdng ttt 1 dSn 2015 vao cac 0 cua bang
thea quy t~c sau:

i) S6 1 dUQc viSt vao 0 (1, 1).


ii) NSu s6 k dUQc viSt vao 0 (i, j) v(ji i > 1, thi s6 k + 1 dUQc viSt
vao 0 (i - 1, j + 1).
iii) NSu s6 k dUQc viSt vao 0 (1, j) thl s6 k + 1 dUQc viSt vao 0 (j + 1, 1).

1 3 6 10 ...
2 5 9 ...
4 8 ...
7 ...
...

Khi do, 86 2015 dUQc viSt vao 0 (m, n). Hay xac dinh m va n.

b) Gia sit a, b, e Ia cac s6 tht!c dUdng thoa man a b + b e + e a + abe ~ 4.


Chang minh r~ng

a2 + b2 + e2 + a + b + e ;:: 2 (ab + be + ea) .

42 Vo Qu6c Ba Cffn - Nguy€n M<;lnh Linh


1. cAc HE THI TUYEN SINH CHfNH THUC

1.8. D~ thi tuy~n sinh THPT chuyen KHTN, 2015 (vong 2)


Bai 1 (3.0 dim),
a) Cho a, b, c la cae s5 thue thoa man
(3a + 3b + 3C)3 = 24 + (3a + b - C)3 + (3b + C - a)3
+(3c+a-b)3.
Chang minh r~ng
(a + 2b)(b + 2c)(c + 2a) = 1.

b) Giiii h~ phuong trlnh:


+ 2y + xy = 5,

Bai 2 (3.0 diim) ,


! 2x
27 (x + y) + y3 + 7 = 26x 3 + 27x2 + 9x.

a) Tim t~t eel cae s5 tt! nhien n de n + 5 va n + 30 d6u Ia s5 ehinh phuong (s5
ehinh phuong la s5 b~ng blnh phuong ella m9t s5 nguyen).
b) Tim t~t eel cae s5 nguyen x, y thoa man d~ng thae
1+ .jx + y + 3 = .Jx + .Jy.
c) Cho x, y, z la cae s5 tht!e IOn hon 2. Tim gia tri nho nh~t ella bieu thac
x y z
p = + + .
.Jy+z-4 .Jz+x-4 .Jx+y-4

Bai 3 (3.0 diim) , Cho tam giae nhQn ABC khong can vdi A B < A C. GQi M
la trung diem eua do.;tn th~ng Be. GQi H la hlnh ehi€u vuong goe ella B tren
do.;tn th~ng AM. Tren tia d5i eua tia A M l~y diem N sao eho AN = 2 M H .
a) Chang minh r~ng B N = AC.
b) GQi Q la diem d5i xang vdi A qua N. Dudng th~ng A C e~t B Q t.;ti D.
Chang minh r~ng b6n diem B, D, N, C eung thu9c m9t dudng tron, gQi
daong tron nay la (0).
c) Dadng tron ngo.;ti ti€p tam giae A QD c£t ( 0) t.;ti G khac D. Chang minh
r~ng N G song song vdi Be.
Bai 4 (1.0 diem), Ky hi~u S la t~p hQp g6m 2015 diem phan bi~t tren m~t
ph~ng. Gicl sa tfit eel cae di€m eua S khong cung n~m tren m9t duong thc1ng,
Chang minh r~ng co it nh~t 2015 dudng th~ng phan bi~t rna m6i dudng th~ng di
qua it nhfrt hai diem cua S.

Vo QU5e Ba Cffn - Nguy~n M~nh Linh 43


CHVdNG 2. cAc HE ToAN ON TAp
1.9. O~ thi tuy~n sinh THPT chuyen, sa GD-OT Hfi N9i,
2015 (dflnh cho chuyen Toan)
Bai 1 (2.0 c1iim).
a) Giiii phuong trinh:

x-,Jx-8-3,Jx+l=0.

b) Giiii h~ phuong trinh:

Bai 2 (2.5 c1iim).


a) Cho s6 nguyen duong n thoa man n va lOla hai s6 nguyen t6 cling nhau.
Chang minh r~ng (n 4 - I) chia het cho 40.

b) Tim tfit ca cac s6 nguyen t6 p va cac s6 nguyen duong x, y thoa man

p - 1 = 2x (x + 2),
1p2 _ 1 = 2y (y + 2).

c) Tim tfit ca cac s6 nguyen duong n sao cho t6n tl;li cac s6 nguyen dUdng
x, y, z thoa man phuong trinh
x3 + y3 + Z3 = nx 2 y2z2.
Bai 3 (1.5 die"m). Cho a, b, e > 0 thoaman (a + b)(b + e)(e + a) 1.
Chang minh ding
3
ab + be + ea ~ 4'

Bai 4 (3.0 diim). Cho tam giac ABC c6 ba g6c nhQn, nQi tiep dUdng tron
( 0 ). Cac dUdng cao AM, B N, C P cua tam giac ABC cling di qua di€m H.
GQi Q la di€m bfrt ky tren cung nho Be (Q khac B, Q khac C). GQi E, F
thea tha h! la dlgm d6i xang cua Q qua cac dUdng th~ng A B va A C.
a) Chang minh r~ng M H . M A = M P . M N.

b) Chang minh r~ng ba digm E, H, F th~ng hang.

c) GQi J la giao digm cua QE va A B, I la giao digm cua QF va A C. Tim


vi tri cua di€m Q tren cung nho B C dg ~ ~ + ~~ nho nhfrt.

44 Vo Qu6c Bit Cftn - NguySn M',mh Linh


1. cAc nE THI TUYEN SINH CHINH THUC

Bai 5 (1.0 diim). Chung rninh r~ng t6n tl;li cae s6 nguyen a, b, c sao eho

1.10. D~ thi tuy~n sinh THPT chuyen, SO' GD-DT Ha NQi,


2015 (d~mh cho chuyen Toan-Tin)
Bai 1 (2.0 diim).

a) Gild phuong trlnh:

(2X2 - 6x + 5)(2x - 3)2 = 1.

b) Giiii h~ phuong trlnh:

X2 + xy + y2 = 1,

12x 3
=x-y.

Bai 2 (2.5 diim).

a) Tim t~t ea cae s6 tt;i nhien x, y thoa man

x 2 -2xy+3 y 2=X+Y.

b) Tim t~t ea cae s6 nguyen duong n sao eho s6 J~n;; la s6 huu ti.

c) Cho a, b, c, d la cae s6 nguyen duong thoa man a b = cd. Chung rninh


r~ng a + b + c + d kh6ng la s6 nguyen t6.
Bai 3 (1.5 diim). Cho x, y, z Ia cae s6 thl!e duong, nho hon 1 thoa man

xyz = (1- x)(1 - y)(1 - z).

Chung minh r~ng trong ba s6 x (1 - y), y (1 - z) va z (1 - x), co it nh~t mQt


s6 kh6ng nho hdn *.

Vo QU6e Ba C§:n - NguySn M:,mh Linh 45


CHudNG 2. cAc HE ToAN ON TAp

Bai 4 (3.0 c1iim). Cho du'dng tron (0), du'dng kinh A B. G9i I Ia di€m bAt ky
tren do~n th~ng A 0 (I khae A, I khae 0). Du'dng th~ng di qua I va vuong goe vdi
A B dt dlldng tron ( 0) t~i cae di€m C va D. G9i E Ia di€m tren du'dng tron ( 0 )
sao eho D Ia di€m ehinh giUa ella eung A E . G9i K Ia giao di€m ella A E va CD.

a) Chang minh r~ng du'dng th~ng 0 K di qua trung di€m ella C E .

b) Du'ong th&ng di qua I va song song vdi C E e~t A E, BE Ifin Iu'<;lt t<;li P va
ta
Q . Chang minh r~ng giae D P E Q Ia hinh chit nh?t.
c) Tim vi tri ella di€m I tren do~n th&ng A 0 sao eho K C = K A + K O.
Bai 5 (1.0 c1iim). Cho 2015 s6 nguyen du'dng phan bi~t khong vu'<;lt qua 3019.
Chung minh r~ng trong 20 1 5 s6 do t6n t~i b6n s6 a, b, c, d sao eho

a+b+c=d.
, ?

1.11. De thi tuyen sinh THPT chuyen DHSP Hfi NQi, 2016
Bai 1 (1.5 c1iim). Chang minh r~ng bi~u thae sau nh?n gia tri nguyen du'ong vdi
m9i gia tri nguyen du'dng ella n :

P = [Jn2 + (n + 1)2 + .j(n -1)2 + n2] V4n2 + 2-2.j4n 4 + 1.

Bai 2 (2.5 c1iim).

a) Tim tAt ca cae s6 nguyen du'dng x, y thoa man

b) Tim tAt ca. cae s6 thl;ie x, y thoa man

x2- 4
x +
y2 - 4
y +8=4
(vx-l+vy-l
j - 1_) .

Bai 3 (2.0 cJiim). Cho S Ia t?P cae s6 nguyen du'dng n co d~ng n = x 2 + 3y2,
trong do x, y Ia cae s6 nguyen. Chang minh r~ng

a) N€u a, b E S thl ab E S.

b) N€u N E S va N eh~n thi N chia h8t eho 4 va ~ E S.

46 Vo Qu6c Ba C~n - NguySn M<;lnh Linh


1. cAc HE THI TUYEN SINH CHINH THUC

Bai 4 (3.0 dim). Chotamgiae ABC nh9n, AB < AC. Keduongeao AH.
Duong tron ( 0) duong klnh A H dt cae e~nh A B, A C tudng ung t~i D va E .
Duong th~ng DE dt duong th~ng B C t~i S.

a) Chung minh r~ng B DEC la tu giae n9i ti~p.

b) Chung minh r~ng S B . S C = S H 2 •

c) Duong th&ng SO dt A B, A C tudng ung t~i M va N, duong th&ng DE


dt H M, H N tudng ung t~i P va Q. Chung minh r~ng ba duong th~ng
B P, C Q va A H d6ng quy.

Bai 5 (1.0 diim). GEl sU' m6i di€m tren m~t ph~ng dUQe to bdi m9t trong ba
mau xanh, do, vang. Chung minh r~ng t6n t~i ba di€m dUQe to cling mau Ia ba
dinh eua m9t tam giae din.

1.12. D€ thi tuy~n sinh THPT chuyen KHTN, 2016 (vong 1)


Bai 1 (3.5 diim).
a) Ghli h~ phudng trinh:

l
x3+y3+XY(x+y) =4,
(xy + 1)(x 2 + y2) = 4.

b) Giai phudng trinh:

J7x+2-
J--
5-x=
8x - 3
5 .

Bai 2 (2.5 diim).


a) Tim tat ea cae gia tri eua tham s6 m sao eho t6n t~i e~p s6 nguyen (x, y)
thoa man h~ phudng trinh

2+mx y2 =3m,

1
2 + m (x 2 + y2) = 6m.

b) V(ji x, y Ia nhii'ng s6 thl,ie thoa man cae di~u ki~n 0 < x ::::; y ::::; 2 va
2 x + Y ? 2 x y, tim gia tri Wn nhat eua bi€u thue

Vo Qu6c Ba Ol'n - Nguy~n M~nh Linh 47


CHVdNG 2. cAc HE ToAN ON TAP

Bai 3 (3.0 diim). Chotamghie ABC nh<,mkhongean (AB < AC) n9i ti€p
dttdng tron ( 0 ). Phan giae eua goe B A C e£t dttClng th&ng B C t~i D va e£t
dttdng tron (0) t~i E khae A. GQi M la trung di€m eua do~n th&ng AD.
DttClng th&ng B M dt dttClng tron (0) t~i P khae B. Gia s11 cae dttClng th&ng
E P va A C e£t nhau t~i N.

a) Chang minh r~ng b6n digm A, P, N, M cling n~m tren m9t dttClng tron
va N la trung digm eua do~n th&ng A C .

sa
b) Gia dttClng tron (K) ngo~i ti~p tam giae EM N dt dttClng th&ng A C t~
Q khae N. Chang minh r~ng B va Q d6i xang nhau qua A E .

c) Gia sa dttClng tron (K) e£t dttClng th&ng B M t~i R khae M. Chang minh
r~ng R A vuong goe voi R C .

Bai 4 (1.0 diim). S6 nguyen a dttQe gQi la s6 "d~p" n~u voi mQi each s£p x~p
thea tha tv tuy y eua I 00 s6 I, 2, ... , I 00 luon t6n t~i I 0 s6 h~ng lien ti~p co
t6ng IOn hdn ho~e b~ng a . Tim s6 "d~p" IOn nhfit.

1.13. O~ thi tuy~n sinh THPT chuyen KHTN, 2016 (vong 2)


Bai 1 (3.5 dilm).

a) Giai h~ phttdng trinh:


2

l x
4x y
+4 y2
2
=5,
+ 8xy2 + 5x + lOy = 1.
b) Giai phudng trinh:

/ 2 64x 3 + 4x
V 5x + 6x + 5 = 5x2 + 6x + 6

Bai 2 (2.5 diifm).


2 1
a) Voi x,y la nhung s6 nguyen thoa man d&ng thae x -1 =
2
y2 -
3
, chang
minh r~ng x 2 - y2 ehia h~t eho 40.

b) Tim tfit ca cae e~p s6 nguyen (x, y) thoa man d&ng thae

X4 + 2X2 = y3.

48 Vo Qu6e Ba Cffn - Nguy~n M~nh Linh


1. cAc HE THITUYEN SINH CHINH THUC

Bai 3 (3.0 diim). Cho hinh vuong ABC D nQi ti~p duong tron Him O. P Ia
di€m thuQe cung nho AD eua duong tron ( 0) va P khae A, D. Cae duong th~ng
P B, P C Iftn IU<,5t dt duong th~ng A D t~i M, N. Duong trung tn;le eua AM dt
cae duong th~ng A C, P B Iftn lu<;lt t!;li E, K. Duong trung tn;ic eua D N dt eac
duong th~ng B D, P C lftn lu<;lt t!;li F, L.
a) Chling minh r~ng ba di€m K, 0, L th~ng hang.

b) Chling minh r~ng duong th~ng P 0 di qua trung di€m eua dO!;ln th~ng E F .

c) Gill sit duong th~ng E K dt duong th~ng B D t!;li S , eae duong th~ng F L
va A C e~t nhau t!;li T. Duong th~ng S T dt eae duong th~ng PC, P B
lftn lu<;lt t!;li U va V. Chling minh r~ng b6n di€m K, L, U, V cung n~m
tren mQt duong tron.
Bai 4 (1.0 diim). Chling minh r~ng v(ji mQi s6 tt;i nhien n ~ 3 luon t6n t!;li mQt
, h sap
~ xep Xi + x k
bQ n sof.. 1 , 2 , ... , n th'anh Xl, X2, . . . , Xn sao eh0 x j I--I- -
cae f. A
2-
v(ji mQi bQ chi s6 (i, j, k) rna 1 ~ i < j < k ~ n.

1.14. D~ thi tuy€n sinh THPT chuyen, SCi GD-DT Ha NQi,


2016 (danh cho chuyen Toan)
Bai 1 (2.0 diim).
a) Gilli phudng trinh:

X4 - 2x 3 +X- .j2(x 2 - x) = O.
b) Giai h~ phu'dng trinh:

~ x 2 + 2y - 4x = 0,
l4x2 - 4xy2 + y4 - 2y +4 = 0.

Bai 2 (2.0 diim).


a) Cho eae s6 tht;ie a, b, c doi mQt khae nhau thoa man a 3 + b 3 + C 3 = 3 abc
va abc =f. 0. Tinh gia tri eua bi€u thlie
ab 2 bc 2 ca 2
P =
a 2 + b2 - c2
+ b2 + c2 - a2
+ c2 + a2 - b2
.
b) Tim tfit ca eac e~p s6 tt;i nhien (x, y) thoa man
2 x • x 2 = 9y2 + 6y + 16.

Vo Qu6e Ha Cc1n - Nguy6n M!;lnh Linh 49


CHUONG 2. cAc HE ToAN ON TAp

Bai 3 (2.0 diim).


a) Cho cac s6 dUdng a, b, c thea man a 2 + b2 + C 2 = 3. Chang minh r~ng

2a 2 2b 2 2c 2
a+ b 2
+ b +c 2 + c+a 2 ;?; a + b + c.

b) Cho s6 nguyen dudng n thea man A = 2 + 2.J 12 n 2 + 1 la s6 nguyen.


Chung minh r~ng A la s6 chfnh phu'dng.
Bai 4 (3.0 diem). Cho tam giac nh9n ABC co A B < A C va n<)i ti~p dUdng
tron ( 0) . Cac du'dng cao B B', C C' dt nhau t~i digm H. G9i M Ia trung digm
cua Be. Tia M H dt dUdng tron ( 0) t~i digm P.
a) Chung minh r~ng hai tam giac B PC' va C P B' d6ng d~ng.

b) Cac du'dng phfm giac cua cac goc B PC', C P B' lfin luQt dt A B, A C t~i
cac digm E va F. G9i 0' la Him du'dng tron ngo~i ti~p tam giac A E F, K
la giao digm cua H M va A 0'.

i) Chung minh r~ng ta giac P E K F n<)i ti~p.


ii) Chung minh r~ng cac ti€p tuy~n t~i E va F cua dudng tron ( 0 ') c£t
nhau t~i m<)t digm n~m tren du'dng tron ( 0) .

Bai 5 (1.0 diem). Cho 2017 s6 hUu ty du'dng du'Qc vi~t tren m<)t du'dng tron.
Chung minh r~ng t6n t~i hai s6 dUQc vi~t c~nh nhau tren du'dng tron sao cho khi
bo hai s6 do thl 2015 s6 con l~i khong thg chia thanh hai nhom rna t6ng cac s6
(j m6i nhom b~ng nhau.

, ?

1.15. De thi tuyen sinh THPT chuyen, SO' GD-OT Ha NQi,


2016 (d~mh cho chuyen Toan-Tin)
Bai 1 (2.0 diem).

a) Giiii phudng trinh:

(2x - 1)2 - 9 = 4.J x 2 - x.

b) Giiii h~ phu'dng trinh:

+ 4y = 3,
I
X2 - y2 - 2x
x 2 +y2=5.

50 Vo Qu6c Ba Cfin - Nguy6n M~nh Linh


1. cAc HE THI TUYEN SINH CHINH THUC

Bai 2 (2.0 dim).

a) Cho cac s6 thtjc a, b, c co t6ng khac 0 thoa man a 3 + b3 + C3 = 3 abc.


Tinh gia tri cua bi€u thuc
a2 b2 c2
p = + 2+ .
b2 + c2 c2 +a a2 + b2
b) Tim tftt ca cac c~p s6 nguyen (x, y) thoa man
x 2 + 2y2 + 3xy -2x - 4y - 5 = O.
Bai 3 (2.0 die'm).

a) Tim tftt ca cac c~p s6 nguyen dl1dng (m, n) sao cho 2 m - 1 chia h~t cho n
va 2 n - 1 chia h~t cho m .

b) Cho cac s6 dl1dng a, b, c thoa man a +b +c = 1. Chung minh r~ng


abc
---+
a +b 2 b +c 2
+ C + a
1
2 :::;;-
1 1)
-+-+-
4 abc
. (1
Bai 4 (3.0 die'm). Cho dl1dng tron (0) va di€m M n~m ngoai dl1dng tron. Tli
M ke cac ti~p tuy~n M A, M B v{ji dl1dng tron (A, B Ia cac ti~p di€m). Dl1dng
th&ng qua M c£t dl1dng tron ( 0) t<;li C va D (M C < M D) sao cho di€m 0
n~m trong tam giac BCD. G9i E 1a di€m d6i xung cua C qua O. G9i S 1a
giao di€m cua E A va Be.

a) Chung minh dng hai tam giac 0 A C va MAS d6ng d<;lng.


b) Dl1dng th&ng S D dt dl1dng tron (0) t<;li di€m thu hai K. Chung minh
r~ng tam giac B K C din.

e) G9i N 1a giao di€m cua M 0 va A E . Chung minh r~ng N D ..1 D A .


Bai 5 (1.0 die'm). Cho 101 s6nguyendl1dngcot6ngb~ng300d119cvi~ttn3n
m('>t dl1dng tron. Chung minh r~ng 1uon t6n t<;li m('>t day cac s6 vi~t li~n nhau co
t6ng b~ng 100.

1.16. O~ thi tuy€n sinh THPT chuyen DHSP Ha NQi, 2017


Bai 1 (1.5 die"m). Cho cac s6 dl1dng a, b, c, d. Chung rninh r~ng trong 4 s6
211211 211 211
a +-+-,
bee
b +-+-,
d
c +-+-,
d a
d +-+-
a b
co it nhftt m('>t s6 khong nho hdn 3.

Vo Qu6c Ba Can - NguySn M<;lnh Linh 51


CHl1dNG 2. cAc DE ToAN ON TAp

BiJi 2 (1.5 (Mm). Giai phuong tdnh:


./(x 2 + 2X)2 + 4(x + 1)2 - ./x 2 + (x + 1)2 + (x 2 + x)2 = 2017.
BiJi 3 (3.0 diim).
a) Tim tfit ca cae s6 nguyen duong a, b, c, d thoa man

{::
a = d
~::
+ 98.
b) Tim tfit ca cae s6 tht;ie x sao eho trong 4 s6
1 1
x - .J2, x2 + 2.J2, x--,
x
x+-
x
eo dung rn('>t s6 kh6ng phfti Ut s6 nguyen.
BiJi 4 (3.0 diim). Cho duong tron (0), ban kinh R va di€rn M n~m phia ngoiti
duong tron (0). Ke cae ti~p tuy~n M A, M B Wi duong tron (0) (A, B lit
eae ti~p di€rn). Tren dOC;ln th~ng A B lfiy di€rn C (C khae A, C khae B). GQi
I, K l~n lU<;'t lit trung di€rn eua M A, Me. Duong th~ng K A dt duong tron
( 0 ) tC;li di€m tha hai D.
a) Chang minh r~ng K0 2 - K M2 = R2.

b) Chang minh r~ng tt1 giae BCD M la ta giae n('>i ti~p.


c) G9i E lit giao di€rn tha hai eua duong th~ng M D voi duong tron (0) va N
lit trung di€m eua K E . Duong th~ng K E dt duong tron ( 0) tC;li di@m thu
hai F. Chung rninh r~ng I, A, N, F eung n~rn tren rn('>t duong tron.

BiJi 5 (1.0 diim). Xet hlnh ve sau:

B ~------~--------~C
D

52 Vo Qu6e Ba Cftn - NguySn MC;lnh Linh


1. cAc DE THI TUYEN SINH CHINH THUC

Ta vi6t cac s5 1, 2, 3, . . . , 9 vao vi tri clla 9 di€m trong hlnh ve tren sao
cho m6i s5 chi xuftt hi~n dung mQt Ifin va t6ng ba s6 tren m6i c~nh clla tam
giac b~ng 1 8. Hai cach vi6t dUQc gQi 180 nhu nhau n6u bQ s6 vi6t (j cac di€m
(A, B, C, D, E, F, G, H) clla m6i cach 180 trung nhau. Hoi co bao nhieu
cach vi6t phan bi~t? T~i sao?

1.17. D~ thi tuy€n sinh THPT chuyen KHTN, 2017 (vong 1)


Bai 1 (3.5 diim).
a) Giiii h~ phuong trlnh:

~X2+y2_xy = 1,
lx+x 2Y=2 y 3.

b) Gilli phuong trlnh:

2 (x + 1) J x + 1 = ( Jx + 1+ J1- x ) (2 -'- J1- X2) .

Bai 2 (2.5 diim).


a) Chung minh r~ng kh6ng t6n t~i cac s5 nguyen x, y thoa man d~ng thuc

12x2 + 26xy + lSy2 = 4617.


b) VCli a, b 180 cac s5 thvc duong, tIm gia tri lCln nhftt clla bi€u thuc

Bai 3 (3.0 diim). Cho hlnh thoi ABC D VCli L BAD < 90 0 • Duong tron tam
I nQi ti6p tam giac A B D ti6p xuc vCli B D, B A Ifin luQt t~i J, L. Tren duong
th~ng L J, Ifty di€m K sao cho B K song song vCli / D. Chung minh r~ng
a) LeBK = LAB/.
b) KC J.. KB.
c) B6n di€m C, K, /; L cung n~m tren mQt duong tron.
Bai 4 (1.0 die~). TIm tftt cll cac hQp s5 nguyen duong n sao cho t6n t~i mQt
cach s£p x6p cac s5 1, 2, . . . , n thanh a 1, a 2, . . . , an ma khi chia cae s6
a 1, a 1 a 2, . • • , a 1 a 2 ••• an cho n , ta dUQc cac s5 du d6i mQt khac nhau.

Vo Qu5c Ba Cffn - Nguy~n M~nh Linh 53


CHVdNG 2. cAc HE ToAN ON TAp

1.18. D€ thi tuy@n sinh THPT chuyen KHTN, 2017 (vong 2)


Ba; 1 (3.5 diim).
a) Giili h~ phu'ong trinh:
~x+y=/x+3y,
~X2+y2+xy=3.
b) Voi a, b Ut cac s6 du'ong thoa man a b + a + b = 1, chung minh r~ng
a b 1 + ab
----:-+
2 = .
l+a l+b 2 /2(1+a 2 )(I+b 2)

Ba; 2 (2.5 c1ie'm).


a) Tim dt eft cac c~p s6 nguyen t6 (p, q) thoa man d~ng thuc
pep - 1) = q(q2 - 1).

b) Voi a, b, c Ul cac s6 tht;Cc du'ong thoa man a b + be + c a + abc = 2,


tim gia tri IOn nhftt cua bigu thuc
a+l b+l c+l
M = 2 + 2 + ----::-----
a + 2a + 2 b + 2b + 2 c 2 + 2c + 2
Ba; 3 (3.0 c1ie'm). Cho tam giac ABC nh9n voi A B < A C. E, F l~n lu'<;lt Ht
trung digm cua cac c~mh C A, A B. Du'dng trung trt;Cc cua dO<;tn th~ng E F dt B C
tl;li D. Giii sli' co digm P n~m trong goc E A F va nfun ngoai tam giac A E F sao
cho L. PEe = L. D E F va L. P F B = L. D FE. P A c~t du'dng tron ngol;li tiSp
tam giac P EF t<;ti Q khac P.
a) Chungminhr~ng L.EQF = L.BAC + L.EDF.
b) TiSp tuYSn t<;ti P cua du'dng tron ngo<;ti tiSp tam giac P E F dt cac du'dng
th~ng C A, A B l~n lu'<;lt t<;ti M, N. Chung minh r~ng b6n digm C, M, B,
N cling n~m tren m9t du'dng tron, g9i du'dng tron nay la (K).
c) Chung minh r~ng (K) tiSp xuc voi du'dng tron ngo<;ti tiSp tam giac A E F .
Ba; 4 (1.0 c1iifm). Cho s6 nguyen n ~ 5. Xet m9t da giac 16i n c<;tnh. Ngu'di ta
mu6n ke m9t s6 du'dng cheo cua da giac sao cho cac du'dng cheo nay chia da
giac da cho thanh dung k mi~n, m6i mi~n la m9t ngu giac 16i (hai mi~n bftt ky
khong co digm trong chung).
a) Chung minh r~ng ta co thg tht;Cc hi~n du'Qc voi n = 20 1 8 va k = 672.
b) Voi n = 201 7 va k = 672 thl co thg tht;Cc hi~n du'Qc khong? Hay giiii thich.

54 VO Qu6cBa Cffn - Nguy~n M<;tnh Linh


1. cAc HE THI TUYEN SINH CHfNH THUC
" ?
1.19. De thi tuyen sinh THPT chuyen, sa GD-DT Ha NQi,
2017 (dfmh cho chuyen Toan)
Bai 1 (2.0 dim).
a) Giiii phlfdng trinh:
,v6x - x 2 +2X2 - 12x + 15 = O.
b) Giiii h~ phuong trinh:

4X2 = Y + ~'
{
4y2 = X +-.
x
Bai 2 (2.5 die'm).
a) Cho s6 nguyen t6 p > 3. Chang minh r~ng 2017 - p2 ehia h€t eho 24.

b) Tim tfrt eii cae e~p s6 nguyen duong (x, y) thoa man
x 3 + y3 - 9xy = O.
c) Cho a, b, c Hl cae s6 nguyen duong. Chang minh r~ng
a + b + 2,v ab + c 2
kh6ng phiii la s6 nguyen t6.
Bai 3 (1.5 die'm). Cho cae s6 tht;te duong x, y, z thoa man x 2 + y2 + Z2 = 3.
Chung minh r~ng
x y z 3
---+
3-yz 3-zx
+ 3-xy ~-.
2
Bai 4 (3.0 diim). Cho tam giae nh9n ABC voi A B < A C , nQi ti€p dUdng tron
( 0 ). G9i I la tam dUdng tron nQi ti€p tam giae ABC, D la hinh ehi€u ella di€m
I tren dUdng th~ng B C va G la giao digm tha hai ella dUdng th~ng A D voi dudng
tron ( 0). G9i F Ul digm ehinh giua eung IOn B Cella dUdng tron ( 0 ). Dudng
th~ng F G e~t dUdng th~ng I D t~i digm H.
a) Chang minh r~ng ta giae I B He Ia ta giae nQi ti€p.

b) G9i J la giao digm tha hai ella dUdng th~ng A I voi dUdng tron ngo~i ti~p
tam ghie B Ie. Chang minh r~ng B H = C J.

c) G9i N la giao digm tha hai ella dUdng th~ng F H voi dUdng trcm ngo~i ti~p
tam giae B Ie. Chang minh r~ng N J di qua trung digm ella Be.

Vo Qu6eBa Can - Nguy~n M,;mh Linh 55


CHVdNG 2. cAc HE ToAN ON TAP

Bai 5 (1.0 diim). Xet t~p hQp S g6m eae s6 nguyen duong eo tfnh ehfit: Vdi hai
phdn ta phfin bi¢t bdt kY x, y thw?c S , ta lu6n co

30lx - yl ;? xy.

Hoi t~p hQp S eo th€ eo nhi~u nhfit bao nhieu phfin tfr?

, ?

1.20. De thi tuyen sinh THPT chuyen, SO' GD-DT Ha NQi,


2017 (danh cho chuyen Toan-Tin)
Bai 1 (2.0 diim).
a) Gilli phuong trlnh:

J 5x - x 2 + 2X2 - lOx +6= O.

b) Giiii h~ phuong trlnh:

Bai 2 (2.5 diim).


l X+ Y +X Y =3,
Jx + Jy = 2.

a) TIm tfit ell eae s6 nguyen duong x, y, z thoa man

+y
j x -z = 2,
3x 2 +2y2_Z2= 13.

b) Cho eae s6 nguyen duong a, b, c thoa man a 2 + b2 = C 2. Chung minh


r~ng a b ehia h~t eho a + b + c.

c) TIm tfitca eae s6 tu nhien n thoa man 2 n + 1, 3 n + 1 Ia eae s6 ehfnh


phuong va 2 n + 9 Ia s6 nguyen t6.

Bai 3 (1.5 diim). Cho eae s6 thl,ie duong a, b, c thay d6i Iuon thoa man
1 1 1
2"
a
+ b2 + 2"
c = 3.

TIm gia trj IOn nhfit eua bi€u thue


i l i
P =
(2a + b + C)2
+ (2b + c + a)2 +----~
(2c + a + b)2 .

56 V6 Qu6e Ba efn - Nguy~n M?nh Linh


2. cAc HE ON TAP THEM

BfJi 4 (3.0 diim). Cho tam giac nhQn ABC vdi AB < AC, nQi ti€p dlidng
tron ( 0). GQi D Hl trung di~m clla c<;lnh B C, E lil hlnh chi€u Clla di~m A tn3n
c<;lnh B C va H la tn;ic Him clla tam giac ABC. Dlidng· th~ng A D dt dlidng
tron ( 0) t<;li di€m thu hai F.

a) Chung minh BC 2 = 4· DA . DF.

b) Tia D H dt dlidng tron (0) t<;li di~m G. Chung minh r~ng b6n di~m
A, G, E va D cling thuQc mQt dlidng tron.

c) Dlidng th~ng F E c~t dlidng tron (0) t<;li di~m thu hai K. Chung minh r~ng
dlidng th~ng B C ti€p xuc voi dlidng tron ngo<;li ti€p tam giac G K E .

BfJi 5 (1. 0 diim). Ta vi€t len blmg 99 86 tl;i nhien lien ti€p 1, 2, ... , 99. Ta
thl;ic hi~n thao tac 8au: Xoa ba 86 a, b, c b§t ky tren bang r6i l<;li vi€t len bang 86
abc + a b + be + c a + a + b + c. Ti€p t1.).C thl;iC hi~n thao tac tren cho d€n
khi tren bang con l<;li dung mQt 86. TIm 86 con l<;li do.

"

BfJi 1 (2.0 diim).


a) Giai phlidng trlnh:
.v15x +5+ ,J5x 2

X3 + y3 + xy = 2x + 4y - 1,
{ xy+x+2y=1.

BfJi 2 (2.5 diim).

a) Cho 86 nguyen dlidng n la mQt l~p phlidng dung. Chung minh r~ng
n2 + 3n + 3
kh6ng la l~p phlidng dung (L{2p phuong dung lit s6 co dr;mg a 3 voi a nguyen).

b) Chung minh r~ng voi mQi 86 nguyen a , t6ng


(a + 1)2 + (a + 2)2 + ... + (a + 99)2
kh6ng th€ vi€t dliQC dlioi d<;lng hly thua IOn hdn 1 clla ffiQt 86 nguyen dlidng.

Vo QU6c Ba Cffn - Nguy~n M<;lnh Linh 57


CHu'dNG3 I
LLOI GIAI cAe DE TOM ON TAP 19

"Each problem that I solved became a rule, which


served afterwards to solve other problems. "
- Rene Descartes
" ?

1. Cac de thi tuyen sinh chfnh thli'c


1.1. D~ thi tuy~n sinh THPT chuyen DHSP Ha NQi, 2014

Bai 1 (1.5 diim). Giasita, b, C, x, y, zUlcacs6tht!ckhacOtheamand6ng


thai cac di~u kien
.
~x + !!.y + £z = 0 va ~a + ,E.b + ~c = 1. Chung minh r~ng

La. giiii. D~t X = ~, Y = t, Z = ~. Ttl' gia thi€t, ta co X + Y + Z = 1 va


o -- lx + lY + lZ -_ xy +yz+zx
XYZ '
suy faXY + YZ + ZX = O. Do do
X2 + y2 + Z2 = (X + Y + Z)2 - 2(XY + YZ + ZX) = 1.
Ta co di~u phai chung minh. D

Bai 2 (1.5 dMm). TIm tAt ell cac s6 tht!c x, y, z thea man
xJl- y2 + yv'2- Z2 + zv'3 _X2 = 3.

Uti giiii. Di~u ki~n: 1 - y2 ~ 0, 2 - Z2 ~ 0 va 3 - X2 ~ O. Sit d\lng bAt d~ng


thuc AM-GM d<;lng ab ~ ¥,
ta co
3 = xJl- y2 + yv'2 - Z2 + zv'3 - x 2
x 2 + 1 - y2 y2 + 2 - Z2 Z2 + 3 - x 2
~
2 2 2
+ +----
= 3.

Vo Qu6c Ba Cffn - NguySn M~nh Linh 93


CHVdNG 3. UJI GIAI cAc HE ToAN ON TAp

Til do suy fa, d~u d~ng thuc phaixay ra, hay ta phai co

x = )1- y2,

I Y = V2-z 2 ,
z=V3-x 2 .
Gicii M nay, ta dUQc x = 1, Y = 0 va z = -J2. V~y phudng trinh da cho co nghi~m
duy nhfit (x, y, z) = (1, 0, -J2) . 0

Nh~n xet. Bai nay cling co the gicii b~ng cach d~t fut ph\! va bi~n d5i h~ng d~ng thuc.

Bai 3 (1.5 diim). Chung minh r~ng vdi s6 nguyen dUdng n ~ 6 thi s6

2·6·1Q .. ·(4n -2)


an = 1 + - - - - - - - -
(n + 5)(n + 6) .. · (2n)

Ia mQt s6 chinh phudng.

LCii gilti. Ta co
2n [1 . 3 . 5 ... (2n - 1)]
an = 1 + ----''--------''-
(n + 5)(n + 6) .. · (2n)
2n • (2n)!
= 1 + -=-------=---------
[2.4.6 ... (2n)](n + 5)(n + 6) .. · (2n)
= 1+ (2n)!
n!(n + 5)(n + 6) .. · (2n)
= 1 + (n + 1)(n + 2)(n + 3)(n + 4).
M~t khac, ta cling co
1 + (n + 1)(n + 2)(n + 3)(n + 4) = 1 + (n 2 + 5n + 4)(n 2 + 5n + 6)
= (n 2 +5n+5)2.
Dodo
an = (n 2 + 5n + 5)2, "In ~ 6.
V~y an Iuon Hl s6 chinh phudng vdi mQi n ~ 6. 0

Bai 4 (1.5 diim). Choa, b, e > Othoamanabe = 1. Chungminhriing

1 1 1 3
- ---+
ab + a + 2 be + b
+
+ 2 ea + e + 2
~-.
4

94 Vo Qu6e Ba efn - Nguyen M?nh Linh


1. cAc HE THI TUYEN SINH CHINH THUC

LCii giai. BM toan nay co ba cach ti€p c~n nhu sau.


Cach 1. Sa dl.lng b~t d~ng thl1C Cauchy-Schwarz d,;mg cQng mfru, ta co
1 1 1( 1 1)
ab +a+2 = (ab + 1) + (a + 1) ~"4 ab +1+ a +1
="4
1 (abe
db + abc
1)
+ a + 1 ="4
1 (e+ e 1 +a+
1)
1 .
Chl1ng minh tUdng 111, ta ding co

1 1( a 1)
be + b + 2 ~"4 a +1+b+ 1
va
1
----~-1( b + 1) .
ea +e +2 4 b+1 e + 1
CQng cac b~t d~ng thl1C tren l;;ti theo v€, ta thu dUQc di€u phiii chl1ng minh. D~u
d~ng thl1c xiiy ra khi va chi khi a = b = e = 1. •
Cach 2. Trudc h€t, ta se chl1ng minh
1 1 1
~---+ + = 1. (1)
ab + a + 1 be + b +1 ea + e +1
Th~t v~y, do a be = 1 nen ta co

1 abc a
be +b+ 1 be + ab 2 e + abc - ab + a + 1 '
1 abc ab
ea +e+ 1 - ea + e + abc - ab + a + 1 .
Dodo
1 1 1 1+a + ab
- - - - = 1.
-a-b-+-a-+-1 + be +b+1+ ea +e +1 ab + a + 1
Bay gib, sa d\mg b~t d~ng thl1c Cauchy-Schwarz d;;tng cQng mfru, ta co
2
1 1 (3 12) 1 (9 )
ab + a +2 ~ 16 ab + a +1 +T = 16 ab + a +1+1 .
Chung minh tUdng tl;l, ta ding co

be
1 1(9 + b + 2 ~ 16
)
be + b + 1 + 1 ,
1 1(9
----~-
ea+e+2 ea+e+1
+1 ) .
16
CQng cac danh gia tren l;;ti va sa d\mg (1), ta thu dUQc di€u phiii chl1ng minh. •

Vo Qu6c Ba Cffn - Nguy~n M;;tnh Linh 95


CHudNG 3. LOI GIAI cAc DE ToAN ON TAP

each 3. Do a, b, e > 0 va abe = 1 nen ta co th€ di;it a = ~, b = ~ va


e = ~ voi x, y, z > 0 (cae s6 x, y, z nay luon t6n t~i, ch~ng h~n ta co th€
chQn x = a, y = 1, z· a b). Khi do, ta co
1 1 y =1- x+y+z.
ab + a +2 2y + z + x 2y + z + x

Tltdng tv, ta cling co

1 =1- x+y+z,
be +b+ 2 2x + y + z
1 =1_x+y+z.
ea + e + 2 2z + x + y
Do do, bftt d~ng thuc dn chung minh tu'dng du'dng voi

3-(x+y+z) ( 1 + 1 + 1 ) :::; ~. (2)


2x +y+z 2y +z+x 2z +x +y 4
Sa dl,lllg bfrt d~ng thuc Cauchy-Schwarz d~ng cQng m§:u, ta co
1 1 1 9
- ---+
2x+y+z 2y+z+x
+ 2z+x+y ~
4(x+y+z)
.
TIt do suy ra
9 3
VT(2):::;3-(x+ y +z)·4( )
x+y+z 4

Ta co di€u phiii chung minh. o

Bfli 5 (3.0 diim). Cho hinh vuong ABC D vdi tam O. GQi M la trung di€m
cua c~h A B. Cae di€m N, P theo thu tv thuQc cac c~h Be, CD sao cho
M N II A P. Chung minh r~ng

a) Tam giac B N 0 d6ng d~g vdi tam giclc DO P va L N 0 P = 45 0 •

b) lam dU'dng tron ngo~ ti8p tam giac NO P thuQc 0 c.


c) Ba dU'dng th~ng B D, AN, PM d6ng quy.

La. giai. a) Do NB I AD, BM I DP, MN II PA nen tamgiac NBM


d6ngd~ng voi tam gicic AD P. Suy ra
BN BN DA DO
BO ~BM - ~DP = Dp·
96 VO Qu6c Ba C§:n - Nguy~n M~nh Linh
1. cAc HE THI TUYEN SINH CHINH THUC

K@t h9P vdi L N B 0 = L PD0 = 45°, suy ra 1:,. B N 0 r-v 1:,. D 0 P. Do do

LNOP = 180° - LNOB - LPOD


= 180° - LNOB - LONB
= LNBO
= 45°.

A M B

D P C

b) Vi 1:,. B NO", 1:,. D 0 P va B 0 = D 0 nen

ON BO DO
OP DP DP'
M~t khae, LN0 P = LNB0 = 45°, suy ra 1:,. 0 N P r-v L::. D 0 P r-v 1:,. B NO.

G<;>i Q la Him dUC5ng tron ngo<;ti ti@p tam giae 0 N P. Chli Yr~ng tam giae 0 N P
d6ng d<;tng vdi tam giae B NO, ta co
0
L.QON = 180 -LOQN = 90 0 -LOPN
2
= LCOB - LBON = LCON.
Do do tia 0 Q trimg vdi tia 0 N. V~y Q thuQe 0 C.
c) G<;>i E, F theo thu t1.,t la giao di€m eua B D vdi M N va P A. Chli Yding
1:,. N B M r-v 1:,. A D P va B D la dllong eheo eua hlnh vuong ABC D, ta co

BM DP SDFP FP
BN DA SDFA FA'

K@t h9P vdi MN II A P, thee blS d€ hlnh thang, B D, AN, PM d6ng quy. 0

Vo QU6e Ba Cffn - Nguy~n M<;tnh Linh 97


CHVdNG 3. LOI GIAI cAc HE ToAN ON TAp

Bai 6 (1.0 c1ie'm). Co bao nhieu t~p con A ella t~p h<Jp {I, 2, ... , 2014}
2
thoa man: A co it nhdt 2 phdn ta WI ntu x E A, yEA, x > y thi /_ yEA.

Loi giiti. Voi Ala m9t t~p con ella t~p h<Jp {I, 2, ... , 2014} thoa man yeu diu
bai toan, gQi a la phfin tti' nho nh~t ella A. Xet b E A, b =1= a, ta co b > a va
ba~a EA. Do a la phfin tti' nho nh~t ella A nen ba~a ~ a, suy fa
b ~ 2a. (1)
G<;>i c, d Ht hai phfin tti' ldn nh~t trong A, c < d. Tit (1), ta co

d ~ 2a ~ 2c. (2)

Theo gia thi~t, dC~c EA. M~t khae, do (2) nen J~c ~ 2~~C ~ c, suy fa

c2
d-c E{c,d}.

Xet cae trUdng h<Jp sau .

• Tr~ong hgp 1: dC~c = d. Trong tntdng h<Jp nay, ta co J = -li vls , mau
thuan do c, d E z+ .

• Truong hgp 2: /~c = c. Trong t111dng h<Jp nay, ta co d = 2c. K~t h<Jp voi
(2), ta suy fa c = a va d = 2a.
Do do A = {a, 2a} voi a = 1,2, ... , 1007. Cae t~p h<Jp tren d€u thoa man yeu
e~u bai toano V~y co t~t ea 1007 t~p h<Jp thoa man. D

1.2. D~ thi tuy~n sinh THPTchuyen KHTN, 2014 (vong 1)

Bai 1 (3.0 cJiim).

a) Gilti phuong trlnh:

b) Gilti h~ phuong trinh:

X2 - xy + y2 = 1,
1 x 2 + xy + 2y2 = 4.

98 Vo Qu6c Ba C§:n - Nguy~n M~nh Linh


1. cAc DE THI TUYEN SINH CHfNH THUC

La. gicii. a) Di@u ki~n: -1 ~ x ~ 1. Ta co

(JX+1 + v'1=X)2 = 2 + 2J1 - x 2.

Do do, phu'dng trinh da cho tu'dng du'dng voi

(JX+1 + v'1=X) 3 = 8,

hay
JX+1 + v'1=X = 2.
GiM phu'dng trinh nay, ta du'Qc x = O. V~y phu'dng trinh co nghi~m duy nhAt x = O.
b) Tit h~ phu'dng trlnh da cho, ta co

4(x 2 - xy + y2) = x 2 + xy + 2y2.


Chuygn v~ va ph§n tich thanh nhan tit, ta du'QC

(x - y)(3x - 2y) = O.

Suy ra x = y ho~c y = ~ x .
• Voi Y = x, thay vao phu'dng trinh dftu clla h~, ta du'Qc x 2 = 1 hay x = ± 1.
Suyrax = y = ±1.

• Voi y = ~ x, thay vao phu'dng trinh dftu clla h~, ta co x 2 - ~ x 2 + ~ x 2 = 1.


Suy ra x = ± 5?
va y = ± ~.

V~y h~ phu'dng trlnh da cho co 4 nghi~m (x, y) la

(1, 1), (-1, -1), (~, ~), (- ~, - ~). D

Bfti 2 (3.0 c1ilm).

a) Gia sit x, y, z la ba s6 tht;tc du'dng thoa man di€u ki~n x +y +z = xy z.


Chung minh r~ng

x 2y 3z xyz(5x + 4y + 3z)
- - + 1 + y2 + 1 + Z2 -
1 + x2 (x + y)(y + z)(z + x)
.

b) Tim tftt ca. cac nghi~m nguyen clla phu'dng trinh

Vo Qu6c Ba Cttn - Nguy~n M~nh Linh 99


CHUONG 3. LOI GIAI cAc HE ToAN ON TAp
La. giai. a) Dat
. a = .1,
x b = .1 z Til' gia thi~t ta co ab + be + ea = 1.
y va e = 1.
D~ng thlic cfin chdng minh tuong duong v(ji
a 2b 3e Sbe + 4ea + 3ab
l+a2 + l+b2 + 2
l+e = (a+b)(b+e)(e+a)' (1)

Vai chu y 1 + a2 = (a + b )(a + e) va cac k~t qua tuong tl;i, ta co


a 2b 3e
VT (1) = + + -----
(a + b)(a + e) (b + e)(b + a) (e + a)(e + b)
a(b + e) + 2b(e + a) + 3e(a + b)
(a + b)(b + e)(e + a)
3ab + 4ea + Sbe
- (a + b)(b + e)(e + a)'
Ta co di~u phai chdng minh.
b) D~t S = x + y va P = x y, phuong trinh da cho tres thanh
SP2 + S = 3 + P,
hay
3+P
S = 1 + p2'
Do x, Y E Z nen S E Z, suy ra 3 + P chia h~t cho 1 + p2. Til'day, ta co p2 - 9
chia h~t cho p2 + 1, hay 10 chia h~t cho p2 + 1.
Lfin lu<Jt xet cac uac nguyen duong cua 10 d€ xac dinh P, S r6i tim nghi~m nguyen
(x, y) tuong ling, thu Il;li v(ji phuong trinh ban dfiu. Phuong trinh co ba nghi~m
(x, y) la (3, 0), (0, 3) va (1, 1). 0

Bai 3 (3.0 diim). Cho tam giac ABC nhQn vai AB < BC. D la di~m thuQc
cl;lnh BC sao cho AD la phan giac cua L.BAC. Dudng th~ng qua C song song
vai AD c£t trung tqtc cua A C tl;li E. Dudng th~ng qua B song song vai AD c£t
trung tnlc cua AB tl;li F.

a) Chdng minh r~ng tam giac ABF d6ng dl;ing vai tam giac ACE.

b) Chdng minh r~ng cac dUdng th~ng BE, C F, AD d6ng quy tl;li mQt
di€m, gQi di€m do la G.

c) DuOng th~g qua G song song vai A E dt duong th~g B F tl;li Q . D1idng
th~g Q E c£t duOng tron ngol;li ti~p tam giac G E C tl;li P kMc E. Chung
minh rrmg cac di~m A, P, G, Q, F cung thuQc mN dUdng tron.

100 Vo Qu6c Ba Cffn - NguySn Ml;lnh Linh


1. cAc HE THI TUYEN SINH CHINH THUC

Uti giai. a) Ta co cac tam giac A B F va ACE lfin h1Qt din t<;li F, E va

LFBA = LBAD = LCAD = LECA.

Do do b. A B F '" b. ACE. Ta co di8u phai chling minh.


A

Q
" .......
,"G~ ............... . .

",
"
",
"" '~
, t
~\
................
................
......
................
" ,,\ ...
" \ '\

B D····· ". ..... C


.................................... ..,--'

b) Gia sit G 1a giao diem cua BE va C F. Ta co


GF BF AB DB
GC CE AC DC'
Suy ra G D I F B . M~t khac, ta cling co F B II AD nen G E AD. V~y ba dliong
th~ng BE, C F va AD d6ng quy t~i G.
c) Ta co
LBQG = LQGA = LGAE = LGAC + LCAE
= LGAB + LBAF = LGAF.

Suy ra ta giac A G QF nQi ti~p. (1)


M~t khac, L QP G = L G C E = L G F Q nen ta giac F Q G P nQi ti~p. (2)
TIt (1) va (2) suy ra cac diem A, P, G, Q, F cung thuQc mQt dlidng tron. Ta co
di8u phai chling minh. 0

Bai 4 (1.0 diim). Gia sit a, b, c la cac s6 tht;ic dlidng thoa man d~ng thuc
a b + be + c a = 1. Chang minh r~ng

Vo QU6c Ba Cffn - Nguy~n M~nh Linh 101


CHUONG 3. LaI GIAI cAc HE ToAN ON TAp
La. giai. Bai toan nay co hai cach tiSp c~n nhl1 sau.
each 1. Sa dt.lng bfit d~ng thlic AM-GM, ta co

1 1 Va6b3e3
a 4 b 2 + -abe 2 + -ae >: 3 = a 2be.
3

3 9:;--- 27

Tl10ng tv, ta cling co

1 1
b 4 e 2 + -bea
3
2 + -ba >: b 2ae
9:;---
4 2 1 1 2
e a + 3eab2 + g eb ~ e ba.

CQng ba bfit d~ng thlic tren l'.li theo vS, ta thu dl1Qc

hay
1 2
4
a b
2
+ b4 e 2 + e4a 2 + 9 ~ 3 abe (a + b + e). (1)
Mat khac, ta cung co

· 1 1
abe (a + b + e) = a b . a e + be· b a + ea· e b ::;; - (a b + be + ea) 2 = -3.
. 3
Dodo
4 4
3 abe (a +b +e)::;; 9. (2)
Tit (1) va (2), ta co

D~ng
.
thlic xay ra khi va chi khi a =b =e = ~.
v3 •
each 2. Sa d1;lng bfit d~ng thlic AM-GM, ta co

Do do, ta chi cfin chling rninh

102 Vo Qu6c Ba Cfln - NguySn M'.lnh Linh


1. cAc HE THI TUYEN SINH CHINH THUC

Bfit d~ng thac tren tlidng dlidng vdi


4
a 2b
2
+ b 2c 2 + c 2a 2 + 3a 2b 2c 2 ;;:: 9'
D~t x =be, y == c a , Z = a b, tit gia thi€t ta co x, y, Z > 0 va x + y + Z = l.
Ta cfrn chang minh
4
x 2 + y2 + Z2 + 3x Y Z ;;:: 9'
D€ Yr~ng, trong ba s6 x, y, Z luon co hai s6 cung khong nho hdn ho~c cung t
khong Wn hdn t.
Khong mfit tinh t6ng quat, ta co th€ gia sil' hai s6 do la x va y .
Khi do, ta co (x - t)
(y - t) ;;::
0, suy ra
1 1 1 1 2 1
xy ;;:: 3(x + y) - 9 = 3(1 - z) - 9 = 9- 3z.
Dodo
2 2
x2 + y2 + Z2 + 3xyz ;;:: x 2 + y2 + Z2 + -z _ Z2 = x 2 + y2 + -z.
3 3
M~t khac, sil' dl,mg bfit d~ng thac AM-GM, ta l~i co
2 1 2 2 1 2
x +9;;::3 x , y +9;;::3 Y '
K€t hQp vdi danh gia d tren, ta thu dliQC
222 21212224 D
x +y +z +3xyz;;::3x-9+3Y-9+3z=3-9=9'

1.3. O~ thi tuy€n sinh THPT chuyen KHTN, 2014 (vong 2)

Bai 1 (3.5 di§m).

a) Gia sil' x, y la cac s6 thl,l'C dlidng phan bi~t thoa man

Y 2y2 4y4 8y 8
-----'--+
x +y x2 + Y
2+ X4 + y 4+ x 8 - y 8=4.

Chang minh r~ng 5 y = 4 x .

b) Giai h~ phlidng trlnh:

2x 2 -3 y2 +x y = 12,
1 6X+X2y = 12+6y+y2 X.

Vo Qu6c Ba Cffn - Nguy~n M~nh Linh 103


CHlJdNG 3. LOI GIAI cAc HE ToAN ON TAp

LO'i giai. a) D~ thfiy d~ng thuc sau dung vdi mQi a =I- ±b:

b b
a+b a-b
Suyra
b b 2b 2
a+b a - b a2 - b2 •
Do do, d~ng thac da cho tudng dUdng vdi

y
- - - x 22y2
x - Y - y2
+2 ( xy22
_ y2
-2y4)
- X4 - -
_ y4
y4 2y8) 8y 8
+4 ( - + -8- - = 4 ,
x4-y4 X8 - y 8 X _ y8

hay
y
= 4.
x-y
Quy d6ng va rut gQn, ta dUQc 5 y = 4 x . Day chinh la k~t qua dn chung minh.
b) H~ phuong tdnh da cho tuong dudng vdi

i (x - y)(2x + 3y) = 12,


~ (x - y)(6 + xy) = 12.

Til' h~, ta suy fax =I- y va

(x - y)(2x + 3y) = (x - y)(6 + xy).


Dodo
(x-3)(y-2)=O.

• Vdi x = 3, thay van phuong trinh dfru cua h~, ta dUQc 1 8 - 3 y 2 + 3 y = 12,
suy ra y = - 1 ho~c y = 2 .

• Vdi y = 2, thay van phuong trinh dfru cua h~, ta co 2X2 + 2x - 12 = 12,
suy ra x = 3 ho~c x = - 4.

V~y h~ da cho co 3 nghi~m (x, y) la (3, - 1), (3, 2) va ( - 4, 2). 0

104 Vo Qu6c Ba Cffn - Nguy~n M~nh Linh


1. cAc DE THI TUYEN SINH CHfNH TRUC

Bai 2 (2.5 c1ie"m).

a) Cho x, y lit cac s6 nguyen Wn hdn 1 sao cho 4 x 2 Y 2 - 7x + 7y Ia s6


chfnh phUdng. Chung minh r~ng x = y.

b) Gia sit x, y Ia cac s6 thl!C kh6ng am thoa man x 3 + Y 3 + X Y = X 2+ Y 2.


Tim gia tri Wn nhAt va nh6 nhAt cua bi€u thuc

1 + V'x
p = + 2 + V'x .
2 + V'y 1 + V'y

La. giai. a) D~t A = 4x 2 y2 - 7x + 7y, taco


A - (2xy - 1)2 = 4xy -7x + 7y - 1 = x(4y - 7) + 7y - 1 > 0
va

(2xy + 1)2 - A = 4xy + 7x -7y + 1 = y(4x -7) + 7x + 1 > O.


Dodo
(2xy _1)2 < A < (2xy + 1)2.
Ma A la s6 chfnh phl1dng nen A = (2 x y ) 2, tit do suy ra x = y.
b) Tit gia thi€t, ta co

hay
(x 2 - xy
+ y2)(X + y - 1) = o.
Suy ra x 2 - X Y + Y 2 = 0 ho~c x + y = 1 .
• Vdi x 2 - x Y + y2 = 0, ta co x = y = 0 va P = ~.

• Vdi x + y = 1, ta co 0 ~ x, y ~ 1. Do do

1+V'T 2+V'T
P ~ + = 4,
2 + V'o 1 + V'o
trong do dAu d~ng thuc xay ra khi va chi khi x = 1 va y = 0; va

I+VO 2+V'O 4
P ~ + =-,
2+V'1 1+V'1 3
vdi dAu d~ng thuc xay ra khi va chi khi x = 0 va y = 1.
Tit cac tfUdng hQp vita xet d tren, ta di den k€t lu~n max P = 4 va min P = 1. 0

Vo QU6c Ba DIn - Nguy~n M~nh Linh 105


CHUONG 3. UJI GIAI cAc HE ToAN ON TAp

Bai 3 (3.0 diim). Cho tam giac ABC nQi tiep duong tron (0) va di€m P
n~m trong tam giac thoa man P B = PC. D la di€m thuQc q.nh B C (D
khac B va D khac C) sao cho P n~m trong duong tron ngoc;ti tiep tam giac
DAB va duong tron ngoc;ti tiep tam giac D A C. Duong th~ng P B dt duong
tron ngoc;ti tiep tam giac DAB tc;ti E khac B. Duong th~ng P C dt duong
tron ngoc;ti tiep tam giac D A C tc;ti F khac C.

a) Chung minh r~ng b6n di€m A, E, P, F cling thuQc mQt duong tron.
b) Gift su duong th~ng AD dt duong tron (0) tc;ti Q khac A, duong
th~ng A F dt duong th~ng Q C tc;ti L. Chung minh r~ng tam giac
ABE d6ng dc;tng vdi tam giac C L F.

c) GQi K la giao di€m cua duong th~ng A E va duong th~ng QB. Chung
minhr~g LQKL + LPAB = LQLK + LPAC.

~# . . . - - ...... - ..
A ...·························..··············..,

. ...,....• \\

"
/
........
)
'"
...

106 Vo QU6c Ba Cfin - Nguy€n Mc;tnh Linh


1. cAc HE THI TUYEN SINH CHfNH THUC

lCii giai. a) Ta co
L.EAF = L.EAD + L.DAF = L.EBD + L.FCB
= 180 o -L.BPC= 180 o -L.EPF,

suy ra b6n di€m A, E, P, F cung thuQc mQt dlldng tron.


b) Til k€t qua tren, ta suy ra L. A E B = L. L F C . M~t khac, ta l<;ti co
L.FCL = L.FCB + L.BCL = L.PBC + L.BAQ
= L.DAE + L.BAQ = L.BAE.

Do do f::. F C L rv f::. E A B. Ta co di6u phiii chling minh.


c) Do f::. F C L rv f::. E A B nen ta co ~ i = ~ i ' hay
FL·EA=FC·EB.
Chling minh tlldng tv, ta ding co
EK· FA = FC· EB.
Do do, ta co FL· E A = E K . FA, hay
FL EK
FA EA
Suy ra E F II K L . Tit do, ta co
L.QLK =L.ALK - L.ALQ = L.AFE - L.ABE
= L.AP E - L.ABE = L.PAB.
Tlldng tv, ta cling co
L.QKL = L.PAC.
Suyra
L.QKL + L.PAB = L.QLK + L.PAC.
Ta co di6u phiii chling minh. D

Bai 4 (1.0 diim). Cho t~p h<Jp A g6m 31 ph~n til' va day g6m m t~p con cua
A thoa man d6ng thdi cac di6u ki~n sau:

i) M6i t~p thuQc day co it nhftt hai ph~n til'.


ii) N€u hai t~p thuQc day co chung nhau it nhftt hai ph~n til' thi s6 ph~n til' cua
hai t~p nay khac nhau.

Chling minh ding m ~ 900.

Vo QU6c Ba Cffn - Nguy~n M<;tnh Linh 107


CHVdNG 3. UJI GIAI cAc HE ToAN ON TAP
Liti giai. TIt gia thi€t d€ thfiy m t~p con thuQc day la phan bi~t. Vi A co 31 phfin
tll' nen s6 t~p con co dung hai phfin tit cua A la 31~30 . KY hi~u ak (2 ~ k ~ 31)
la s6 cac t~p co dung k phfin tll', n~m trong day da cho, suy ra

Xet mQt t~p h<Jp co k phfin tit thi s6 cac t~p con co hai phfin tit cua t~p do la k (\-1) ,
suy ra ak t~p nay se co ak . k(\-l) t~p con hai phfin tit. Ma theo gia thi€t vdi hai
phfin tit bfit ky cua A thi chung khong thg d6ng thdi thuQc hai t~p co k phfin tit cua
day, suy fa cac t~p con hai phfin tit noi tren la phan bi~t. Tu do ta co

k(k-l) 31·30
2 ak ~ 2 '

hay
1
ak ~ 31 ·30· k(k _ 1)'

Cho k = 2, 3, . . . , 3 1 , r6i cQng cac bfit dfuIg thdc tt1dng tl;l1~i, ta thu du'qc

m = a2 + a3 + ., . + a31 ~ 31 .30. (_1_ + _1_ + ... + __1_)


1.2 2 .3 30 . 31

= 31 .30 . (1 _! + ! _ ! + ... +
2 2 3
_1 _ _
30
1 )
31
= 900.

V~y m ~ 900. Ta co di~u phlti chdng minh. o


, ?

1~4. De thi tuyen sinh THPT chuyen, SO' GD-DT Ha NQi,


2014 (d~mh cho chuyen Toan)

Biti 1 (2.0 diim).

a) GiM phu'dng trinh:

X (5x
3
+ 2) - 2 ( -J2x + 1- 1) = O.

b) GiM h~ phu'dng trinh:

+ 1) - = -3,
I x2(4y
X
2
(X
2
- 12y)
2y
+ 4y2 = 9.

108 Vo Qu6c Ba Cgn - Nguy€n M~nh Linh


1. cAc HE THI TUYEN SINH CHINH THUC

L{ji giai. a) Di~u ki~n: x ~ - t. Phuong trinh da cho co th€ vi~t l~i thanh
5x 4 + ( ~2x + 1 - 1) 2 = O.

Do 5 x 4 ~ 0, (,.j 2 x + 1 - 1) 2 ~ 0 nen phuong trinh tren chi thoa man khi


5X4 = 0,

{ ( ~2x + 1 - 1) 2 = O.

Giiii h~ nay, ta dU<Jc x = O. V~y phUong trinh da cho co nghi~m duy nhftt x = O.
b) H~ phuong trinh da cho co th€ vi~t l~i dudi d~ng
~(X2 - 2y) + 4x 2 y = -3, (1)
~ (x 2 - 2y)2 - 8x 2 y = 9. (2)
Xet (2) + 2 x (1), ta dU<Jc
(x 2 - 2y)2 + 2(x 2 - 2y) = 3,
hay
(x 2 - 2y - l)(x 2 - 2y + 3) = O.
Tit do suy ra x 2 = 2Y + 1 ho~c x 2 = 2Y - 3.
• Vdix 2 = 2y+l,thayvao(I),tadu<Jcx2y = -1,hay2 y 2+ y +l = O.
Phuong trinh nay khong co nghi~m tht;ic .
• Vdi x 2 = 2y - 3, thay vao (1), ta dU<Jc x 2 y = O. Suy ra x = 0 (tudng
ang, y = ~) ho~c Y = 0 (tuong ang, x 2 = -3, vo nghi~m).
V~y h~ phuong trinh da cho co duy nhftt mQt nghi~m (x, y) la (0, ~) . D

Bai 2 (2.5 c1iim).

a) Chang minh r~ng n~u n la s6 nguyen duong thi 25 n +7 n _4n (3n + sn)
chia h~t cho 65.

b) TIm cac c~p s6 nguyen (x, y) thoa man

x 2y + xy - 2X2 - 3x + 4 = o.
c) TImcacbQs6tt;inhien (aI, a2, ... , a2014) thoaman

al + a2 + ... + a2014 ~ 2014 2 ,

I ai + a~ + ... + a~014 ~ 2014 + 1.


3

Vo Qu6c Ba Cfin - Nguy~n M"mh Linh 109


CHUONG 3. LOI GIAI cAc HE ToAN ON TAp

Uti giiti. a) D~t A = 2S n + 7n - 4 n (3n + sn), ta co


A = (2Sn - 12n) - (20n - 7n) (1)
= (2Sn - 20n) - (12n - 7n). (2)

Do 2S n - 12 n chia h€t cho 13 va 20 n - 7 n chia h€t cho 13 nen tu (1), ta suy ra


A chiah€t cho 13. Tlldng tt;i, do 2S n - 20 n chia h€t cho S va 12 n - 7 n chia h€t
cho S nen tu (2), ta cling suy ra A chia h€t cho S. Do (S, 1 3) = 1 nen ta co A
chia h€t cho S . 1 3 = 6 S .
b) Phudng trinh da cho tlldng dlldng v(ji

xy(x + 1) = 2X2 + 3x - 4.
Vi x = 0 khong thea man phlldng trinh nen ta co x =/:.0 va
2x2 + 3x - 4 4
y(x+l)= =2x+3--.
x x
Do Y (x + 1) va 2x + 3 la ca.c s6 nguyen nen tu tren, ta suy ra x la l1(jc 4, tuc
x E {± 1, ± 2, ± 4 }. B~ng cach thli trt;ic ti€p VaG phlldng trinh da cho, ta tim
dll<;1C cac nghi~m (x, y) cua phlldng trinh la ( - 2, - 1 ), (4, 2).
c) Xet A = (a 1 - 20 1 4) 2 + . . . + (a 20 14 - 20 14) 2 . Tu ghlthi€t, ta co

o~ + ... + a~014 - 2 . 2014 . (a 1 + ... + a20l4) + 2014 3


A = ai
~ 2014 3 + 1 - 2.2014.2014 2 + 2014 3 = 1.

Ma A E Z nen ta co A = 0 ho~c A = 1 .
• Tnidng hgp 1: A = O. Trong tr110ng h9P nay, ta co
(aI, a2, ... , a20l4) = (2014,2014, ... , 2014).
Thli li;li, ta th~y thea man .

• Tnidng hgp 2: A = 1. Trong tr110ng h<;1p nay, ta co (aI, a2, ... , a20l4)
thuQC ffiQt trong cac bQ (2013, 2014, ... , 2014), (201S, 2014, ... , 2014)
va cae hoan vi. Thli lI;ti, ta th~y:

o Voi (aI, a2, ... , an) E {(2013, 2014, ... , 2014) va cae hmin vi} , ta
+
co al a2 + ... + a20l4 = 20142 - 1 < 20142 , mau thu§:n.
o Voi (aI, a2, ... , an) E {(201S, 2014, ... , 2014) va cac hoan vi}, ta
co ai + ... + a~014 = 20143 + 2·2014 + 1 > 20143 + 1, mau thu~n.
V~y co duy nh~t mQt bQ s6 thoa man yeu cAu la (2014, 2014, ... ,2014). D

110 Vo Qu6c Ba Cffn - Nguy€n Mi;lnh Linh


1. cAc HE THI TUYEN SINH CHINH THUC

Nh~n xet. Y c) eua b~li toan con co ghii b~ng cach sit dVng bftt d~ng thuc Cauchy-
Schwarz nhl1 sau: Ta co

20144 ::::; + a2 + ... + a2014)2


(al
::::; 2014(ai + a~ + ... + a~OI4) (3)
::::; 2014(2014 3 + 1)
< (20142 + I?
Suy ra
20142 + a2 + ... + a2014 < 20142 + 1,
::::; al
rna al + a2 + '" + a2014 E Z nen al + a2 + ... + a2014 = 20142 . Tli (3) ta thfty

2014 ::::; ai + a~ + ... + a~014 ::::; 2014 + 1


3 3

nen aT + a~ + ... + a~014 E {2014 3 , 2014 3 + I} (do aT + a~ + ... + a~014 E Z) .


• Vdi aT + a~ + ... + a~014 = 2014 3 , ta thfty dftu b~ng trong bftt d~ng thuc
(3) phai xay ra, hay al = ... = a2014 = 2014 .

• Vdi aT + a~ + ... + a~014 = 2014 3 + 1, ta co

al (al - 1) + a2(a2 - 1) + ... + a2014(a2014 - 1) = 2014 - 2014 + 1.


3 2

M6i s6 h~ng d v~ trai Ia m9t s6 ch~n nen t6ng d v~ trai Ia s6 ch~n, trong khi
do v~ phai 1a s6 Ie, mau thu§:n.

Bai 3 (1.5 c1iim). Vdi ba s6 dl1dng x, y, z thay d6i thoa man x +y +z = 1,


tim gia tri Wn nhM cua bigu thuc
x y z
Q=
x + Jx + yz + y + Jy + zx + z + Jz + xy .

LCii giai. Ta se chung minh max Q = 1 d~t du'<;jc khi va chi khi x = Y = z = i.
Dgyr~ng
x + yz = x(x + y + z) + yz = (x + y)(x + z),
y + z x = y (x + y + z) + z x = (y + z) (y + x),
z + x y = z (x + y + z) + x y = (z + x) (z + y),
bftt d~ng thuc Q ::::; 1 tl1dng dl1dng vdi

x + y + z ~1.
x + V(x + y)(x + z) Y + ";(y + z)(y + x) z + ";(z + x)(z + y)

Vo Qu6c Ba Cfrn - Nguy~n M~nh Linh 111


CHU'dNO 3. LOI OIAI cAc HE ToAN ON TAp

D€n day, ta co b5n cach ti€p c~n nhu sau.


each 1. Sa dl;lng b&t d~ng thuc Cauchy-Schwarz, ta co
J(x + y)(x + z) ~ ,JXZ + JxY.
Dodo
x x VX
-x-+-y'r(:=x=+=y=7):::;=(x=+=z=7) :::; x + yIxY + fo = VX + v0' + viz'
Chung minh tl1dng tl!, ta cling co

y ~ v0'
y + J(y + z)(y + x) ~ VX + v0' + viz'
z viz
-z-+-y'r(:=z=+=x=)=(z=+=y=) :::; vx + v0' + viz'

CQng ba b&t d~ng thuc tren l~i, ta


each 2.
thu dl1Qc di6u phru. chung minh.
Sa d\lng b&t d~ng thuc Cauchy-Schwarz, ta co

J(x + y)(x + z) ~x + .ffz.
Dodo
x x
---;=;===;=::;====:=:::; .
x + J(x + y)(x + z) 2x +~
Chung minh tUdng tl!, ta cling co
y ~ y
y + J(y + z)(y + x) ~ 2y + .y'ZX'
z z
---;======:::;
z + J (z + x) (z + y) 2z + yIxY
.
Nhu th€, ta chi dn chung minh
x y z
----+ + :::;1,
2x + ~ 2y + .y'ZX 2z + yIxY
hay
~
h
----+ + ~ :::; 2.
2x + ~ 2y + .y'ZX 2z + yIxY
B&t d~ng thuc tren co thg vi€t l~i thanh

(1- 2x :xJYZ) + (1- 2y ~y.y'ZX) + (1- 2z :~) ~ 1,


112 Va Qu5c Ba C~n - Nguy€n M~nh Linh
1. cAc HE THI TUYEN SINH CHINH THUC

hay
pz + Fx + -JXy
---'---- 1
;?: .
2x + pz 2y + Fx 2z + -JXy
Sit dl:lng bfit dAng thdc AM-GM, ta co
yz yz yz
-~---;?: -----
2x + ffi 2x pz + y z x (y + z) + y z xy + yz + zx
Chang minh tl1dng W, ta cling co

Fx
----;?:
zx
,
2y +~ xy + yz + zx
-JXy xy
---'----- ;?: - - - - -
2z + -JXy xy + y z + zx
CQng ba bfit dAng thdc tren l~i theo v€, ta co di@u phiii chdng minh.
each 3. Sit dl:lng bfit dAng thuc Cauchy-Schwarz d~ng cQng m§:u, ta co

Dodo

M~t khac,
x + ~(x : y)(x + z) " H ~(X+ :~(X+ zJ
sit dl:lng bftt dAng thuc AM-GM, ta cling co
I
+

4x 2x 2x
-;::::====;:=;====:= ~ - - + - -.
.j(x + y)(x + z) x +y x +z
K€t hQp vdi bfit d&ng thuc tren, ta thu dl1Qc

x ~!(1+~+~).
x + .j(x + y)(x + z) 9 x +y x +z
Danh gia tl1dng hl, ta cling co

y ~!(1+~+~)
Y + .j(y + z) (y + x) '" 9 y +z y +x '

z ~!(1+~+~).
z+.j(z+x)(z+y) 9 z+x z+y

CQng ba bfit dAng thuc tren l~i theo v€, ta thu dUQc di@u phiii chdng minh. •

Vo Qu6c Ba Cgn - Nguy~n M~nh Linh 113


CHVdNG 3. LOI GIAI cAc HE ToAN ON TAp

each 4. Sa d\lng bfit d&ng thuc AM-GM, ta co


1 v(x + y)(x + z) - x
v(x+y)(x+z)+x xy + yz + zx
x+y+x+z _ x
~ 2
--.::::
xy + yz + zx
y +z
2(xy + yz + zx)'
Dodo
x + zx
xy
----~=========~
x + V(x + y)(x + z) 2(xy + yz + zx)
.
Chung minh tlidng tt;!, ta cling co
y yz + xy
----~========= ~
y + V(y + z)(y + x) 2(xy + yz + zx)
,
z zx + yz
----r==========~
z + V(z + x)(z + y) 2(xy + yz + zx) .
CQng ba bfit d~ng thuc tren l~i thea v~, ta thu dli<Jc di~u phai chung minh. D

Nh~n xet. sv dt;mg y tlidng clla cach 3 d tren, ta co thg chung minh dli<JC k~t qua
t6ng quat hdn nhli sau: Vdi mri s6 th1!c du(}ng a, b, c, k, ta diu co

a b
-----;:=====+-.--:---;=====
ka + v(a + b)(a + c) kb + V(b + c)(b + a)
c 3
+ ~--.
kc + V(c + a)(c + b) k +2

Bai 4 (3.0 cJiim). Cho tam giac d~u ABC nQi ti~p dlidng tron ( 0), H la
trung digm ella Be. M Ia digm bfit ky thuQC do~n th&ng BH (Mkhae B).
Lfty digm N thuQc do~n th&ng C A sao cho eN = B M. GQi I la trung
digm ella M N.

a) Chung minh r~ng b6n digm 0, M, H, I cung thuQc mQt dlidng tron.

b) GQi P Hl giao digm clla cac dudng th&ng 0 I va A B. Chung minh


r~ng tam giac M N P Hl tam giac d~u.

c) Xac dinh vi tri Clla digm M dg tam giac I A B co chu vi nho nhk

114 Vo Qu6c Ba Cfin - Nguy€n M~nh Linh


1. cAc HE THI TUYEN SINH CHINH TRUC

Llti giai. a) Do H Uttrung di€m cua B C nen 0 H J.. B C. (1)


Ta co f:,OBM = f:,OCN (c-g-c) nen OM = ON, tu do suy ra tam giac
OM N can t~i M. Ma I la trung di€m cua M N nen ta co a I J.. M N. (2)
Tu (1) va (2), suy ra b6n di€m 0, I, H, M cung thuQc du'dng trim du'dng kinh 0 M.

B \" M / H ./ C
'\"" ,/ ."",,#
"'...........,' ,."
. . . . -I............................ .
I
I
I
I
I
I
I

b) Do f:, 0 B M = f:, aCN nen LBO M = L CON, suy ra

LMON = LMOC+LCON = LMOC+LBOM = LBOC = 120°.

Dodo
LPON = 360° - LMON = 1200
2
Taday, ta suy ra L PAN + L paN = 60° + 120° = 180° nen tu giac
AP ON nQi ti~p. Do do

LOPN = LOAN = 30°.

Chung minh tu'dng tl)', ta cling co LOP M = 30° nen

(3)

M~t khac, do P thuQc trung tn;ic cua M N nen

PM = PN. (4)

Ta (3) va (4), ta suy ra tam giac M P N la tam giac d~u.

Vo Qu6c Ba Cftn - Nguy~n M~nh Linh 115


CHVdNO 3. UJI olAI cAc nE ToAN ON TAp

c) Theo k~t qua diu a), ta co L I H C = L 10M = 60 0 = LAB C nen


I H II A B. Suy fa duong th~ng I H c6 dinh. 09i K Ia trung di€m cua A C thl ta
co ba di€m H, I, K th~ng hang (do cling n~m tfen duong th~ng song song voi
qmh AC).

Lfty di€m T d6i xung voi A qua HI. R6 dmg T c6 dinh. Ta co

AI +BI + AB = TI + IB + AB ~ BT + AB = const.

Do do, chu vi tam ghic A I B nha nhftt b~ng B T + A B, d~t dU9C khi ba di€m
B, I, T th~ng hang. Khi do I Ia trung di€m cua B T c6 dinh (theo tinh chftt
duong trung hinh cua tam giac BAT). Suy fa tu giac B M TN Ia hinh hinh hanh
va TN" B C. L~i co B H = K T, B K = H T nen ta co tu giac B H T K Ia
hinh hlnh h~mh va T K " B C. Ttl do N =
K, suy fa M H. = 0

Bai 5 (1.0 diem). Cho bang 0 vuong kich thuoc 3 x n (3 hang, n cQt, n Ia
s6 tl! nhien IOn hon 1) dU9C t~o hdi cac 0 vuong nha kich thuoc 1 xl. M6i 0
vuong nha dU9C to hdi mQt trong hai mau xanh ho~c da. Tim s6 n he nh§.t d€
voi m9i cach to mau nhu th~ Iuon tim dU9C hinh chit nh~t t~o hdi cac 0 vuong
nha sao cho 4 0 vuong nha d 4 goc cua hinh chit nh~t do cling mau.

Loi giai. Co 2 3 = 8 cach to cho m6i cQt.

D D D X X X D X
D D X D X D X X
D X D D D X X X
(1) (2) (3) (4) (5) (6) (7) (8)

mnh chit nh~t 3 x 6 g6m cac cQt til (2) d~n (7) khong co hinh chit nMt 4 goc cling
mau, do do n ~ 7. Xet voi n = 7:

• Tru'ong hQ'p 1: Co ci)t (1) ho(ic ci)t (8). Khong mftt tinh t6ng qmlt,ta
gHl sft co cQt (1).

o N~u trong 6 cN con I~i co it nhftt mQt cQt Io~i (2), (3), (4) thl cQt do va
CQt (1) co hinh chit nh~t thaa man.
o N~u trong 6 cQt con I~i khong co cQt Io~i (2), (3), (4) thi trong 6 cQt
con I~i chi co cac cQt Io~i (5), (6), (7), (8). Theo nguyen Iy Dirichlet, co
it nhftt mN Io~i cQt I~p I~i hai IAn va xuftt hi~n hinh chit nh~t thoa man.

116 Vo Qu6c Ba C~n - Nguy~n M(~mh Linh


1. cAc HE THI TUYEN SINH CHINH THUC

• TrLidng hgp 2: Kh6ng co C9t (1) va C9t (8) . Trong tntbng hQp nay, ta th~y
7 C9t chi thu9c 6 lo<;li c9t tu (2) d€n (7). Theo nguyen Iy Dirichlet, co m9t
IO<;li C9t l~p l<;li hai I~n va xu~t hi~n hlnh chu nh~t thoa man.

V~y ghl tri nho nh~t cua n la 7. o

1.5. D~ thi tuy~n sinh THPT chuyen, Sit GD-DT Ha NQi,


2014 (danh cho chuyen Toan-Tin)

Bai 1 (2.0 diim).

a) Giai phudng trlnh:

5x 4 + 2x + 2 - 2,J2x + I = O.

b) Gild h~ phudng trlnh:

~ x (2y + 1) - y = -3,
l x 2 + y2 - 6xy = 9.

LO'i giai. a) Gi6ng d~ chuyen Tmln a tren.


b) H~ phudng trlnh da cho co th€ vi€t l<;li thanh

x - y + 2xy = -3, (1)


I (x-y)2-4xy=9. (2)

Xet 2 x (1) + (2), ta dUQc

(x-y)2+2(x-y) = 3,

hay
(x - y - I) (x - Y + 3) = O.
Suy ra x =y + 1 ho~c x = y - 3. Xet cac tntbng hQp sau.

• TrLidng hQ'p 1: x = y + 1. Thay VaG phudng trlnh (1) cua h~, ta dUQc
2y(y + 1) = -4.

Phudng trlnh nay vo nghi~m.

Vo Qu6c B<i Cftn - Nguyen M<;lnh Linh 117


CHU'dNG 3. LOI GIAI cAc HE TOAN ON TAp

• TnJO'ng hQ'p 2: x = y - 3. Thay vao phl1dng trinh (1) cua h~, ta dl1Qc

2y(y - 3) = O.

Suy ra y = 0 (tl1dng ung, x = -3) ho~c y = 3 (tl1dng ung, x = 0).


V~y h~ da cho co hai nghi~m (x, y) la (-3, 0) va (0, 3). o

Bfti 2 (2.5 diim).

a) Chung minh r~ng n€u n la s6 nguyen dl1dng thi

chia h€t cho 21.

b) TIm cac c~p s6 nguyen (x, y) thoa man

Sx 2 + y2 - 2xy + 2x - 2y - 4 = O.

c) Chung minh r~ng trong 20 1 4 s6 nguyen dl1dng ai, . . . , a 2014 thoa man
111
-+-+···+--~4
2 2 2:?"
a1 a2 a 2014

luon tim dl1Qc it nhfit ba s6 b~ng nhau.

La. giai. a) D~t A = sn (sn + 3n ) - 2 n (9 n + 11 n). Ta co


A = (2Sn - 18 n ) - (22n - ISn) (1)
= (2Sn - 22n) - (18 n - ISn). (2)

Do 2S n - 18 n : 7 va 22 n - Is n : 7 nen tu (1), ta co A chia h€t cho 7. Tl1dng tv,


do 2S n - 22 n : 3 va 18 n - Is n : 3 nen tu (2), ta ciing co A chia h€t cho 3. Ma
(3, 7) = 1 nen A chia h€t cho 3 . 7 = 21 vdi m9i n nguyen dl1dng.
b) Phl1dng trinh da cho co th€ dl1Qc vi€t l~i thanh
4x 2 + (x - y + 1)2 = s. (1)

Tuday,taco4x 2 ~ Snenx E {O, -1, 1}(dox E Z).

• Vdi x =0, thay vito (1), ta co (y - 1)2 = S. Phl1dng trinh nay kh6ng
co nghi~m nguyen.
118 Vo Qu6c Ba Cffn - Nguy~n M,;mh Linh
1. cAc HE THI TUYEN SINH CHfNH THUC

• Vdi x = 1, thay vao (1), ta co (y - 2)2 = 1. Suy fa y = 3 ho~c Y = 1.

• Vdi x = -1, thay vao (1), ta co y2 = 1. Suy fa y = 1 ho~c Y = -1.


V~y cac c~p s6 nguyen (x, y) dn tim lit (1, 1), (1, 3), (- 1, 1), (- 1, - 1 ) .
c) Gia sli trong cac s6 da cho chi tim du<;!c t6i da hai s6 b~ng nhau. Khi do, ta co
1 1 1
2 + 2 + ... + - 2 - ::::; 2
a1 a2 a 2014
(1-12 + 221 + ... + 10071 2) ' (1)

Mi;it khac, ta ding co


1 1 1 1 1 1
12 + 22 + ... + 10072 < 1 + + ~ + ... + 1006· 1007
N
111
=1+1--+ .. · + - - - - -
2 1006 1007
1
=2---
1007
< 2. (2)

Til (1) va (2), ta suy fa


111
-2+ - + · · · + -2- < 4 '
a1 a 22 a 2014
mau thu§:n vdi gia thi€t da cho. V~y luon t6n tl;li it nh~t ba s6 b~ng nhau. 0

Bfli 3 (1.5 diim). Vdi ba s6 dUdng x, y, z thoa man di~u ki~n x +y+z = 1,
chang minh f~ng

1 - x2 1- y2 1- Z2
---+
x+yz y+zx
+ z+xy ~ 6. (1)

Uti gifli. Bai toan nay co hai cach ti8p c~n nhu sau.
each 1. D6 Yf~ng
2
1- x = (1 - x)(1 + x) = (y + z)(2x + y + z)
va
x + y z = x (x + y + z) + y z = (x + y)(x + z),
taco
1- x2 a (b + c) a a
--- = = -+-,
x + yz be b e
Vo QU6c Ba Cffn - Nguy~n Ml;lnh Linh 119
CHVdNG 3, LOI GIAI cAc HE ToAN ON TAP

trong do a = y + z, b = z + x, c = x + y, Tlidng ttl, ta cling co


1 - y2 b b
- - - = -+-,
y + zx c a
1 - Z2 C c
z+xy=a+b'

Do do, sa dl,mg bftt d~ng thuc AM-GM, ta co

VT (1) = (~ + :) + (~ + ~) + (~ + :) ~ 6,
Dftu d~ng thuc xay ra khi va chi khi a = b = c, hay x = y = z = t, •
each 2. Sa dl,mg bftt d~ng thuc AM-GM dl,mg ab :::;; (a~b)2, ta co

(Y+Z)2 (l-X)2
X + yz :::;; x + 4 = x + --4--
Dodo

l-x 2 4(l-x 2) 4(l-x)


x + yz ~ (1 + X)2 = (1 + x) = 4
(2+ x-I ) ,
1

Tlidng ttl, ta cling co

1- y2
---~4
y+zx
(2
l+y
) -1,

1- Z2 ~ 4(_2
z+xy~
__ 1) .
l+z

C9 n g cac danh gia tren l<;li theo v€ va sa dt;mg bftt d~ng thuc Cauchy-Schwarz
d<;lng c9ng m§:u, ta dliQC

VT (1) ~ 8 1 + l+y
(l+x 1) -
1 + l+z 12
9
~ 8. - 12
3+x+y+z
= 6.
Day chinh la k€t qua cAn phai chung minh. D

120 Vo Qu6c Ba Cffn - Nguy€n M<;lnh Linh


1. cAc DE THI TUYEN SINH CHINH THUC

Bai 4 (3.0 dim). Cho tam giac d~u ABC nQi ti€p du'dng trcm ( 0), H la
trung diSm cua Be. M la diSm bfit ky thuQc do~n th~ng BH (Mkhac B).
Lfiy diSm N thuQc do~n th~ng C A sao cho eN = B M. GQi I Ia trung
diSmcua M N.

a) Chang minh r~ng b6n diSm 0, M, H, I dmg thuQc mQt du'dng tron.

b) Xac dinh vi tri cua diSm M dS do~n th~ng M N co dQ dai nho nhk

c) Khi diSm M thay d5i va thoa man di~u ki~n d~ bai, chang minh r~ng di~n
tich tam giac I A B khong d5i.

LCii giai. a) Xem cau a) d d€ chuyen Toan.

A
/ ........................................,...
, ..
,,,
,,"
,
"...
·,,
· .
..
··· ...
·\ .
·· N i
..
··. ..

b) Chang minh tUdng tv nhu' do~n dftu phftn b) d d~ chuyen Toan, ta co 0 I ..1 M N
va LMON = 120 0 nen

M N = 2M I = 2 . 0 M . sin 60 0 = OM J3.
Khi M chuySn dQng tren B H thi 0 M ~ 0 H vdi dfiu d~ng thac xay ra khi va
chi khi M trung H. Ttf do suy ra

minMN = OHJ3,
d<;lt dUQc khi va chi khi M trung H.
c) Chang minh tU'dng tv phftn c) d d€ chuyen Toan, ta co I H II A B. Ttf do suy ra
dUdng cao h~ ta
1 va ttf H cung vuong goc vdi A B co dQ dai b~ng nhau. Do do,
di~n tich tam giac I A B luon b~ng di~n tich tam giac A H B khong d5i. 0

Vo Qu6c Ba Can - Nguy6n M~nh Linh 121


CHVdNG 3. UJI GIAI cAc HE ToAN ON TAP

Bai 5 (1.0 c1ie"m). Cho t~p hQp A g6m 36 s6 tl1 nhien lien ti~p ttl 1 d~n 36.
Chung minh r~ng trong 25 phfin til' bfit ky cua t~p hQp A luon tim dUQc 3 phfin
til' la 3 s6 doi m9t nguyen t6 cling nhau.

Lai giai. Chia t~p A thanh 6 t~p co d~ng

Ak = {6k + 1, 6k + 2, ... , 6k + 6}
voi k = 0, 1, . . . , 5. Theo nguyen ly Dirichlet thi trong 25 phfin til' cua t~p A
co it nhfit 5 s6 cling thu9c m9t t~pAk. Ta ch9n 5 s6 nay.

• N~u trong 5 s6 do co 3 s6 Ie thl b9 3 s6 Ie lien ti~p nay thoa man yeu cfiu d~ bai.
• N~u trong 5 s6 do co hai s6 Ie va ba s6 ch~n: Trong ba s6 ch~n lien ti~p do co
dung m9t s6 chia h~t cho 3 va cO nhi~u nhfit m9t s6 chia h~t cho 5.

Ta se chung minh s6 ch~n khong chia h~t cho cii 3 va 5 cling voi hai s6 la Ie
ba s6 thoa man yeu du d~ bai. G9i ba s6 do la aI, a 2, a 3. Vi aI, a 2, a 3
thuQC t~p A nen 1 ~ Ia i-a j I ~ 5. Gia sil' co (a i, a j) = p voi
pEN, P > 1. Khi do, ta co p ~ 7 (vi n~u p < 7 thi P = 3 ho~c 5 la
vo lY). D~t a i = m p, a j = n p voi m, n E N *, m f=. n, ta co

V~y aI, a 2, a 3 doi m9t nguyen t6 cling nhau. o

1.6. D€ thi tuy~n sinh THPT chuyen DHSP Ha NQi, 2015

Bfli 1 (2.5 c1ie"m).

a) Cho a ~ 0, a f=. 1. Rut g9n bi€u thuc

s = J6-4~. V2o+ 14~


3/ - [ a-I ]
+v(a+3).J a - 3a - 1 -=- 2(.Ja-1)-1 .

b) Cho
. 0 < x, y < 1 thoa man --x
x1 y1 = 1. Tinh gia tri. cua bi€u thuc
+ --y
p =x +y + .jx 2 - xy + y2.

122 Vo Qu6c Ba Cin - Nguy€n M.;mh Linh


1. cAe HE THI TUYEN SINH CHfNH THUC

Ldi giai. a) D€ y r~ng

J 6 - 4 V2 = J( V2)
2- 2 = 2- vi
V20 + 14 V2 = V( + V2)
2 3 = 2 + vi
'V(a + 3)va - 3a -1 = 'V(va _1)3 = va -1,
-----;-a----==-_l---;- _ 1 = Va + 1 _ 1= Va-I .
2(va-1) 2 2

Tli do, ta co

b) Tli ghi thi€t, ta suy ra 2 (x + y) - 1 = 3 x y. Do do

x 2 - xy + y2 = (x + y)2 - 2(x + y) + 1 = (x +y - 1)2.

Suy ra

p =x +y + J (x + y - 1)2 = x + y + Ix + y - 11·

Mi;it khac, tu gia thi€t da cho, ta cling co 0 < l':X < 1 va 0 < l~Y < 1. Suy ra
o < x, y < ~ va x + y < 1. Do do
P = x + y + (1 - x - y) = 1. o
Nh~n xet. D6i voi y b), ta cling co th€ sit d\mg phep d~t ffn ph\! S =x +y va
P = x y (S 2 ~ 4 P) d€ ti~n cho cac bi€n doi va ly lu~n.

Bai 2 (2.0 c1iim). MQt xe tai co chi8u rQng la 2.4 rn va chi8u cao la 2.5 m
mu6n di qua mQt cai cong hlnh parabol. Bi€t khoang cach giua hai chan
cong la 4 rn va khoang cach tu dinh cong (dinh parabol) toi rn6i chan cong la
2.../5 rn (bo qua dQ day cua cong).
a) Trong m~t ph~ng tQa dQ 0 x y , gQi parabol (P) : y = a x 2 voi a < 0
la hinh bigu di€n cong rna xe tai rnu6n di qua. Chung minh r~ng a = - 1 .

b) Hoi xe tai co th€ di qua cong du<;jc kh6ng? T~i sao?

Vo QU6c Ba Cffn - Nguy€n M~nh Linh 123


CHVdNO 3. LOI olAI cAc HE ToAN ON TAp

Uti giai. a) Gill sit tren m~t ph~ng tQa dQ, dQ dfli cua cac dOl;m th~ng dU<;jc tinh thea
ddn vi met. Do khoang cach giUa hai chan c6ng b~ng 4 m nen M A = N A = 2.
Tli gill thic3t, ta co 0 M = 0 N = 2.)5. Do do, thea dinh ly Pythagore, ta tinh
dU<;jc 0 A = V0 M 2 - M A 2 = 4. V~y M (2, - 4), N ( - 2, - 4) .
M~t khac, do M, N thuQc parabol nen - 4 = a . 2 2 , suy ra a = - 1 va parabol
(P) co phu'dng trlnh: y = -x 2 .
b) D€ dap ling dU<;jc chi~u cao, trUdc hc3t xe tai phai chQn phudng an di VaG chinh
giftac6ng. Trenparabol (P), xethai di€m H (%, -;~) va T (-%, -;~) d6i
xling nhau qua 0 y va H T = 2.4 m (ling vdi chi~u rQng cua xe tai).
y

-2 -1 0 0 1 2
x

____ ___
~ ~4_ _______ ___ _
~

A
-5

GQi {B} = HT n Oy, khi do ta co


64
AB = -25 > 2.5.

Do do, xe tlli co th€ di qua c6ng. o

Bfli 3 (1. 5 dim). Cho hai s6 nguyen a, b thoa man di~u ki~n

a
2
+ b2 + 1 = 2(ab + a + b).
Chling minh r~ng a va b la hai s6 chinh phu'dng lien tic3p.

Uti giai. Bai to{m nay co hai cach ti~p c~n nhu sau.
each 1. Khong mfit Hnh t6ng quat, ta co th€ gill sit a ~ b. Nc3u a = b thl gia thic3t,
ta co a = b = *'
mau thufin do a, b E Z. Do do, a > b.
124 Vo Qu6c Ba Cgn - Nguy€n M~nh Linh
1. cAc HE THI TUYEN SINH CHfNH THUC

Gia thi6t da cho co th€ dUQc vi6t 1~i dudi d~ng

(a - b)2 +1= 2(b - a) + 4a,


hay
4a = (a-b+ 1)2.
Suy ra a - b + 1 chia h6t cho 2 va
(1)

Tl10ng tv, ta cling co a - b - 1 chia h6t cho 2 va

(2)

Do a-~+l > a-~-l ;? 0 va a-~+l - a-~-l = 1 nen k6t hQp vdi 0), (2), ta suy fa
a, b 1a hai s6 chfnh phudng lien ti6p. •
each 2. N6u b = 0 thi a = 1 va ro rang a, b 1a hai s6 chfnh phudng lien ti6p. Xet
tntdng hQp b =I- 0, gia thi6t da cho co th€ dUQc vi6t 1~i dudi d~ng

a 2 - 2(b + l)a + b 2 - 2b + 1 = o.
Xem day 1a mQt phudng trlnh b~c hai gn a. Phudng trlnh nay co nghi~m nguyen
nen bi~t thac jJ.' cua no phai 1a mQt s6 chfnh phudng. Ma

jJ.' = (b + 1)2 - (b 2 - 2b + 1) = 4b
nen ta co b Ia s6 chfnh phudng. f)~t b = x 2 vdi x E N* (do b =I- 0) thi ta cong thac
nghi~m cua phudng trlnh b~c hai, ta co

a = (b + 1) + y't;i = x 2 + 1 + 2x = (x + 1)2
ho~c
a = (b + 1) - y't;i = x2 + 1- 2x = (x - 1)2.
Trong eft hai trl1dng hQP, ta d€u co a, b 1a hai s6 chfnh phudng lien ti6p. 0
Nh~n xet. Ngoai hai cach giai tren, ta cling co th€ ti6p c~n bai toan b~ng each
chang minh a, b :;? 0 rbi vi6t gia thi6t dudi d~ng

4ab = (a +b-1)2.
ta
Bl1dc ti6p theo, ta co th€ chang minh (a, b) = 1 d€ do suy ra a, b 1a hai s6
chfnh phudng. d phfin sau cling cua cach giai, ta chi dn d~t a = x 2 , b = y2 vdi
x, Y E· N rbi thay vao gia thi6t, rut gQn d€ thfty x, y 1a hai s6 tv nhien lien ti6p. Til
do suy fa di€u phcli chang minh. .

Vo Qu6c Ba Cgn - Nguyen M~nh Linh 125


CHVdNG 3. LOI GIAI cAc HE TOAN ON TAp

Bai 4 (3.0 die'm). Cho tam giae nhQn ABC (AB < AC), M 1[1 trung diem
ella e:;tnh B C, 0 Ia tam ella dttdng tron ngo:;ti ti€p tam giae. Cae dttdng eao
AD, BE, CF ella tam giae ABC d6ng quy t:;ti H. Cae ti€p tuy€n voi (0) t:;ti
B va C dt nhau t:;ti S. GQi X, Y Ifin Itt<;1t Ia giao diem eua dttdng th~ng EF voi
cae dttdng th~ng B S, A O. Chung minh r~ng

a) MX -L BF.
b) Rai tam giae S M X va D H F d6ng d:;tng.
c) EF _ BC
FY - CD'

LUi giai. a) Vi BE -L CA va CF -L AB nen BCEF Ia ttl' giae nQi ti€p, suy ra

LXFB = LACB = LXBF.

Do do, tam giae X F Bean t:;ti X. Ma tam giae M F Bean t:;ti M nen M X ..L BF.
A ......······················ .....•....
/ ,

X' )
..
C

S
b) D@ th&y M X -L AB (theo a» va H F -L AB nen M X II H F. Ta cling co
M S ..L B C va H D ..L B C nen M S II H D. M~t khae, do C A F D la tu giae
126 Vo Qu6e Ba Ctln - Nguy@n M:;tnh Linh
1. cAc HE THI TUYEN SINH CHINH THUC

n9i tiep va SB ti€p xuc voi (0) t~i B nen LSBD = LBAC = LBDF, suy
ra S x II F D. Do do f:, M X S '" f:, H F D (co cac c~nh tu'dng ling song song).
c) D~ dang chling minh du'<;jc f:, A E F '" f:, ABC. Ta co
0

LOAE = 180 - LAOC = 90 0


_ LABC = 90°- LAEF,
2
suy ra 0 A .-L E F. TIt day, ta cling chling minh du'<;jc f:, A F Y '" f:, A CD, suy ra
FY AF EF
CD - AC - BC'
Ttl' day, taco
EF BC
o
FY CD

Bili 5 (1.0 c1iim). Trongm~tph~ngt9ad9 Oxy, chotamgiac ABC co cae


dinh Ul cac di~m nguyen (m6i di~m du'<;jc g9i Ia di~m nguyen neu hoanh d9 va
tung d9 cua cac di~m dola s6 nguyen). Chung minh r~ng hai lfin di~n tich cua
tam giac ABC la m9t s6 nguyen.

Liti gicii. Ta ke m~t ph~ng 0 x y thanh bfmg caro boi cac du'ong th~ng song song
va vuong goc voi trl;lc hoanh (g9i t£t Ia du'ong ngang va du'ong d9C) d€ giao cua cac
dl1dng nay t~o nen tftt cii cac di€m nguyen cua m~t ph~ng.
• N€u trong ba dinh cua tam giac ABC, co hai dinh, ch~ng h~n la A va B ,
cling n~m tren m9t du'ong ngang ho~c m9t du'ong d9C, thl r5 rang d9 dai do~n
th~ng A B va chi~u cao ke tu C xu6ng A B cua tam giac ABC la nhung s6
nguyen. Do do, hai lfin di~n tfch tam giac ABC la m9t s6 nguyen.

• N€u cac digm A, B, C n~m tren cac du'ong ngang va cac du'ong d9C phan
bi~t thi khong mftt tinh t6ng quat, ghi sii' du'ong ngang chlia B n~m giua cae
du'ong ngang chlia A va C. Co hai kha nang xay ra:
M A N

p C

Vo Qu6c Ba Gin - Nguy~n M~nh Linh 127


CHVdNO 3. LOI olAI cAc HE ToAN ON TAP

o MQt la, dudng dQc chua B khong bi kvp giua hai dudng dQc chua A va
chua C. Khi do, ta co

la nhung 86 nguyen.
A M

P t--~=

N Q C

o Hai la, dudng dQc chua B n~m giua hai dudng dQc chua A va chua C.
Khi do, ta co

la mQt 86 nguyen. o

1.7. O€ thi tuy@n sinh THPT chuyen KHTN, 2015 (vong 1)

Bai 1 (3.0 c!iim).

a) Gift 8U a, b la hai 86 tht;ic phan bi~t thoa man a 2 + 3 a = b 2 + 3 b = 2.


Chung minh r~ng a +b = -3 va a 3
+b 3
= -45.

b) Gifti h~ phuong trinh:

2X + 3y = 5xy,
! 4x 2 + y2 = 5xy2.

Liti giai. a) Ttl gift thi~t, ta co

hay
(a - b)(a + b + 3) -:- O.
128 Vo Qu6c Ba C~n - Nguy~n M~nh Linh
1. cAc HE THI TUYEN SINH CHiNH Ti-uJ'c
Ma a f=. b nen ta co a +b = - 3. Tli do suy fa

a 2 +b 2 =4-3(a+b)=13
va

Tli day, ta tinh du'<;Je

a 3 +b 3 = (a+b)(a 2 +b 2 -ab) = -45.


b) Tli h~ phu'dng trlnh da eho, ta co
4x 2 + y2 = y(2x + 3y),
hay
(x - y)(2x + y) = O.
SUYfa y = x ho~e y = -2x.
• Vdi Y = x, thay vao phu'dng trlnh thd nhfit eua h~, ta du'<;Je 5 x = 5 x 2. Suy
ra x = 0 (tu'dng dng, y = 0) ho~e x = 1 (tu'dng dng, y = 1).

• Vdi Y = - 2 x , thay vao phu'dng trlnh thd nhfit, ta du'<;Je - 4 x = - lOx 2 .


Suy fax = 0 (tu'dng dng, y = 0) ho~e x = ~ (tu'dng dng, y = -~).

V~y h~ da cho co cae nghi~m (x, y) la (0, 0), (1, 1) va (~, - ~) . D

Bili 2 (3.0 c1iim).

a) Tim tM cit cae s6 nguyen x, y kh6ng nho hdn 2 sao eho x y - 1 chia
h8t eho (x - 1)( Y - 1).

b) Vdi x, y la cae s6 thve thay d6i thoa man di6u ki~n x 2 y 2 + 2 y +1 = 0,


tim ghi tri !dn nhfit va nho nhfit eua bi€u thde

p = xy
3y + 1

La. giai. a) D~t a = x-I va b = y - 1. Theo yeu du eua bai toan, ta dn tim
cae s6 nguyen dU'dng a, b sao eho (a + 1) (b + 1) - 1 = a b + a + b ehia h8t
cho a b, hay a + b ehia h8t cho a b. Suy fa

a+b;::ab . .
Kh6ng mAt tinh t6ng quat, gia sti' a ;:: b. Khi do, ta co 2 a ~ a +b ;:: a b, suy
ra. b :::; 2. Ma b E Z + nen b = 1 ho~e b = 2.

Vo Qu6e Ba C~n - Nguy€n Ml;lnh Linh 129


CHVdNG 3. U)I GIAI cAc DE ToAN ON TAp

• Vdi b = 1, ta co a ?; b = 1 va a + 1 chia h~t cho a nen a = 1. MQt


each tUdng ung, ta co x = y = 2.

• Vdi b = 2, ta co a ?; 2 va a + 2 chia h~t cho 2 a nen a = 2. MQt each


tlldng ling, ta co x = y = 3.
V~y cae d,ip s6 (x, y) thoa man yeu du d~ bai la (2, 2) va (3, 3).
b) Ta co bi~n d6i sau
p = 2xy 2xy 2xy 2xy
6y + 2 3(2y + 1) - 1 -3X 2 y 2 - 1
Su d\lng bfrt d£ng thuc quen thuQc - ( a 2 + b 2) ~ 2 a b ~ a 2 + b 2 , ta co
_(3x 2 y2 + 1) ~ 2xy.J3" ~ 3x 2 y2 + 1.
Dodo
.J3 .J3
--:< P:<-
3" "3'
Bfrt d£ng thlic ben trai xay ra d~u b~ng khi va chi khi x y = - ~3 , k~th<Jp vdi gia
thi~t, ta Hnh dU<Jc y = - ~ va x = {!. Bfrt d&ng thuc ben phai xay ra dfru b~ng
khi va chi khi x y = f, k~t h<Jp vdi gia thi~t, ta co y = - ~ va x = _ ~3 .
V~y ta co max P = f va min P = - f. D
Nh~n xet. Yb) con co thg giai b~ng cach su dt,mg bfrt d&ng thuc Cauchy-Schwarz
nhu sau: Tu gia thi~t, ta co y =1= 0 va x 2 + ~ + -\ = 0, hay

C+ r
y y

2
x + 1 = 1.

D~t z = f- + 1, ta co x 2 + Z2 = 1. Bigu thuc P co thg dll<JC vi~t I~i thanh


x x
P = -----,-
3+ ly 2 + z'
hay
2P=x-Pz.
D~n day, su d\lng bfrt d&ng thuc Cauchy-Schwarz, ta co
4p2 = (x - PZ)2 ~ (1 + p 2 )(X 2 + Z2) = 1 + p2,
suy ra

--:< P:<-
.J3 .J3
3" "3'
Ph§n con I~i, ta chi dn xet dfru b~ng nita la xong.
130 Vo Qu6c Ba Cffn - Nguy~n M~nh Linh
1. cAc HE THI TUYEN SINH CHfNH THUC

Ba i 3 (3.0 dim). Cho tam ghlc nhQn ABC kh6ng can co tam dudng tron n(>i
ti€p Ia di€m I . DUdng th~ng A I dt B C t~i D. GQi E, F Ifin IUQt Ia cac di€m
d6i xung cua D qua I C; lB.

a) Chung minh r~ng E F song song v6i B C.

b) GQi M, N, J Ifin IUQt Ia trung di€m cua cac do~n th~ng


DE, D F, E F. Dudng tron ngo~i ti€p tam giac A EM dt dudng
tron ngo~i ti€p tam giac A F N t~i P khac A. Chung minh r~ng b6n
di€m M, P, N, J cung thu(>c m(>t dudng tron.

c) Chung minh r~ng ba di€m A, J, P th~ng hang.

LUi giai. a)D~th~y E E CA va F E AB. Dodo

BF BD CD CE
BA - BA - CA - CA .

Tit' day, ta suy ra E F II BC.

.......................:

..
...... ,"'"

B D C

Vo QU6c Ba Cfin - Nguy~n M~nh Linh 131


CHudNG 3. L<JI GIAI cAc HE TOAN ON TAp
b) Ta co
LMPN = LMPA + LNPA = LMEC + LNFB
= LMDC + LNDB = 180 - LMDN
0

0
= 180 -LMJN.
Do do, td giac M P N J nQi ti~p.
c) Do td giac M P N J nQi ti~p nen
LMPJ = LMNJ = LMEJ
= LEDC = LDEC
= LMPA.

Suy ra, ba digm A, J, P th~ng himg. o

Bai 4 (1.0 cfiim).

a) Cho bang 0 vuong 20 15 x 20 1 5. Ky hi~u 0 (i, j) Ut 0 d himg thd i ,


cQt thd j. Ta vi~t cac s6 nguyen dUcJng tit 1 d~n 20 15 viw cac 6 cua
bang theo quy t~c sau:

i) S6 1 dUQc vi~t vao 0 (1, 1).


ii) N~u s6 k dUQc vi~t vao 0 (i, j) v{ji i > 1, thl s6 k + 1 duqc
vi~t vao 0 (i - 1, j + 1).
iii) N~u s6 k duqc vi~t vao 0 (1, j) thl s6 k + 1 dUQc vi~t vao 0
(j+l,I).

1 3 6 10 ...
2 5 9 ...
4 8 ...
7 ...
...

Khi do, s6 2015 dUQc vi~t vao 0 (m, n). Hay xac dinh m va n.

sa
b) Gia a, b, e la cac s6 thl!C dUcJng thoa man a b + b e + e a + abe ~ 4.
Chdng minh r~ng

a2 + b2 + e2 + a + b + e ~ 2 (ab + be + ea) .
132 Vo Qu6c Ba C~n - Nguy~n Mc;mh Linh
1. cAc HE THI TUYEN SINH CHINH THUC

La. giai. a) Theo quy t£c di~n s6 d d~ bai, ta thfry s6 d hang 1, C9t k b~ng
1 + 2 + ... + k = k (\+ 1) • Ta co 6 3~64 = 2016 nen s6 2016 d hang 1, c9t
63. Tu do suy ra s6 20 1 5 d hang 2, c9t 62.
b) Gill thi~t d1l cho co th€ dll<Jc vi~t l<;li dlloi d<;lng

12 + (ab + be + ea) + 4(a + b + c)


~ 8 + 4(a + b + c) + 2(ab + be + ea) + abc,

hay

(2+a)(2+b) + (2+b)(2+e) + (2+e)(2+a) ~ (2+a)(2+b)(2+e).

M9t cach tlldng dlldng, ta co

111
--+
2+a2+b
+ 2+e ~l. (1)

D~n day, s11 dt;mg bfrt d~ng thuc Cauchy-Schwarz, ta co

1 a+ b2 + e 2 a + b2 + e 2
-------~-~~ ~ .
2+a (a + 1 + 1)(a + b 2 + e 2 ) (a + b + e)2
Tl1dng hi, ta cling co

1 b + e2 + a2 1 e + a2 + b2
--~------ --~-----
2 + b " (a + b + e)2' 2 +e (a + b + e)2 .

CQng cac bfrt d~ng thuc tren l<;li, ta thu dll<JC

1 1 1 2(a 2 + b2 + e2 ) + a + b + e
1~
2+a
+ 2+b
+---~
2+e (a+b+e)2
.
Suyra

hay
~ 2(ab
2
a + b2 + e2 + a + b + e + be + ea).
Dfru d~ng thuc xay ra khi va chi khi a = b = e = 1. o
Nh~n xet. Sau khi vi~t l<;li gill thi~t dlloi d<;lng (1), ta con co th€ ti~p c~n bai toan
b~ng cach bi~n d6i gill thi~t thanh

(~- 2 ~ a) + (~- 2 ~ b) + (~- 2 ~ c) ~ ~,


Vo Qu6c Ba Cffn - NguySn M<;lnh Linh 133
CHU'ONG 3. UJI GIAI cAc HE ToAN ON TAp

ha
abe
--+
a+2 b+2
+ e+2 :::;;1. (2)

Den day, sit dtJng bftt d~ng thuc Cauchy-Schwarz d<;tng cQng rnftu, ta co
a2 b2 e2
VT (2) = a 2 + 2a + b2 + 2b + e2 + 2e
(a + b + e)2
~ .
a2 + 2
b + e 2 + 2(a + b + e)

K€t h<;Sp vdi gia thi€t (2), ta dl1<;Sc

a2 + b2 + e2 + 2(a + b + e) ~ (a + b + e)2,
hay
a +b +e ~ ab + be + ea.
M~t khac, sa dtJng bftt d~ng thuc AM-GM, ta cling co
2 2 2 a 2 + b2 b2 + e2 e2 + a2
a +b +e = 2 + 2 + 2 ~ab+be+ea.

K€t h<;sp hai bftt d~ng thuc tn3n l<;ti, ta thu dl1<;Sc di6u phai chung minh.
Ngoai hai cach tren, ta cling co rnQt cach khac niia nhl1 sau: Trudc h€t, ta se chung
minh bftt d~ng thuc sau vdi rnQi x, y, z > 0,

x2+y2+z2+2xyz+l~2(xy+yz+zx). (3)

Trong ba s6 x, y, z Iuon co hai s6 cling khong nhi) hon 1, ho~c cling khong
IOn hon 1. Khong rnftt tinh t6ng quat, gia sa
hai s6 do la x va y. Khi do, ta co
2z(x - 1)(y - 1) ~ 0, suyra

2xyz ~ 2xz + 2yz - 2z.

Do do, ta chi cful chung rninh

x2 + y2 + z2 + 2xz + 2yz - 2z +1~ 2(xy + yz + zx),


hay
(x - y)2 + (z - 1)2 ~ O.
Bftt d~ng thuc cu6i hien nhien dung nen (3) dl1<;Sc chung minh.

134 Vo Qu6c Ba C§:n - Nguyen M<;tnh Linh


1. cAc HE THI TUYEN SINH CHINH THUC

Trd l~i bai toan, xet cac trridng hQp sau:

• TniO'ng hgp 1: ab + be + ea ;? 3. Tu gilt thi~t, ta co


abc ~ 4- (ab + be + ea) ~ 1.

Ngoai ra, thea b~t d~ng thuc quen thuQc (a + b + e)2 ;? 3(ab + be + ea)
(co thg chung minh b~ng bi~n d6i tttdng dttdng), ta cling co

a +b +e ;? ./3(ab + be + ea) ;? 3 ;? 1 + 2abe.


Do do, thea b~t d~ng thuc (3), ta co

a
2
+ b 2 + e2 + a + b + e ;? a 2 + b 2 + e2 + 2abe + 1 ;? 2(ab + be + ca).
• TniO'ng h«;fp 2: ab + be + ea < 3. Khi do, ta co

a + b + e ;? ./3(ab + be + ea) > ab + be + ea


va
a 2 + b 2 + e 2 ;? ab + be + ea.
CQng l~i, ta thu dttQc b~t d~ng thuc dn chung minh.

1.8. D~ thi tuy~n sinh THPT chuyen KHTN, 2015 (vong 2)

Bfli 1 (3.0 c1iim).

a) Cho a, b, cIa cac s6 thl.,iC thoa man

(3a + 3b + 3e)3 = 24 + (3a +b- e)3 + (3b + e - a)3


+ (3e + a - b)3.

Chung minh r~ng

(a + 2b)(b + 2e)(e + 2a) = 1.

b) Gilti h~ phttdng trlnh:

+ 2y + xy = 5,
j2x
27 (x + y) + y3 + 7 = 26x 3 + 27x2 + 9x.

Vo QU6c Ba Cffn - Nguy~n M~nh Linh 135


CHU'dNG 3. UJI GIAI cAc DE ToAN ON TAp

La. giai. a) D~t x = 3a + b - c, y = 3b +c - a va z = 3c + a-b. Khi do,


ghl thi~t da cho co thg dUQc vi~t l?i thanh

(x + y + Z)3 = x 3 + y3 + Z3 + 24.
M~t khac, ta l?i co (x + y + Z)3 = x 3 + y3 + Z3 + 3(x + y)(y + z)(z + x) nen
3(x + y)(y + z)(z + x) = 24.

Ttl do suy fa
24(a + 2b)(b + 2c)(c + 2a) = 24,
hay
(a +2b)(b + 2c)(c + 2a) = l.
b) H~ phudng trinh da cho co thg dl1Qc vi~t l?i duoi d?ng

ex + 2)(y + 2) = 9,
l
(1)
x 3 + y3 + 8 + 27(x + y) = (3x + 1)3. (2)

Taco

x 3 + y3 + 8 + 27(x + y) = x 3 + y3 + 8 + 3(x + y)(x + 2)(y + 2)


= (x + y + 2)3
nen phuong trinh (2) co thg dl1Qc vi~t l?i duoi d?ng

suy fa y = 2x - l. Thay vao phl1dng trinh (1), ta dl1Qc

(x + 2)(2x + 1) = 9,

hay
(x - 1)(2x + 7) = O.
Suy fax = 1 (tl1dng ang, y = 1) ho~c x = -~ (tl1ong ang, y = -8).
V~y h~ da cho co cac nghi~m (x, y) la (1, 1) va (-~, -8). D

136 Vo Qu6c Ba Cffn - Nguy~n M?nh Linh


1. cAc HE THI TUYEN SINH CHfNH THUC

Bai 2 (3.0 aim).


+ 5 va n + 30 d€u la s6 chinh phuong
a) TIm tfit cii cac s6 tv nhien n d€ n
(s6 chinh phuong la s6 b~ng blnh phuong cua mQt s6 nguyen).

b) TIm tfit ca cac s6 nguyen x, y thoa man d~ng thuc

1+ Jx + y + 3 = JX +..;y. (1)

c) Cho x, y, z Ia cac s6 thvc ldn hon 2. TIm gia tri nho nhfit cua bi€u thuc
x y z
p = Jy +z - 4
+
,Jz +x - 4
+
,Jx +y - 4
.

La. giai. a) D~t n +5 = a 2 va n + 30 = b 2 vdi a, b E Z+, a < b. Khi do, ta co

b2 - a 2 = 25,

hay
(b - a)(b + a) = 25.
Do b - a, b +a E Z va 0 < b - a < b + a nen chi co mQt tru'ong hQp sau xay ra:

b-a = 1,
!b + a = 25.

Giiii h~ tren, ta dUQc a = 12 va b = 13. Ttl do suy ra n = 139 (thoa man).


V~y chi co mQt ghi tri n thoa man yeu du la n = 139.
b) Trudc h€t, ta se chung minh cac be> d€ sau:
Ba d~ 1. Cho s6 tif nhien x thoa miin J x la s6 hilu ti. Khi do, x ta s6 chinh phuong.
ChU'ng minh. D~t Jx = f vdi p, q EN, q =I- 0, (p, q) = 1. Khi do, ta co

p2
X=-.
q2

Suy ra p 2 chia h€t cho q 2 . Ma (p, q) = 1 nen ta co q = 1, suy ra

B6 d€ dUQc chung minh.



Vo QU6c Ba Gin - Nguy~n M~nh Linh 137
CHudNG 3. LOI GIAI cAc DE ToAN ON TAp

Ba d@ 2. Cho hai s6 tT! nhien phdn bi¢t x, y thoa miin y' x - y' y ia m9t s6 hilu
tl. Khi do, cd hai s6 x, y (Mu ia s6 chfnh phuong.

ChU'ng minh. Do x f. y nen ta co


- _ x-y
y'x + y'y = y' y"
x- Y
Tit' do, k@t h<;jp vdi gilt thi@t, ta suy fa -Vi + y'Y E Q. Do do

/-
y X--
(y'x + y'y) + (y'x - y'y) tn\
E'I,£ .
2

Theo ba d@ 1, ta co x Ut s6 chfnh phudng. Chung minh tudng tt;l, ta cling co y la


s6 chinh phu'dng. B5 d~ du<;jc chang minh. •

Ba d@ 3. Cho hai s6 tT! nhien x, y thoa miin y' x + y' y ia m9t s6 hiiu d. Khi
do, cd hai s6 x, y diu ia s6 chfnh phuong.

LO'i gifti. TUdng tt;l nhu' b8 d@ 2.

Tfd l<;ti bai toan, blnh phu'dng hai v@ phudng trinh da cho va nit g<;m, ta dUQc

y'xy - y' x + y + 3 = 2. (2)

Do do, thea b8 d@ 2, ta co x y va x + y + 3 la s6 chinh phu'dng. M~t khac, d)ng


hai phudng trlnh (1) va (2) l<;ti thea v@, ta cling co

y'xy - 1 = y'x + y'y. (3)

Suy fa .Jx + y'Y E Q. Theo b8 d@ 3, ta co x, y la cac s6 chinh phu'dng.


D~t y' x = a va y' y = b vdi a, b E Z + . Phu'dng trinh (3) co thS vi@t l<;ti thanh

ab-l=a+b,
hay
(a-l)(b-l) =2.
Tit' day, vdi chu y a, b E Z + , ta tinh du'<;jc a = 2, b = 3 ho~c a = 3, b = 2.
M<)t cach tu'dng ung, ta co x = 4, y = 9 ho~c x = 9, y = 4 (thea man).
V~y co hai c~p s6 (x, y) thea man yeu c~u bai toan la (4, 9) va (9, 4).

c) Sit d\lng b<1t dang thuc AM -GM, ta co

/ y'4(y+z-4) 4+(y+z-4) y+z


vy+z-4= ~------
2 4 4
138 Vo Qu6c Ba Cffn - Nguy~n M<;tnh Linh
1. cAc HE THI TUYEN SINH CHINH THUC

Tli d6 suy ra
x 4x
~======7 ~ .
,Jy+z-4 y+z
Chung minh tudng hj, ta cling c6
y 4y
-r======~ ~ ,
,Jz+x-4 z+x
z 4z
-r=;====:::;:~ .
,Jx+y-4 x+y
CQng cac bAt d~ng thuc tren ll;li va sit dl,mg bAt d~ng thuc Cauchy-Schwarz dl;lng
cQng mftu, ta dU<;Jc

P~4(
y+z
x
z+x
+
x+y
Y + z )

X2 y2 Z2)
=4 (
xy + xz
+ yz + yx
+ - --
zx + zy
4(X+y+Z)2
~ ---'-------:-
2(xy + yz + zx)
2(x + y +Z)2
xy + yz + zx
M~t khac, ta ll;li c6 (x +
y + z) 2 ~ 3 (x Y + y z + z x) (c6 thg chung minh
b~ng bi6n d6i tUdng dudng) nen k~t h<;Jp v{ji danh gia tren, ta suy ra

2(x + y + Z)2 6
P ~ ~ .
xy + yz + zx
Dfiu d~ng thuc xay ra khi va chi khi x = Y = z = 4. V~y min P = 6. 0

Bfti 3 (3.0 diim). Chotamgiacnh9n ABC khongdin v{ji AB < AC. G9i
M la trung digm clla dOl;ln th~ng B C. G9i H la hlnh chi~u vuong g6c clla B tren
do(;l.ll th~ng AM. Tren tia d6i clla tia A M lAy digm N sao cho AN = 2 M H .

a) Chung minh r~ng B N = AC.


b) G9i Q Ia digm d6i xung v{ji A qua N. Duong th~ng A C dt B Q tl;li
D. Chung minh r~ng b6n digm B, D, N, C cung thuQc mQt duong
tron, g9i duong tron nay la ( 0 ) .

c) Duong tron ngol;li ti~p tam giac A Q D c£t (0) tl;li G khac D. Chung
minh r~ng N G song song v{ji B C.

Vo QU6c Ba Cffn - NguySn Ml;lnh Linh 139


cRlJdNG 3. UJI GIAI cAc DE ToAN ON TAP

La. giai. a) GQi K la hlnh chi~u cua C tfen AM. Ta d~ th~y B H = C K va


HM = M K. Ttl do suy fa AN = 2 H M = H K va H N = A K. Ttl day, ta
chung minh du'Qc !:::. BH N = !:::. C K A (c-g-c), suy fa B N = AC.

B .......... , .. " .
/C
'..... K .." .. " .'
.............................................................. ,

b) Do !:::. B H N = !:::. C K A nen ta co L B N H = L C A K, suy fa


LBNQ = LCAN.
co AN = QN va BN = CA nen !:::.BNQ = !:::.CAN (c-g-c), suy fa
L~i
LQBN = LNCA hay LDBN = LNCD. V~ytugiac BDNC nQi ti~p.
c) Ta co LGQA = LGDA = LGBC va LGAQ = LGDQ = LGCB
nen !:::. G Q A '" !:::. G B C. Rai tam giac nay co cac trung tuy~n tu'ong ung la G N
va GM nen !:::.GQN rv !:::.GBM va !:::.GNA '" !:::.GMC.
Ttl do ta co 6. G N M '" !:::. G A C, suy fa
LGNQ = 180 0 - LGNA = 180 0 - LGAC = LGAD.
K~t h<Jp voi L G QN = L G D A, ta suy fa !:::. G QN rv !:::. G D A. Do do
DA DA DG
NA QN QG'
M~t khac, ta cling co L DAN = L D G Q nen !:::. DNA !:::. D Q G. Suy fa rv

LGNC = LGDC = LNDQ = LNCB.


Ttl day, ta suy fa N G II B C. D

140 . Vo Qu6c Ba Cgn - Nguy~n M~nh Linh


1. cAc HE THI TUYEN SINH CHfNH THUC

Bfti 4 (1.0 aim). Ky hi~u S Ia t~p hQp g6m 2015 diSm phan bi~t tren m~t
ph~ng. Giii sa
tftt cii cac diSm clla S khong cung n~m tren mQt dlidng th~ng.
Chung minh r~ng co it nh~t 20 1 5 dlidng th~ng phan bi~t rna m6i dlidng th~ng
di qua it nh~t hai diSm clla S .

La. giai. Ta se chung minh bai toan dung v(ji t~p hQp S co n diSm (n ~ 3).
Tru(jc hSt, ta chung minh co t6n tl;li dlidng th~ng di qua dung hai diSm clla S. Giii
sii' L la t~p hQp cac dlidng th~ng di qua it nh~t hai diSm clla S. VI S, L la cac t~p
hQp hifu hl;ln, suy ra ta co diSm A E S va dlidng th~ng.e E L sao cho khoiing cach
tit A dSn .e 1ft nho nh~t (A ¢ .e). GQi B la chan dlidng cao hl;l tu A dSn dlidng
th~ng .e, diSm B chia dlidng th~ng .e thanh hai naa dlidng th~ng va ta chung minh
sa
d m6i phia clla B chi t6n tl;li nhi€u nh~t mQt diSm clla S . Giii phiin chung co mQt
nii'a dlidng th~ng co chua hai diSm clla S , Ia C I, C 2 (C I t=
C 2) .
A

Taco 0 :::;; IBCII < IBC 2 1 va~ACIC21atuho~cvuong(khiCl =


B),suy
ra LAC I C 2 la IOn nh~t. Ta co IC I C 21 < IA C 21, khi do khoiing cach tit C I
dSn A C 2 nho hon khoiing cach IA B I (mau thu~n v(ji giii thiSt khoiing cach tit
A dSn.e la nho nh~t). Suy ra .e di qua nhi€u nhM hai diSm clla S, va do do .e di
qua dung hai diSm clla S.
Ta chung minh tiSp bai tmin b~ng quy nl;lp thea n ~ 3. Trlidng hQp n = 3 la hiSn
sa
nhien: Giii kSt lu~n dung v(ji n - 1, ta chung minh kSt lu~n clla bai toan dung
v(ji n . Ap d\lng kSt quii bai toan tren v(ji t~p hQp S , ta co t6n tl;li mQt dlidng thing
di qua dung hai diSm, gQi hai diSm do la A, B. .
Ta xet t~p hQP S' =S \ {A}, khi do co hai trlidng hQp xiiy ra:

• Trtiang hQ'p 1: Tat ed edc-dilm eua S' (S' g6m n - 1 dilm) dtng ndm
tren m(Jt duong thang .e. Khi do, ta nh~n dliQC t~p hQp g6m n dlidng thing
phan bi~t {AX: XES'} U {.e}.

V6Qu6c Ba Cfin - NguySn Ml;lnh Linh 141


CHVdNG 3. LOI GIAI cAc DE ToAN ON TAp

• Tnidng hQ'p 2: Tilt cd cdc dilm cila S' kh6ng thu¢c ciing m¢t duong
thang. Khi do, thea gia thi€t quy n~p co it nh~t n - 1 dlldng th~ng nbi cac
di~m clla S'. Vi dlldng th~ng e = A B ph an bi~t vdi n - 1 dlldng th~ng
tren (e n S' = {B}).
Voi n = 2015, ta dllQC di~u phai chdng minh. o
" ?
1.9. De thi tuyen sinh THPT chuyen, SO' GD-DT Ha NQi,
2015 (danh cho chuyen Toan)

B~1i 1 (2.0 diim).

a) Gicli phlldng trlnh:

x-,Jx-S-3,Jx+l =0.

b) Giai h~ phlldng trlnh:

(1)
(2)

La. giai. a) Di~u ki~n: x ~ S. Phlldng trlnh da cho tlldng dlldng voi

2x - 2,Jx - S - 6,Jx +2 = 0,

hay
(,JX-S-l)2 +(,JX-3)2=0.
Phudng trlnh nay tlldng dlldng dlldng voi

l ,Jx-S-l=o,
,Jx-3=0.

Gild h~ nay, ta dllQC x = 9. V~y phlldng trlnh da cho co nghi~m duy nh~t x = 9.
b) Tit (1) va (2), ta suy fa
x3 + 2 y 3 = 2(x 2 + y2)(X - 2y),
hay
(x - 2Y)(X2 + 2y2) = 0.

142 Vo Qu6c Ba Clln - Nguy@n M~nh Linh


1. cAc HE Tl:II TUYEN SINH CHINH THUC

Suy ra x = 2 y. Thay vao phuong trinh 0), ta du<;Jc 5 y 2 = 5. Ttl do, ta co y = 1


(tuong ling, x = 2) ho~c Y = -1 (tuong ling, x = - 2).
V~y h~ phuong trinh co hai nghi~m (x, y) 1a (2, 1) va ( - 2, - 1 ) . D

Bfti 2 (2.5 dim).

a) Cho s6 nguyen duong n thoa man n va lOla hai s6 nguyen t6 cling nhau.
Chang minh r~ng (n 4 - 1) chia h€t cho 40.

b) Tim tfrt ca cac s6 nguyen t6 p va cae s6 nguyen duong x, y thoa man

p - 1 = 2x (x + 2),
1p2 _ 1 = 2y (y + 2).

c) Tim tfrt cii cac s6 nguyen duong n sao cho t6n t~i cac s6 nguyen duong
x, y, z thoa man phuong trinh

Lui giai. a) Vi (n, 10) = 1 nen n khong chia h€t cho 2 va 5.


Do n khong chia h€t cho 5 nen n 2 chia 5 du 1 ho~c 4, suy ra n 4 chia 5 du 1 . Tit
do, ta co n 4 - 1 chia h€t cho 5. (1)
Do n khong chia h@tcho 2 nen n co d~ng n = 2k + 1 (k E N). Suy ra

+ 1)(n 2 + + 1)(n 2 + 1).


4
n - 1 = (n - 1)(n 1) = 4k(k

Vi k (k + 1) chia h@t cho 2 nen ttl tren, ta suy ra n 4 - 1 chia h€t cho 8. (2)
Tit (1), (2) va (5, 8) = 1,tasuyran 4 -1chiah@tcho40.
b) H~ phuong trinh da cho co th~ dUQc vi€t l~i dudi d~ng

l p+1=2(x+1)2,
p2+1=2(y+1)2.

DS thfry p Ie. Ttl (3) va (4), ta suy ra y > x (do p2 + 1 > p +


(3)
(4)
1). Ngoai ra, do

2p2 > p2 +1= 2(y + 1)2


nen ta cling co p > y + 1. Tom l~i, ta co
p - 1 > Y > x > O.

Vo QU6c Ba Cfin - Nguy~n M~nh Linh 143


CHVdNG 3. LOI GIAI cAc HE ToAN ON TAp

Bay gid, xet (4) - (3), ta dUQc

pep - 1) = 2(y - x)(y + x + 2). (5)

Do p la s6 nguyen t6 nen ti:t tren, ta suy ra p la uoc cua m9t trong cac s6 2, y - x
va y + x + 2. Tuy nhien, do p Ie va 0 < y - x < p nen hai s6 d~u khong th€
chia h€t cho p. Suy ra y + x + 2 chia h€t cho p.
M~t khac, ta Il;li co 0 < y + x + 2 < 2 P nen ti:t tren, ta suy ra y + x + 2 = p.
Thay vao (5), ta dUQc y - x = P~l . Do do, p = 4x + 3 va y = 3x + 1. Thay
k€t qua nay vao phuong trinh (3), ta dUQc

4(x + 1) = 2(x + 1)2,

suy ra x = 1 (do x E N*). 1\1 do, ta co p = 7 va y = 4 (thea man). V~y co duy


nh&t mQt b9 s6 (x, y, p) thea man yeu cftu la (1, 4, 7).
c) Khong m&t tinh t6ng quat,gia sit x ~ y ~ z. Tit phuong trinh
(6)

ta co 3x 3 ~ x 3 + y3 + Z3 = nx 2 y 2z 2. Suy ra

ny2z2
x ~-'---
3
M~t khac, ta ding co y3 + Z 3 chia h€t cho x 2 , do do ta co

hay
18~n2yz4. (7)
N€uz ~ 2thitaciingcoy ~ 2(doy ~ z).Dodon 2 yz 4 ~ yz4 ~ 32> 16,
mau thu§:n v{ji (7). V~y ta phai co z = 1.
Xet cae trl1dng hQp sau:

• TnfO'ng hgp 1: y = 1. Do y3 + Z3 chia h€t cho x 2 nen ta co 2 chia h€t cho


x 2 , suy ra x = 1. Thay vao (6), ta dUQc n = 3.

• TnfO'ng hgp 2: y = 2. Do y3 + Z3 chia h€t cho x 2 nen ta co 9 chia h€t cho


x 2 , suy fa i = 3 (do x ~ y = 2). Thay vao (6), ta dUQc n = 1.
• TnfO'ng hgp 3: y ~ 3. Ti:t (7), ta co 18 ~ n 2y ~ 3n 2, suy ra n 2 ~ 6. Ma n
Ia s6 nguyen duong nen n E {I, 2}.

144 Vo Qu6c Ba Cffn - Nguyen M<;lnh Linh


1. cAc HE THI TUYEN SINH CHfNH THUC

o Voi n = 2, tit (7), ta co 4y :::;:; 18. Ma y E N* va y ~ 3 nen y E {3, 4}.


Tuy nhien, khi thti'trl;tc ti€p voi n = 2 va y E {3, 4}, ta d~u khong tim
dUQc cac s6 nguyen duong x thoa man (6).
o Voi n = 1, phuong trinh (6) co thg dUQc vi€t h,ti thanh
(8)

Do x ~ y ~ 3 nen ta co

suy ra x > ;1 y2. M~t khac, do y3 + 1 chia h€t cho x 2 nen ta co


5 28 25
_y3 > _y3 >- y3 + 1 >- x2 > _y4
4 27?' ?' 121'
hay
121
y:::;:; 20'
Tit do, ta co y E {3, 4, 5, 6}. Thay l<;li vao (8), ta d~u khong tim dUQc
cac s6 nguyen duong x thoa man.

Tom l<;li, co hai gia tri n thoa man yeu d.u d~ bai Ia n = 1 va n = 3. o
Nh~n xet. Trong Idi gilti Y c) (j tren, ta ciing da tim dUQc t§.t ca cac bi? s6
nguyen duong (x, y, Z, n) thoa man phuong trinh (6) la (1,2, 3, 1), (1, 3,2, 1),
(2, 1, 3, 1), (2, 3, 1, 1), (3, 1, 2, 1), (3, 2, 1, 1) va (1, 1, 1, 3).

Bai 3 (1.5 c1ilm). Cho a, b, e > 0 thoa man (a + b)(b + e)(e + a) = 1.


Chung minh r~ng
3
ab + be + ea ~ 4'

LUi giai. Truoc h€t, ta se chang minh


8
(a + b)(b + e)(e + a) ~ 9(a + b + e)(ab + be + ea). (1)

Do (a + b + c) (a b + be + e a) = (a + b)( b + e)( e + a) + abc nen b§.t


d~ng thuc tren tl10ng duong voi
8
(a + b)(b + e)(e + a) ~ 9"[(a + b)(b + e)(e + a) + abc],
Vo Qu6c Ba Cffn - Nguy€n M<;lnh Linh 145
CHVdNG 3. LOI GIAI cAc HE ToAN ON TAP

hay
(a + b)(b + e)(e + a) ;:? 8abe.
D€n day, sit d\lng bfit d~ng thuc AM -GM, ta co

(a + b)(b + e)(e + a) ;:? 2J7ib· 2.../bC. 2,Jea = 8abe.

Bfit d~ng thuc (1) du<;jc chung minh. Dfiu d~ng thuc xiiy ra khi va chi khi a = b = e.
Ttl' (1) va gia thi€t b~ti toan, ta suy ra
9
(a + b + e)(ab + be + ea) ::::; 8'
hay
81
(a + b + e)2(ab + be + ea)2::::; 64.

M~t khac, sit d\lng bfit d~ng thuc quen thuQc (a + b + e) 2 ;:? 3 ( a b + be + ea )
(co th€ chung minh b~ng bi€n deli tu'dng dudng), ta dU<;jc
81
3(ab + be + ea)3 :::; (a + b + e)2(ab + be + ea)2 :::; 64'

suyra
3
ab + be + ea :::; 4.
Dfru d~ng thuc xiiy ra khi va chi khi a = b = e = ~. D

Bili 4 (3.0 c1iim). Cho tam giac ABC co ba goc nh9n, nQi ti€p dUClng tron
( 0). Cac du'Clng cao AM, B N, C P cua tam giac ABC cling di qua di€m
H. G<;>i Q la di€m bfrt ky tren cung nh6 Be (Q khac B, Q khac C). G<;>i
E, F thea thu tl,i Ia di~m d6i xung cua Q qua cac duClng th~ng A B va A C .

a) Chung minh r~ng M H . M A = M P . M N.


b) Chung minh r~ng ba di~m E, H, F th~ng hang.
c) G9i J la giao di~m cua QE va A B, I la giao di€m cua QF va A C .
TIm vi tri cua di~m Q tren cung nh6 B C d~ ~ ~ + ~~ nh6 nhftt.

LO'i gicii. a) Ta co LAM B = LAN B = 90° nen tu giac AN M B n<;'>i tiSp,


suy ra L BAM = L B N M va LAB N = LAM N . (1)
Tu'dng ttl, do LBP H = LBH M = 90° nen tu giac BM H P nQi ti€p, suy fa

LABN = LPMA. (2)


146 Vo Qu6c Ba C~n - NguySn M~nh Linh
1. cAc HE THI TUYEN SINH CHfNH THUC

Tu (1) va (2), ta suy fa LP M H = LAM N. L<;li c6 LBAM = LBN M (do


(1)) nen f::. A PM rv Tu
f::. N H M (g-g). d6, ta c6

PM AM
HM NM'
hay
PM ·MN = AM· HM.
A.........................................
..........
"'\

/" C
/
.........
.
'
........................... ... .................. .
Q
b) Do LABC = LAQC va LAQC = LAFC nen LABC = LAFC.
M~t khac, ta l<;li c6 LAB C + LA H C = LAB C + L M H P = 18 0 0 (do ta
giac B M H P nQi ti€p) nen LA F C + LA H C = 1 80 0 • Tu d6 suy fa ta giac
AHCF n9i ti€p. Do d6 LCH F = LCAF, rna LCAF = LCAQ nen

LCHF = LCAQ. (3)

Chang minh tu'dng t\!, ta cling c6

LBAQ = LBHE. (4)

Tu (3) va (4), vdi chu y tu giac A PH N n9i ti€p, ta suy fa


L.BHC + LBHE + LCHF = LCAQ + LBQA + LPHN
= LNAP + LPHN
0
= 180 •

Do d6, ba di€m E, H, F th&ng hang.

Vo QU6c Ba Cffn - Nguy~n M<;lnhLinh 147


CHUONG 3. LOI GIAI cAc HE ToAN ON TAp
c) Ke Q K -L B C (K E B C). Tren B ClAy D sao cho L D Q C = L B Q A .
Ta co L DC Q = L B A Q nen 1:,. A B Q ~ 1:,. CD Q (g-g). Hai tam giac nay co
cac dlldng cao tu'dng ung la QJ va QK nen
AB CD
(5)
QJ QK·
Do LDQC = LBQA va LDQC + LDQB LBQA + LCQA nen
L C Q A = L D Q B. TU day, chung minh tu'dng tl;l' nhu' (5), ta cung co
AC BD
(6)
QI QK·
Ttl (5) va (6), ta suy ra
AB AC CD + BD BC
QJ + QI = QK QK·

Nhu' th~, t6ng ~~ + ~~ nhi) nh~t khi va chi khi ~i- nhi) nh~t. Ma B C kh6ng d6i
nen di6u nay xay ra khi va chi khi Q K IOn nh~t, tuc Q Ia di6m chfnh giua cua cung
nhi) B C. V~y di6m Q dn tIm chinh la trung di6m cua cung nhc> B C. 0

Bai 5 (1.0 diim). Chung minh r~ng t6n t<;li cacsb nguyen a, b, c sao eho

LUi giai. BM toan nay co hai cach ti~p c~n nhu' sau:
Cach 1. Trude h~t, ta chung minh b6 d6 sau:
Ba d~ 1. Cho x, y, z la cac siJ nguyen thoa man
x +y-J2+z-J3 = O.
Khi d6 x = y = z = o.
Chung minh. Vdi y = 0, ta co
x+z-J3=0.
N~u z =/:. 0 thl -J3 = -~ E <Q, mau thu§:n. Do do z = 0, ttl do suy ra x = o.
Tlldng tl;l', vdi z = 0, ta cung co x = y = O.
Xet trlldng h<Jp y z =/:. o. Khi do, ttl gia thi~t, ta co
x 2 = (y-J2 + z,J3)2 = 2y2 + 3z 2 + 2yz-J"6,
2 2
suy ra v'6 = x -2y2-3z
2yz
E <Q, mau thu§:n. Tom l<;li, x = Y = z = O. •
148 Vo Qubc Ba Cftn - Nguy~n M<;lnh Linh
1. cAc HE THI TUYEN SINH CHINH THUC

Trd I~i bai tmin, xet cac s6 nguyen x, y, z voi x, y, Z E [0, 1 03]. Ta co

Ix + yV2 + z.J31 ~ Ixl + lyl.J2 + Izl.J3 < 3


5.10 .

. Bay gd, ta chia do~n [0, 5 . 1 03] thanh 5 . 1 0 6 khoang gia tri nhu sau:

~) , [~,
[ 0, 10 10
~)
10
, ... ,
3
[5.10 - ~,
10
3
5.10 ) .

Vi x, y, Z E [0, 1 03] nen co (1 03 + 1) 3 bl) s6 nguyen (x, y, z). Theo


nguyen ly Dirichlet, t6n t~i it nh&t hai bl) phan bi~t (x 1, Y 1, Z 1) va (x 2, Y 2, Z 2)
sao cho hai s6 U = Xl + Yl.J2 + ZI.J"3 va v = X2 + Y2.J2 + Z2.J"3 thul)c
cung ml)t khoang gia trio Tli do, ta co
1
IU - v I < 10 3 '
M~t khac, vi day la hai bl) phan bi~t nen theo b6 d@ 1, ta co U - v =I- O. Do do

0< l(x 1 - X2) + (Yl - Y2).J2 + (ZI - Z2).J31 < 1~3'


Kh~ng dinh bai toan dUQc chang minh. •
Cach 2. Truoc h€t, ta chung minh bi) d€ sau:
Ba d@ 2. Vdi m9i n nguyen duong, t6n tt;li cac s6 nguyen an, bn sao cho
( .J2 - 1) n = a n .J2 - b n .

Chang minh. Ta chung minh b~ng quy n~p theo n . D@ th&y kh£ng dinh dung voi
sa
n = 1. Gia kh£ng dinh dung d€n n = k, tuc Ia t6n t~i ak, bk E Z d~

( .J2 - 1) k = a k .J2 - bk.


Khi do, ta co

( .J2 - 1) k+l = ( .J2 _ 1) k ( .J2 - 1)

= (a k .J2 - bk) ( .J2 - 1)


= (2ak + bd - (ak + bk) .J2.
ChQn a k + 1 = - a k - b k. b k + 1 = - 2 a k - b k thi ta co a k + 1, b k + 1 E Z va
/- )k+l r
( v2-1 =ak+lv2-bk+l.

D~ do, kh~ng dinh cling dung voi n = k + 1. Theo nguyen Iy quy n~p, ta co
khang dinh dung voi mQi n nguyen dudng. •

Vo QU6c Ba C§:n - Nguy@n M~nh Linh 149


CHUONG 3. L(JI GIAI cAc HE ToAN ON TAp

Trd l~i bai toan, do 0 < .J2 - 1 < 1 nen t6n t~i s6 nguyen duong n de
1
o < ( .J"2 - 1) n < 10 00'

Theo bB d~ 2, t6n t~i cac s6 nguyen an, b n sao cho

Bay gid, chQn a = -b n , b = an va c = 0 thi taco

Cac s6 a, b, c dUQc chQn nhu tren thoa man yeu cau bai toano o
... ?

1.10. De thi tuyen sinh THPT chuyen, SCi GD-OT Him NQi,
2015 (d~mh cho chuyen Toan-Tin)

Bai 1 (2.0 c1iim).

a) Gild phuong trinh:

(2X2 - 6x + 5)(2x - 3)2 = 1.

b) Gild h~ phuong trinh:

iX23+XY +y2 = 1,
~ 2x = x - y.

Ufi giai. a) Phuong trinh da cho tuong duong vdi

(4x 2 - 12x + 10)(2x - 3)2 = 2,

hay
[(2x - 3)2 + 1] (2x - 3)2 = 2.
D~t t = (2x - 3)2 (t ~ 0) thi ta co

t(t'+1)=2,
hay
(t - l)(t + 2) = O.
150 Vo QU6c Ba Gin - NguySn M~nh Linh
1. cAc HE THI TUYEN SINH CHINH THUC

Do t ;;? 0 nen tli tren, ta suy ra t = 1, hay

(2x - 3)2 = 1.

Giai phuong trinh nay, ta dU<;lc x = 1 ho~c x = 2. V~y phuong trinh da cho co
hai nghi~m la x = 1 va x = 2.
b) Tit h~ da cho, ta suy ra

2x 3 = (x - Y)(X2 + xy + y2),

hay
2x 3 =X 3 _ y 3.
Tit day, ta co y = - x . Thay vao phuong trinh thu nhfit eua h~, ta dU<;lC x 2 = 1.
Suy ra x = 1 (tuong ung, y = -1) ho~e x = - 1 (tuong ung, y = 1).
V~y h~ da eho co hai nghi~m (x, y) la (1, - 1) va ( - 1, 1). D

Bai 2 (2.5 diim).

a) Tim tfit ca ca.c s6 tt;! nhien x, y thoa man

x 2 -2xy+3 y 2=X+Y·

b) Tim tfit ea cac s6 nguyen duong n sao cho s6 .j ~n;52 la s6 huu ti.
c) Cho a, b, c, d la cac s6 nguyen duong thoa man a b = cd. Chung
minh ding a + b + c + d khong la s6 nguyen t6.

La. gicii. a) Phuong trinh da cho co thg dU<;lC vi€t'll;li du6i dl;lng

x 2 - (2y + l)x + 3y2 - Y = o.


Xem day la phudng trinh b~c hai ftn x . Phuong trinh nay co nghi~m khi va chi khi
bi~t thue D. cua no co gia tri khong am. Ma

D. = (2y + 1)2 - 4(3y2 - y) = 1- 8y(y -1)

nen ta phai co
8y(y-l)::;;1.
.
N€u y ;;? 2 thi ta co 8 y (y - 1) ;;? 16 > 1, mau thu~n. Do do, ta phai c6 y ::::; 1.
Ma YEN nen Y E {O, I}.

Vo Qu6c Ba Cffn - Nguy~n Ml;lnh Linh 151


CHUONG 3. U)I GIAI cAc HE ToAN ON TAp

• Vdi y = 0, ta co x 2 - X = O. Do do X = 0 ho~c X = 1.

• Vdi y = 1, ta co X 2 - 3X +2= O. Do do X = 1 ho~c X = 2.

V~y cac c~p s6 (x, y) thoa man yeu cfiu Ut (0, 0), (1, 0), (1, 1) va (2, 1).

b) Ta co
j4n - 2 = J (4n - 2)(n + 5) .
Y n+5 n+5

Do J~n;s2 1a s6 huu ti nen J (4n - 2) (n + 5) 1a s6 huu ti. Theo b6 d~ 1 Wi


giai cau 2b) cua d~ tuy~n sinh THPT chuyen KHTN nam 2015 - vong 2 (b~n dQc
vui long xern l~i ph§.n tntdC), ta co (4n - 2) (n + 5) 1a s6 chinh phudng.
M~t khac, do n E N * nen d~ thfiy

(2n + 1)2 < (4n - 2)(n + 5) < (2n + 5)2.


a
K~t hQp vdi 19 1u~n tren va chu 9 r~ng (4 n - 2) (n + 5) chan, ta suy ra
(4n - 2)(n + 5) nh~n m9t trong cac gia tri (2n + 2)2 va (2n + 4)2.

• Vdi (4n - 2)(n + 5) = (2n + 2)2, ta co IOn = 14. Khong t6n t~i s6
nguyen dudng n thoa man.

• Vdi (4n - 2)(n + 5) = (2n + 4)2, taco 2n = 26. Suyran = 13.

V~y co duy nhfit m9t s6 nguyen dUdng n thoa man yeu cfiu 1a n = 13.
c) Gia sa a +b+e +d = p 1a m9t s6 nguyen t6. Khi do, tit gia thi~t, ta co

ep = ae + be + e 2 + cd = ae + be + e 2 + ab = (a + e)(b + c).

Suy fa p 1a uoc nguyen t6 cua m9t trong hai s6 a + e va b + e. Thy nhien, di~u
nay khong the xay fa, do 0 < a + e < p va 0 < b + e < p. M§u thu§:n nh~n
dUQc cho ta k~t qua cfin chang minh. 0

Nh~n xet. Ngoai cach giai neu tren, 9 b) con co th~ gi:ii b~ng cach khac nhu sau:
D~t J ~n;s2 = f vdi p, q E N * va (p, q) = 1. Khi do, ta co
4n - 2
n+5
hay
(1)
152 Vo Quac Ba Ciln - Nguy~n M~nh Linh
1. cAc HE THI TUYEN SINH CHINH THUC

Tli day, ta suy ra q 2 ( 4 n - 2) chia h~t cho p 2. Ma (p, q) = 1 nen ta co 4 n - 2


chia h~t cho p 2. D~t
4n - 2 = kp2 (2)
Vdi k E N*. Thay vao (1), ta ding co

n+5=kq2. (3)
Xet 4 x (3) - (2), ta du'<;lC

22 = k(4q2 - p2) = k(2q - p)(2q + p).


D6 Yr~ng 2 q - p, 2 q + P co cling tinh chan Ie nen tli phu'ong trinh tren, ta
suy ra hai s6 nay phai Ie (vI ngu'<;lC ll;li se dftn d~n 22 chia h~t cho 4). Do do, tich
(2 q - P ) (2 q + p) Ia u'dc du'dng cua 11 . Ma 2 q + P > 1 nen ta co 2 q + p = 11,
suy ra 2 q - P = 1 va k = 2. Tli day, ta tim du'<;lC p = 5, q = 3 va n = 1 3 .
Y c) cling co th€ giiii b~ng cach khac nhu' sau: D~t (a, c) = m (m E N *) .
Khi do, t6n tl;li cac s6 nguyen du'dng x, Y vdi (x, y) = 1 sao cho a = m x va
c = my. Thay vao gia thi~t, ta du'<;lC

bx = dy.
Suy ra b x chia h~t cho y. Ma (x, y) = 1 nen b chia h~t cho y. D~t b = n y
vdi n E N* thi ta cling co d = nx. Tli day, ta co

a +b+c +d = mx + ny + my + nx = (m + n)(x + y)
Ia h<;lp s6 do m + n > 1, x +Y > 1.

Bai 3 (1.5 c1ie"m). Cho x, y, z Ia cac s6 thvc du'dng, nho hdn 1 thoa man

xyz = (1 - x)(1 - y)(1 - z).

Chdng minh r~ng trong ba s6 x (1 - y), y (1 - z) va z (1 - x), co it nhftt


mQt s6 kh6ng nho hdn t.
La. giai. Bai toan nay co hai cach ti~p c?n nhu' sau:
each 1. Tit gia thi~t, ta co

(XYZ)2 = [x(1- x)][y(1- y)][z(1- z)].

Su d\lng bAt d~ng thdc AM-GM dl;lng ab ~ (a~b)2, ta co

1 1 1
x(l-x)~- Y(1-Y)~4' z(1-z)~4'
'" 4'
Vo Qu6c Ba C§:n - Nguy€n Ml;lnh Linh 153
CHVdNG 3. UJI GIAI cAc HE ToAN ON TAp

Nhan cac bfrt dfulg thtic tren l~i theo v8, ta suy fa

2 1
(xyz) :::;; 64'

hay
1
xyz :::;; S.
Bay gid, gQi A la s6 Wn nhfit trong cac s6 x (1 - y), y (1 - z) va z (1 - x).
Khi do, til gia thi8t, ta co

3A;?: X(1 - y) + y(1 - z) + z(1 - x)


= 1 - xyz - (1 - x - y - z + xy + yz + zx - xyz)
= 1 - xyz - (1 - x)(1 - y)(1 - z)
= 1- 2xyz

>-~4
:;.--

Ttl cac danh gia tren, ta suy ra A ~ i. Day chinh la k8t qua dn chting minh. •
each 2. Gia thi8t dfi cho co the dU<;lC vi8t l~i dudi d~ng

Do 0 < x, y, z < 1 nen ±- 1, } - 1, ~ - 1 > o. K8t h<;lp voi d~ng thtic d


tren, ta suy fa t6n t~i cac s6 dUdng a, b, c (xem them each 3 - bai 4, de
tuyen
sinh THPT chuyen DHSP Ha NQi nam 2014) sao cho

1 a
x- 1 =b'
1 b
- - 1 =-,
y c
1 c
- - 1 = -,
z a
hay
b
x=a+b'
c
y=b+c'
a
z=--
c+a
154 VO Qu6c Ba Cffn - Nguy~n Ml;mh Linh
1. cAc HE THI TUYEN SINH CHfNH THUC

Khi do, ta co
b2
x(1-y)= (b+c)(b+a)'
c2
y(1-z)= (c+a)(c+b)'
a2
z(1-x)= (a+b)(a+c)
Khong m~t tfnh tong quat, ta co th€ gia sir a = max { a, b, c}. Khi do, ta co
o < (a + b)( a + c) :::;; 4 a 2 nen suy ra
a2 1
z(1-x)= (a+b)(a+c) ~4'
Bai toan dl1<;1e ehti'ng minh xong. o
Bai 4 (3.0 c1iim). Cho dl1dng tron ( 0), dUdng kfnh A B. G9i I la di€m b~t
ky tren do:;tn th~ng AO (/ khae A, I khae 0). Dl1dng th~ng di qua I va
vuong goe vdi A B dt dl1dng tron ( 0) t:;ti cae di€m C va D. G9i E la di€m
tren dl1dng tron (0) sao eho D la di€m ehfnh giii'a ella eung A E . G9i K la
giao di€m ella AE va CD.

a) Chung minh r~ng dudng th~ng 0 K di qua trung di€m ella C E .


b) Dl1dng th~ng di qua I va song song vdi C E dt A E, BE lfin lU<;It t:;ti P
va Q. Chung minh r~ng tti' giae D P E Q la hinh chii' nh~t.

c) Tim vi trf ella di€m I tren do:;tn th~ng A 0 sao eho KC = KA + K O.

LUi giiti. a) D@ th~y A la di€m ehfnh giii'a ella eung nho CD. L:;ti co D la diSm
ehfnh giii'a ella eung nho A E nen A C = AD = DE. Suy ra tti' giae CAD E Ia
hinh thang can. Ma K la giao di€m ella A E va CD nen ta co

KC = KE.
L:;ti co 0 C = 0 E nen 0 K la dudng trung tn;ie ella do:;tn th~ng K C. Tlt day, ta
suy ra K 0 di qua trung di€m ella C E .
b) Ta co
LPEQ = 180° - LPEB = 90°. (1)
M~t khae, ta cung co L D I Q = L D C E = L DB E = L D A P nen cae tti' giae
DI BQ va AI P D nQi ti~p. Suyra

LDQE = 180° - LDIB = 90° (2)


VO QU6e Ba Cffn - Nguy@n M:;tnh Linh 155
CHl1dNG 3. LOI GIAI cAc DE ToAN ON TAp

va
LAPD = LAID = 90°. (3)
Tli (1), (2) va (3), ta suy ra tu giac D P E Q la hinh chii nh~t.
C ....·······························....
. ,
.,... \\
. )B
.. ...
"""

-- ....,'........ ,'"
'

Q
c) Ta co
LAOC = sdAC
va
1 ---
LAKC = "2(sdAC ---
+ sdDE).
--- --
Mil sdAC = sdDE nen LAO C = ,
LAKC. Suy ra tu giac AKOC nQi tiep.
Theo dinh 19 Ptolemy, ta co
AO . KC = AC . KO + KA . CO,
hay
R . KC = AC . KO + R . KA.
Thay C K = A K + 0 K vao d~ng thue tren, ta dUQe
R(KA + KO) = R· KA + AC· KO,
hay
AC = R.
Do do, tam giae A COla tam giac d€u, suy ra Ila trung digm eua dOl;ln A 0 .
V~y diem I c§.n tim ehinh la trung digm eua dOl;ln th~ng A 0 . o
156 Vo Qu6e Ba Cfin - Nguy~n M~nh Linh
1. cAc HE THI TUYEN SINH CHINH THUC

Blii 5 (1.0 c1ie'm). Cho 2015 s6 nguyen du'dng phan bi~t khong vu'<;1t qua
3019. Chung minh r~ng trong 2015 s6 do t6n t~i b6n s6 a, b, c, d sao cho

a+b+c=d.

Lui giili. GQi 2015 s6 nguyen du'dng do Ht aI, a 2, . . . , a 20 15. Khong mfitt6ng
qu:it,giasi10 < al < a2 < ... < a2015 ~ 3019. Khido,taco

a2015 = (a2015 - a2014) + (a2014 - a2013) + ... + (a2 - al) + al


~ 1 + 1 + ... + 1 +al
"'"---...,,.--.,,,1
2014 s6 1
= al + 2014.
Maa2015 ~ 3019nenal ~ 1005.D~tun = a n +al (n = 3, ... ,2015).
Khi do, ta co

U3 < U4 < ... < U2015 ~ 3019 + 1005 = 4024.

D~t vn = an - a 2 (n = 3, ... , 2015). Ta co

V3 < V4 < ... < V2015 ~ 3019 < 4024.

VI (u n) va (V n) la hai day s6 du'dng phan bi~t, g6m t6ng cQng 4026 s6, m6i s6
khong vu'<;1t qua 4024 nen theo nguyen ly Dirichlet, t6n t~i i, j E {3, . . . , 20 IS}
sao cho U i = V j . MQt cach tu'dng du'dng, ta co

hay
aj = ai +al +a2.
Ro rang i =I- j nen tu day, ta suy ra diSu phai chung minh. o

1.11. O~ thi tuy~n sinh THPT chuyen DHSP Ha NQi, 2016

Blii 1 (1.5 diim). Chung minh r~ng bi€u thuc sau nh~n gia tri nguyen du'dng
vdi mQi gia tri nguyen du'dng cua n :

Vo Qu6c Ba Cffn - Nguy~n M~nh Linh 157


CHU'dNG 3. LOI GIAI cAc DE ToAN ON TAp

4n 4 + 1 = (2n2 + 1)2 - 4n 2 = (2n2 + 2n + 1)(2n2 - 2n + 1),


4n 2 + 2 = (2n2 + 2n + 1) + (2n2 - 2n + 1),
ta co

M~t khac, ta ding co

vn 2 + (n + 1)2 + ..j(n -1)2 + n 2


= ..j2n 2 +2n + 1 + ..j2n 2 -2n + 1.
Dodo,
P = (2n 2 + 2n + 1) - (2n2 - 2n + 1) = 4n.
V~y P luon nh~n gia tri nguyen dl1dng v{ji mQi n nguyen dudng. o

Bai 2 (2.5 dim).

a) TIm tfit ea cae s6 nguyen dUdng x, y thoa man

b) TIm tfit ea cae s6 thve x, y thoa man

_x2_-_4 + y2 - 4 + 8 = 4 (y'-X---l + ..j-y---l) .


x y .

Ldi giai. a) Tit gia thi~t, d€ thfiy x > y > O. GQi (x, y) = d (d E N*) va
d~tx = da, y = db (a, b E N*, a > b, (a, b) = 1). Khido,phu'dng
trlnh dff cho co th€ dUQe vi~t l~i thanh

(1)

Suy ra 95 (a 2 + b 2) ehia h~t eho a 2 + a b + b 2. Ma

nen tit tren, ta suy ra 95 chia h~t cho a 2 + a b + b2 .


158 Vo Qu6c Ba Cftn - Nguy€n M~nh Linh
1. cAc DE THI TUYEN SINH CHfNH THUC

• N6u a 2 + a b + b 2 chia h6t cho 5 thl ta co 4 (a 2 + a b + b 2) chia h6t cho


5, hay (2 a + b) 2 + 3 b 2 chia h6t cho 5.

N6u b khong chia h6t cho 5 thl ta co b 2 chia 5 du 1 ho~c 4, suy fa 3 b 2 chia
5 du 3 ho~c 2. Tu do suy fa (2a + b)2 chia 5 du 2 ho~c 3, mau thu§:n.

Do do b chia h6t cho 5. Ma a 2 + a b + b 2 chia h6t cho 5 nen a ciing chia


h6t cho 5, mau thu§:n do (a, b) = 1.

• Do v~y, ta phlti co a 2 + a b + b 2 khong chia h6t cho 5. Ma a 2 + a b + b 2 > 1


va la udc cua 95 nen ta co

a2+ ab + b 2 = 19.
Do a > b > 0 nen 3 b 2 < a 2 + a b + b 2 = 1 9, suy ra b
E {l, 2} (do
b E N*). Thu tnJc ti6p, ta tIm du<;jc b = 2 va a = 3. Thay vao phuong
trlnh(I),tadu<;jcd = 65. Tudo, taco x = 195vay = 130.

V~y co duy nhfit mQt c~p s6 (x, y) thoa man yeu cftu d~ bai la (195, 130).
b) Di~u ki~n: x, y ~ 1. Phuong trlnh dii cho tuong duong vdi
4 4
x +4- -
x
+y +4- - = 4.Jx - 1
y
+ 4VY - 1,

hay
x + 4(x - 1) _ 4.Jx _ 1 +y+ 4(y - 1) - 4VY - 1.
x y
Phuong trlnh cu6i co th€ dU<;jc vi6t l~i thanh

x y
TIt do, ta co

lX-2.Jx-l=0,
y-2VY-l=0.
GiM h~ tren, ta dU<;jc x = y = 2. V~y phuong trlnh da cho co duy nhfit mQt
nghi~m (x, y) la (2, 2). D

Nh~n xet. Y a) cung co th€ du<;jc giai b~ng mQt cach khac "thu cong" hon nhu
sau: D~t a = x + y va b = x - y thl ta co a, bEN *, a > b va

a+b a-b
X= y- - -2- .
2
Vo QU6c Ba C§:n - Nguy€n M~nh Linh 159
CHOdNG 3. UJI GIAI cAc HE ToAN ON TAP

Thay vao phl1dng trinh da cho va nit gQn, ta dl1Qc

b (3a 2 + b 2) = 190(a
2 + b 2 ), (2)

hay
(3)
Gilt sit b khong chia h6t cho 5, khi do tu (2), ta suy ra 3 a 2 + b 2 chia h6t cho 5.
Ma b 2 chia 5 da 1 ho~c 4 nen 3a 2 chia 5 du 4 ho~c 1, tuc a 2 chia 5 du 3 ho~c
2, mau thu~n. Do do b chia h6t cho 5. (4)
2
'l1' (3) t 190-b Suy ra 3b-190
190-b l'a b'nh
.lll ,a co'ab 2 = 3b-190. l phl1dng cua mQt so h-uu tI. (5) ? A '" ?

1:::'-;10 > 1, suy ra 1~0 < b < 95. K6t hQp vdi (2),
Ngoai ra, do a 2 > b 2 nen 3
ta suy ra b E {65, 70, 75, 80, 85, 90}. Trong t~p hQp s6 nay, chi co duy
nhfrt b = 65 thoa man (5). Do do b = 65. Thay vao (3), ta dUQc a = 325. TIt
do, ta co x = 1 95 va y = 13 °.
Bai 3 (2.0 c1iim). Cho S la t~p cac s6 nguyen dl1dng n co d';lng n = x 2 + 3 y 2 ,
trong do x, y la cac s6 nguyen. Chung minh r~ng

a) N6u a, b E S thi ab E S.

b) N6u N E S va N ch~n thi N chiah6tcho 4 va ~ E S.

Uti gicii. a) Gilt sit a = m 2 + 3n 2 vab = p2 + 3q 2 vdim, n, p, q E Z.


Khi do, ta co

ab = (m 2 + 3n 2 )(p2 + 3q2) = (mp + 3nq)2 + 3(mq - pn)2.

Do m p + 3 n q, m q - p n E Z nen abE S.
b) Gilt sit N = x 2 + 3y2 vdi x, yES va N ch~n. D8 thfry x va y cling tinh
chfuI Ie. Xet cac trl1dng hQp sau:
• TrLiong hQ'p 1: x, y citng chdn. Khi do, d8 thfry N chia h6t cho 4 va

N (X)2
""4 = 2" + 3 (y)2
2" E S.

• TrLiong hQ'p 2: x, y citng Ii. Khi do, ta co x 2 chia 4 du 1, y2 chia 4 du


1 nen N chia h6t cho 4.

o N€u x, y co cimg s6 dl1 khi chia cho 4, ta co

160 Vo Qu6c Ba Cfin - Nguyen M';lnh Linh


1. cAc HE THI TUYEN SINH CHINH THUC

Dodo
~ =c:3Yr+3(X~YrES.
o N~u x, y khong cling s6 du khi chia cho 4, ta co x +y chia h~t cho 4
va

Dodo

~=c~3Yr+3c:YrES.
Tom l~i, trong mQi tn],dng h<;5p, ta d€u co N chia h~t cho 4 va ~ E S. D

Biti 4 (3.0 c1iim). Chotamgiac ABC nhQn, AB < AC. Kedu'dngcao AH.
DUdng tron ( 0) du'dng kfnh A H dt cac c~nh A B, A C tu'dng ang t~i D va
E . DUdng th&ng DE c£t du'dng th&ng B C t~i S.

a) Chang minh r~ng B DEC la tu giac n9i ti~p.


b) Chang minh r~ng S B . S C = S H 2.
c) DUdng th&ng SO dt A B, A C tu'dng ang t~i M va N, dUdng th&ng
DE dt H M, H N tu'dng ung t~i P va Q. Chung minh r~ng ba du'dng
th&ng BP, C Q va AH d6ng quy.

Uti giili. a) Do tli ghic AD H E n9i ti@p nen L. A DEL. A HE (cling ch~n
cung AE). L~i co L.AH E = L.ACB (cling phlJ voi L.H AC) nen

L.ADE = L.ACB.

Tit do, ta co

L.BDE + L.ECB = L.BDE + L.ADE = 180°.


Suy ra ta giac B DEC n9i ti~p.
b) Ta co S H la ti~p tuy@n cua du'dng tron (0) t~i H nen L. DE H = L. D H S
(cling ch~n cung D H). Tit day, ta d~ dang suy ra l:::, S D H '" l:::, SHE (g-g). Suy ra
SD SH
SH SE'
hay
(1)

Vo Qu6c Ba Cffn - Nguy~n M~nh Linh 161


CHUdNG 3. LOI GIAI cAc HE ToAN ON TAp
M~t khac, do tu giac BDEC n9i ti€p nen LSDB = LSCE (cling bli vdi
L B DE). Ttl do, ta chung minh dl1Qc f::. S B D rv f::. SEC (g-g), suy fa
SB SE
SD SC'
hay
SB·SC=SD·SE. (2)
Tti (1) va (2), ta suy fa S B . S C = S H 2.

K S B H C

c) Ta chung minh AM H N la hinh binh hanh, ttl do suy fa B P, C Q Ia cac dl1dng


cao clla tam giac ABC. LAy K d6i xung vdi H qua S, ta co SOil A K. Do do
BM BS BS
(3)
MA SK
Theo diu b), ta co S B . S C = S H 2. Ma S C = S H + H C nen
SB· HC = SH(SH - SB) = SH· BH,
hay
SB BH
(4)
SH - HC·
Tti (3) va (4), ta co ft~= Z~. Suy fa M H II AC va
LBHP + LBDP = LBCE + LBDP = 180°.
Do do, tu giac B D P H n9i ti€p, suy fa B P ..1 M H. Ma M H II A C nen ta co
B P ..1 A C, Wc B P la dl1dng cao clla tam giac ABC.
Chung minh tuang tV, ta cung co C Q la dl1dng cao clla tam giac ABC. Do v~y,
cac dl1dng th~ng A H, B P va C Q d6ng quy. 0
162 V6 Qu6c Ba Cffn - Nguy~n M~nh Linh
1. cAc HE THI TUYEN SINH CHINH THUC

Nh~n xet. Dg chung minh H M II A C va H N II A B thi ngoai cach dUQc


trinh bay (j tren, ta con co thg chung minh b~ng cach khac dlla tren dinh ly
Menelaus nhu sau: Ke H M' I A C va H N' II A B (M' E A B, N' E A C ) .
Ta co tu giac AM' H N' la hinh binh hanh nen 0 la trung digm cua A H thi
cling la trung digm cua M' N' . (5)

A
.......................
.............
\
'

...•

j
IE

S B H c
Sit dVng dinh ly Menelaus cho tam giac ABC va cat tUYc3n S DE, ta co

SB EC DA _ 1
SC . EA ' DB - .

Ma ~ ~ = ~ ~ ~ va g~ = Z~ ~ (h~ thuc luc,mg trong tam giac vuong) nen


2
SB HC _ 1
SC ' HB2 - .

LC;li co ~~ = ~:~ = ~;~ (theo dinh ly Thales) nen

SB N'C M' A
SC'N'A'M'B=1.
Ap dvng dinh ly Menelaus, ta co ba digm S, M', N' th~ng hang, (6)
Til (5) va (6), ta suy ra b6n digm S, M', 0, N' th~ng hang. Suy ra M' trung M
va N' trling N. Do do, ta co H M II A C va H N II A B,

Bai 5 (1.0 diem). Gift SU' m5i digm tren m~t ph~ng dUQc to bdi mQt trong ba
mau xanh, do, vang. Chung minh r~ng t6n tC;li ba digm dUQc to cling mau la
ba dinh cua mQt tam giac can.

Vo Qu6c Ba C§:n - NguySn M,.mh Linh 163


CHUONG 3. LOI GIAI cAc HE ToAN ON TAp

Uti giai. Xet da giac d~u A 1 A2 ... A 10, nQi ti~p dUdng tron Him O. Khong m~t
tinh t6ng quat, t~ gift sir di€m 0 du'Qc to m~lU do.
N~u co hai dinh X, Y dUQc to mau do thl tam giac 0 X Y thoa man.
N~u co khong qua mQt dinh cua da giac dUQc to mau do thi t6n t~i it nh~t 5 dinh cung
du'Qc to mau xanh ho~c mau yang. Gift sir co 5 dinh dUQc to mau xanh. Chia cac dinh
cua da giac da cho thanh hai nhom: Nhom 1 g6m cac dinh A 1, A 3, A 5, A 7, A 9
va nhom 2 g6m cac dinh A 2, A 4, A 6, A 8, A 10 .
Theo nguyen If Dirichlet, co ba dinh mau xanh thuQc cung mQt nhom, ch~ng h~n la
nhom 1. VI A 1 A 3 A 5 A 7 A 9 la ngli giac d~u nen ba dinh do t~o thanh mQt tam giac
din va ta co di~u phfti chung minh. 0
, ?

1.12. De thi tuyen sinh THPT chuyen KHTN, 2016 (vong 1)

Bai 1 (3.5 c1iim).

a) GiM h~ phuclng trinh:

l
x3+y3+XY(x+y) =4,
(xy + l)(x 2 + y2) = 4.

b) Gifti phu'clng trinh:

8x - 3
,J7x+2-,J5-x= 5 .

Lui giai. a) DB th~y x 2 + Y2 o. Bay gid, ta vi~tl~i h~ phuclng trinh du'oi d~ng
=1=

(x + Y)(X2 + y2) = 4,

1(xy + l)(x 2 + y2) = 4.


Tit do, ta suy fax + y = x y + 1, hay
(x - l)(y - 1) = o.
Do do x = 1 ho~c y = 1.
• Voi y = 1, thay vao phu'clng trinh thu nh~t, ta dUQc x 3 + X 2 + X - 3 = 0,
hay (x -1)(x 2 +2x + 3) = O. Tit'do, taco x = 1 (do x 2 +2x + 3 > 0).

• Tu'clng tlj, voi x = 1, ta cling co y = 1.


V~y h~ phuclng trinh da cho co duy nh~t mQt nghi~m (x, y) la (1, 1).

164 Vo Qu6c Ba Cffn - Nguy~n M~nh Linh


1. cAc DE THI TUYEN SINH CHINH THDc
b) Di€u ki~n: - t : :; x :::; 5. Phlldng trlnh da cho tudng dlldng voi
8x - 3 8x - 3
(1)
v'7x+2+v'5-x 5

Ro dmg x = ~ la mQt nghi~m cua phlldng trlnh. Xet tntdng h<Jp x 1= ~, khi do
phlldng trlnh (1) co thg dll<JC vi~t l:;ti dlloi d:;tng

v'7x +2+ v'5-x = 5.

CQng phlldng trlnh nay voi phlldng trlnh da cho theo v~, ta dll<JC

/ 4x + 11
-y7x+2= .
·5

Binh phlldng hai v~ cua phlldng trlnh tren, ta dll<JC

25(7x + 2) = (4x + 11)2,


hay
(x - 1)(16x - 71) = o.
Suy ra x = 1 ho~c x = i ~ . Thi11:;ti, ta thAy chi co x = 1 thoa man.
V~y phlldng trlnh da cho co hai nghi~m la x = 1 va x = ~. D

Bai 2 (2.5 c1iim).

a) Tim tAt ca cac gia tri cua tham s6 m sao cho t6n t:;ti c~p s6 nguyen (x, y)
thoa man h~ phlldng trlnh

+ mx y2 = 3m,
!
2
2 + m (x 2 + y2) = 6m.

b) Voi x, y la nhi1ng s6 thvc thoa man cac di€u ki~n 0 < x :::; y :::; 2 va
2x + y ;? 2x y , tim gia tri IOn nhAt cua bigu thlic

L?,i giai. a) Tit h~, d6 thAy m 1= O. Trli hai phlldng trlnh cua h~ tlldng ling theo
ve voi chti y m # 0, ta dll<JC

xy2_x 2 _y2=_3,

Vo QU6c Ba Cffn - Nguy6n M:;tnh Linh 165


CHlJdNG 3. LOI GIAI cAc HE ToAN ON TAp

hay
(x - l)(y2 - X - 1) = -2. (1)
Do do x - I la tidc cua 2, suy ra x E {- 1, 0, 2, 3}. Thit tnic ti€p, ta thAy cac
c~ps{)nguyen(x, y)thoaman(I)la(2, 1), (2, -1), (-1, l)va(-I, -1).

• Vdi (x, y) = (2, ± 1), thay vao h~, ta co m = 2.


• Vdi (x, y) = (- 1, ± 1), thay vao h~, ta co m = t.
V~y co hai gia tri m thoa man yeu cftu d~ bai Ia m = 2 va m = t.
b) Ta se chling minh P :::;; 22 vdi dAu dang thuc xay ra khi va chi khi x l ,
y = 2. D~t a = ~ va b = }, khi do ta co a ~ b ~ t, a + 2 b ~ 2 va
a2 + 1 b2 + 1
P=
a4
+---
b4
BAt dang thlic P :::;; 22 tl1dng dl1dng vdi

( 2-
a2
a
+
4
1) + ( 20 -
b
2
+
b4
1) ~ 0,

hay
(a - 1)X + (2b - I)Y ~ 0,
trong do

x = (a + 1) (2a
a4
2
+ 1) (.!. + _1 ) (2 + ~) ,
=
a a2 a2
+ 1) = (2 _1) ( _1 )
2
Y = (2b + 1)(5b
b4 + b + b2 5 b2 .

Do a ~ b ~ t nen d~ thAy Y > X > O. Tu do, ta co

(a - I)X + (2b - I)Y ~ (a - I)X + (2b -l)X


= (a + 2b - 2)X
~ O.

DAu dang thlic xay ra khi va chi khi 2 b - 1 = 0 va a + 2b - 2 = 0, tlic x = 1


va y = 2. V~y max P = 22. D
N h~ n xet. a y b), d~ chling rninh gia tri cua P khong vl1<;'t qua 22, ta con
co mQt cach khac nhl1 sau: N€u 0 < x < 1 thi ta co X 2 (X 2 + 1) < 2 va
y 2 (y 2 + 1) ::::; 20 (do 0 < y :::;; 2) nen hi~n nhien P < 22.

166 Vo Qu{)c Ba DIn - Nguy~n M,.mh Linh


1. cAc HE THI TUYEN SINH CHfNH THUC

X6t tntdng h<;jp 1 ~ x ~ y ~ 2. Khi d6, ta e6 (x 2 - 1)( X 2 - 4) ~ 0, suy ra


2 2
x (x + 1) = (x 2 - 1 )(x 2 - 4) + 6x 2 - 4 ~ 6x 2 - 4.
M~t khae, ta cling e6 (x - 1)( x - 2) ~ 0 nen x 2 ~ 3 x - 2, suy ra
2 2
X (X + 1) ~ 6x 2 - 4 ~ 6(3x - 2) - 4 = 18x - 16.

Chang minh tttdng 111, ta cling e6


y2(y2 + 1) ~ 18y - 16.

Suyra
P ~ 18 (x + y) - 32. (2)
Do 2x +Y ~ 2x Y > 0 nen
2(x + y) ~ (x + y)(2x + y) = (x - y)(2x - y) + 6.
xy xy

Ta co x - y ~ 0 va 2 x - Y ~ 2 - Y ~ 0, do do 2 (x + y) ~ 6. Ttl' day, k€t


h<;jpvoi(2),tasuyraP ~ 9·6-32 = 22.

Bai 3 (3.0 cfiim). Cho tam giae ABC nhQn khong can (AB < AC) nQi
ti€p dl10ng tron (0). Ph an giae eua g6e B A C c~t dttong th~ng B C t~i D
va e~t dttong tron ( 0) t~i E khae A. GQi M la trung diem eua do~n th~ng
AD. Dttong th~ng B M e~t dttong trim ( 0) t~i P khae B. Gilt cae dttong sa
th~ng E P va A C e~t nhau t~i N.

a) Chang minh ding b6n diem A, P, N, M eung n~m tren mQt dttdng
trim va N la trung diem eua do~n th~ng A C .

b) Gilt sa dl1dng tron (K) ngo~i ti€p tam giae EM N e~t dttong th~ng A C
t~i Q khae N. Chling minh r~ng B va Q d6i xling nhau qua A E .
sa
c) Gilt dl1dng tron (K) e~t dttong th~ng B M t~i R khae M. Chling minh
r~ng R A vuong g6e voi R C .

La. gifli. a) Ta co LM P N = LBAE = LCAE nen tli giae AP N M nQi


ti€p. Suy ra LAN M = LAPM = LACB va do d6 M N II BC. Ma M HI.
trung di~m eua A D nen N Ia trung diem eua A C .
b) Do M N II B C va tli giae M N QE nQi ti€p nen ta e6
L.MEQ = LMNA = LACB = LAEB,
suy ra 6. A E B = 6 A E Q (g-e-g). Tu d6, d6 thfiy B, Q d6i xling nhau qua A E .

Vo Qu6c Ba Cftn - Nguy~n M~nh Linh 167


CHUONG 3. LOI GIAI cAc DE ToAN ON TAP

c) Tlwng 1\1 nhtl diu b), ta co .


LREN = LPMN = LPBC = LPEC,
suy fa 1::,. PRE = 6. P C E (g-c-g) nen R va C d6i xl1ng nhau qua P E . Tli do,
ta co N R = N C = N A, suy fa LA R C = 90°. 0

Bfti 4 (1.0 c1iim). S6 nguyen a dllQC g<;>i la s6 "d~p" n~u voi m<;>i cach siip x~p
theo thl1 tt! tuy y cua 100 s6 1, 2, . . ., 1 00 luon t6n tl;li 10 s6 hl;lng lien ti~p
eo t6ng IOn hon ho~c b~ng a. TIm s6 "d~p" Ion nhk

La. gifJi. Gilt si'i Xl, la mQt hoan vi tuy Yeua cac s6 1, 2, ... ,
X2, ••. , X100
100. Ta chia thanh 10 nhom, m6i nhom g6m t6ng 10 s6 hl;lng lien ti~p:
Sl = Xl + X2 + ... + X10, ... ,
·Taco
Sl+S2+',,+S10 = X1+X2+"'+X100 = 1+2+ .. ·+100 = 5050
nen eo it nhfit mQt chi s6 i (i EN, 1 :s:; i :s:; 10) sao cho Si ~ 5~~0 = 505.
V~y a = 505 la mQt s6 "d~p".
Ta xet mQt hoan vi d~c bi~t 1 00, 1, 99, 2, 98, 3, ... , 5 1, 50. Khi do, t6ng
eua 10 s6 h<;tng lien ti~p ho~e b~ng 505 (n~u s6 d~u tien Ia IOn nh!t) ho~e b~ng
500 (n~u s6 dfru tien la nh6 nh!t). Suy fa, n~u a > 505 thi a khong Ia s6 "dtfp".
V~y max a = 505. 0

168 Vo Qu6c Ba C~n - Nguy~n Ml;lnh Linh


1. cAe HE THI TUYEN SINH CHINH THUC

1.13. D€ thi tuy~n sinh THPT chuyen KHTN, 2016 (vang 2)

Bai 1 (3.5 dim).

a) Giiii h~ phudng trinh:

X +4 y2 =5,
2

! 4x 2 y + 8xy2 + 5x + lOy = 1.

b) GiM phudng trinh:

/ 2 64x 3 + 4x
v 5x + 6x + 5 = 5x2 + 6x + 6 .

LUi giai. a) H~ phudng trinh da cho co th€ du<jc vi€t l~i du(ji d~ng

x 2 + 4y2 = 5, (1)

!(x + 2y)(5 + 4xy) = 1. (2)

Thay (1) vao (2), ta du<jc

(x + 2y)(x 2 + 4y2 + 4xy) = 1,

hay
(x + 2y)3 = 1.
Tlt do, ta co x = 1 - 2 y. Thay vao (2), ta du<jc

5+4y(1-2y) = 1,

hay
(1 - y)(l + 2y) = O.
Tlt day, ta co y = 1 (tudng ling, x = -1) ho~c Y = -t (tudng ling, x = 2).
V~y h~ da cho co cae nghi~m (x, y) la ( - 1, 1) va (2, - t) .
b) Do 5 x 2 + 6 x + 5 = 3 (x + 1) 2 + 2 x 2 + 2 > 0, vX E ffi. nen phlldng
trinh luon xac dinh v(ji mQi x E ffi.. D~t a = J 5x 2 + 6x + 5 va b = 4x. Khi
do, phlldng trinh da cho co th€ du<jc vi€t l~i thanh

b3 +b
a = a2 + l'

Vo Qu6c Ba Cftn - Nguy~n M~nh Linh 169


CHUONG 3. UJI GIAI cAc HE ToAN ON TAP

hay
a3 +a = b
3
+ b.
N€u a > b thi VT > VP, con n€u a < b thi VT < VP. Do do, a = b, hay
.j 5x 2 + 6x + 5 = 4x.

Tit phlidng trinh nay, ta suy fax > 0 va

5x 2 + 6x + 5 = 16x 2 •

Tit do, ta co x = 1. V~y phuong trinh da cho co nghi~m duy nhfit x = 1. 0

Bili 2 (2.5 c1iim).


2
a) Vdi x, y la nhung s6 nguyen thoa man d~ng thac x -1
2
= y2 -
3
1
, chang
minh f~ng x 2 - Y 2 chia h€t cho 40.

b) TIm tfit eft cac c~p s6 nguyen (x, y) thoa man d~ng thac

lCii giai. a) Gift thi€t da cho co th€ dU<;IC vi€t I~i thanh
3x 2 - 2y2 = 1. (1)

Tit day, ta co x Ie nen x 2 chia 8 du 1, suy fa 3 x 2 chia 8 du 3. Tit do, ta co 2 y 2


chia 8 du 2. Suy fa y Ie va y2 chia 8 du 1. Do do x 2 - Y 2 chia h€t cho 8. (2)
Ti€p theo, ta se chang minh x 2 - Y 2 chia h€t cho 5. (3)
Chu y f~ng s6 du cua a 2 (a E Z) khi chia cho 5 la 0, 1, 4 .

• N€u y2 chia h€t cho 5 thi ttl (1), ta co 3x 2 chia 5 da 1, mau thufin do s6 du
cua 3 x 2 khi chia 5 chi co th€ Ia 0, 3, 2.

• N€u y 2 chia 5 du 4 thi ttl (1), ta co 3 x 2 chia 5 du 4, mau thufin.

• Do do, ta phiii co y2 chia 5 du I. Khi do, ttl (1), ta cling suy fax 2 chia 5 da
1. Dfin d€n x 2 - y2 chia h€t cho 5.

Tit (2) va (3), vdi chu y (5, 8) = 1, ta thu dU<;Ic x 2 - Y 2 chia h€t cho 40.
b) Phuong trlnh da cho co th€ dU<;Ic vi€t I~i thanh

(x 2 + 1)2 = (y + 1 )(y2 - Y + 1). (4)


170 Vo QU6c Ba Cffn - NguySn M~nh Linh
1. cAc HE THI TUYEN SINH CHfNH THUC

Tli day, ta co y ~ -1. Ngoai ra, do x 2 khong th€ chia 3 du 2 nen x 2 + 1 khong
chia h€t cho 3, suy ra y + 1 va y 2 - Y + 1 dSu khong chia h€t cho 3. (5)
Ta se chdng minh
(y + 1, Y2 - Y + 1) = 1. (6)
sit y + 1 va y 2 - Y + 1 co udc nguyen t6 chung la p. Khi do, ta co
Th~t v~y, gia
y2 - Y + 1 = (y + l)(y - 2) + 3 nen 3 chiah€tcho p, suyra p = 3. K€t
qua nay mau thufin vdi (5).
Tli (4) va (6), ta suy ra y + 1 va y2 - Y + 1 la cac s6 chfnh phuong. D~t
y2 _ Y + 1 = a2
vdi a E N thl ta co

hay
(2a -12y -11)(2a + 12y -11) = 3.
Do 2a - 12 y - 11 < 2a + 12 y - 11 va 2a + 12 y - 11 > 0 nen ttt tren, chi
co mQt trl1dng h<;lp sau xay ra:

i 2a - 12y - 11 = 1,
l2a + 12y - 11 = 3.
Giiii h~ tren vdi chu y y + 1 la s6 chfnh phuong, ta dU<;lC y = O. Thay tn!c ti€p
trd l~i phuong trlnh da cho, ta du<;lc x = O.
V~y co duy nhfit mQt c~p s6 nguyen (x, y) thea man yeu cftu la (0, 0). D

Bai 3 (3.0 diim). Cho hlnh vuong ABC D nQi ti€p dUdng tron tam o. P la
di€m thuQc cung nhe AD cua dUdng tron (0) va P khac A, D. Cac dudng
th~ng P B, P C l~n lU<;It dt dudng th~ng A D t~i M, N. Dudng trung trlJe
eua A M c~t cae dUdng th~ng A C, P B l~n lu<;It t~i E, K. DUdng trung trlJe
eua D N dt cae dudng th~ng B D, P C l~n lu<;It t~i F, L.

a) Chdng minh r~ng ba di€m K, 0, L th~ng hang.


b) Chdng minh r~ng duCJng th~ng P 0 di qua trung di€m cua do~n thkg E F .

c) Giii sit dUdng th~ng E K dt dudng th~ng B D t~i S, cac dudng


th~ng F L va A C dt nhau t~i T. Dudng th~ng S T dt cac dUdng
th~ng PC, P B l~n lU<;lt t~i U va V. Chdng minh r~ng b6n di€m
K, L, U, V cling n~m tren mQt dudng tron.

Vo Qu6c Ba Cffn - Nguy~n M~nh Linh 171


CHlJdNG 3. UJI GIAI cAc DE ToAN ON TAp
Uti giai. a) Vi tam giac C N D vuong t~i D nen L Ut trung diem cua N C. L~i
co 0 la trung diem cua A C nen 0 L II AD. Tu'dng tv, ta ding co 0 K II AD.
Do do, ba diem 0, K, L th~ng hang.

B ....................................................., ...., C

.'

".
~----~~~~----------~
A ..... D
.'.'
................................
P

b) GQi Q la giao diem cua EM va PO. Ta co tam giac E A M vuong din t~i E
nen L EM A = 45°. Do do L EM A = L B D A = 45 ° , suy ra M Q II B 0 .
Tl1dng tv, ta cung co N F II CO. (1)
Do MQ II BO nen taco
PM PQ
PB PO
M~t khac, do M N I B C nen ~~ = ~ ~ , suy fa

PQ PN
PO - PC'
hay
QN II ~C. (2)
Tit (1) va (2), ta suy fa ba diem Q, N, F th~ng hang. Ttl' do, ta co tti ghic 0 F QE
la hinh chfr nh?t nen PO di qua trung digm R cua E F .
c) Theo tfnh chAt d6i xting thi S T, E F, K L d6ng quy t~i G (truong hqp P la
diem chinh giua cung AD thi S T, E F, K L song song vdi nhau nen d~ suy ra
U, V, K, L cung thuQc mQt du'Clng tron).
172 Vo QU6c Ba Cftn - Nguy€n M~nh Linh
1. cAc HE THI TUYEN SINH CHfNH THUC

G9i H la giao diem eua P 0 va S T. Theo cau b) thl R 0 = R F, L 0 = L F nen


R L la phan ghic eua L FRO. M~t khac, ta cling co G L la phan giac cua LTG R
nen L Ia tam dUdng tron nQi ti8p cua tam giac G HR. Tit day, do L R L G = 1 3 5 °
nen LRHG = 90°. Suyra UV l.. PO.
Ta co tam giac 0 P C can tl;l.i 0 nen

LOPC = LOCP = LABP = LVKS.

Ma L H P U + LV U L = 90° va L S K 0 = 90° nen

LVUL + LVKL = 180°.

Do do, U, V, K, L cling thuQc rn9t dUdng tron. o


Bai 4(1.0 diim). Chang minh r~ng voi rnQi s6 tll nhien n ~ 3 luon tbn
tl;l.i mQt cach s~p x€p bQ n s6 1, 2, . . . , n thanh Xl, X 2, . . . , X n sao cho
x j =I- x i ~ X k voi rnQi bQ chi s6 (i, j, k) rna 1 ~ i < j < k ~ n.

La. gia i. Dayal, a 2, . . . , a s co chi~u dai s ~ 3 dUQc g9i la day "t6t" n€u
a j =I- a i ~ a k voi rn9i bQ chi s6 (i, j, k) rna 1 ~ i < j < k ~ s.
Ta co nh~n xet sau: N€u day aI, a2, ... , as la day "t6t" thl day 2a 1, 2a2, ... ,
2a s va day 2a 1 - 1, 2a2 - 1, ... , 2a s - 1 cling la day "t6t".
Tit day, ta suy ra rnQt nh~n xet khac nhu sau: N€u day Xl, X 2, . . . , x s la day "t6t"
dUQc sinh ra tit sl1 s~p x€p cac s6 1, 2, . . . , s (s ~ 3) thl day 2 Xl, 2 x 2, . . . ,
2 x s, 2 x 1 - 1, 2 x 2 - 1, . . . , 2 x s - 1 la day "t6t" cua cac s6 1, 2, . . . , 2 s
(chli y r~ng 2Xk+~Xm-l khong Ia s6 nguyen).

Ta co (1, 3, 2) la day "t6t" dUQc sinh ra tit sl1 s~p x€p cac s6 1, 2, 3. Do do,
thea nh~n xet tren, ta suy ra tbn tl;l.i day "t6t" dUQc xay dt;ing tit sl1 s~p x€p cac 86
1, 2, . . . , 3 . 2 m voi rn9i m tt;i nhien.
Bay gid, d€ Yr~ng voi rn6i s6 tV nhien n ~ 3, luon tbn tl;l.i s6 nguyen duong k d€
3 . 2 k - 1 < n ~ 3 . 2 k • Nhu da chang minh (j tren, ta xay dllng dUQc day "t6t"
tit cac s6 1, 2, ... , 3 . 2 k . Bo di cac s6 n + 1, n + 2, ... , 3 . 2 k , ta nh~n
dUQc day "t6t" tit cac s6 1, 2, ... , n. (Chu y r~ng tren day "t6t", n8u ta bo di cac
s6 hl;l.ng b&t ky thl day con Il;l.i v§n la day "t6t".) 0

Vo QU6c BaC§:n - Nguy6n Ml;l.nh Linh 173


CHVdNG 3. LOI GIAI cAc HE ToAN ON TAP

1.14. O€ thi tuy€n sinh THPT chuyen, SO' GD-DT Ha NQi,


2016 (danh cho chuyen Toan)

Bai 1 (2.0 diim).

a) Giai phuong trinh:


X4 - 2x 3 +X- J2(x 2 - x) = O.
b) Gild h~ phuong trlnh:
X2 + 2y - 4x = 0,

14x 2 - 4xy2 + y4 - 2y + 4 = o.

Uti giai. a) Di€u ki~n: x ~ 1 ho~c x ~ O. Phuong trlnh da cho Wong duong v6i
(x 2 - X)2 - (x 2 - x) - J2(x 2 - x) = O.
D~t ( = .j 2 ( X 2 - x) (( ~ 0). Phuong trinh tren co thg dUQc vi~t h;ti thanh
(4 (2
----(=0
4 2 '
hay
((t - 2)((2 + 2( + 2) = o.
Suy ra ( = 0 ho~c ( = 2.
• V6i ( = 0, ta co x 2 - X = O. Do do x - 0 ho~c x = 1.

• V6i t = 2, ta co x 2 - X = 2. Do do x =- 1 ho~c x = 2.
V~y phuong trlnh da cho co 4 nghi~m la - 1, 0, 1, 2.
b ) H~ phuong trlnh da cho tuong duong v6i

X2 - 4x + 2y = 0,
1(2x - y2)2 + 4 - 2y = O.

CQng tuong dng hai phuong trlnh tren theo v~, ta dUQc

(2x - y2)2 + (x - 2)2 = O.

Tv do, ta co x = 2 va 2x = y2. Suy ra y = 2 ho~c y = -2. Thi't l~i, ta thfty


chi co c~p s6 (x, y) = (2, 2) thoa man.
V~y h~ phuong trlnh da cho co nghi~m (x, y) duy nhftt la (2, 2). 0
174 Vo Qubc Ba C~n - Nguy~n M~nh Linh
1. cAc HE THI TUYEN SINH CHINH THUC

Bai 2 (2.0 die'm).

a) Cho cac s6 thvc a , b, e d6i m9t khac nhau thoa man a 3 + b 3 + e 3 = 3 abe
va abe =I O. Tinh gia tri cua bi€u thuc

b) Tim t~t ca cac c~p s6 tv nhien (x, y) thoa man

2 x . x 2 = 9y2 + 6y + 16.

LcYI giai. a) Ta co .

a
3
+ b3 + e3 - 3abe = (a + b + e)(a 2 + b 2 + e 2 - ab - be - ea).

M~t khac, do a, b, e phan bi~t nen

Tit day, k6t h<Jp vdi gia thi6t, ta suy ra a +b +e = O. Do do

ab 2 b
a 2 + b 2 - ( -a - b)2 -2ab 2

Chung minh tttong tv, ta cling co

be 2 e ea 2 a
b2 +e 2 -a 2 -'2' e 2 +a 2 -b 2 2
Tit do, ta co
p = _ a + b + e = O.
2
b) Do 9 2
+ 6y +
16 chia 3 dtt 1 nen tu gia thi6t, ta co 2 X • x 2 chia 3 dtt 1. Suy
ra x kh6ng chia h€t cho 3 va x 2 chia 3 dtt 1 . Tit day, ta co 2 x chia 3 dtt 1 nen x
Ia s6 chan. D~t x = 2k (k E N), khi do phttong trinh da cho co th€ vi6t l~i thanh

hay
2 2k + 2k 2 = (3y + 1)2 + 15.
Vo Qu6c Ba Cffn - Nguy~n M~nh Linh 175
CHVdNG 3. LOI GIAI cAc HE ToAN ON TAP

Tu day, ta co
(2 k + 1 k - 3y - 1)(2 k + 1 k + 3y + 1) = 15.

Dok, y EN,2 k + 1 k-3y-l < 2 k + 1 k+3y+lva2 k + 1 k+3y+l > 0


.nen xay fa cac tn1dng hQp 8au:
• Tnfdng hQ'p 1: 2k+ 1 k - 3y -1 = 1, 2k+1k + 3y + 1 = 15.Trongtntdng
hQp nay, ta co 2k +1 k = 8. Khong t6n t~ kEN thoa man.

e =
Tnidng hQ'p 2: 2k+1 k - 3y - 1 3, 2k+1 k + 3y + 1 5. Trong tn1dng =
hQp nay, ta co y = 0 va 2k + 1k = 4. Suy ra k = 1 (tuong ung, x = 2). .

V~y co duy nhfitmQt c~p 86 ttl nhien (x, y) thoa man yeu cAu la (2, 0). . D

Bai 3 (2.0 diim).

a) Cho cac 86 duong a, b, e thoa man a 2 + b2 + e 2 = 3. Chung minh rkg

2a 2 2b 2 2e 2
--b-2 +b
a+ +e
2+ e+a 2~a+b+e. (1)

b) Cho 86 nguyen duong n thoa man A = 2 + 2 -J 12 n 2 + 1 la 86 nguyen.


Chung minh r~ng A Ia 86 chinh phuong. .

Uti gi<ii. a) Bfit d~ng thuc da cho co th€ dUQc vi8t l~i thanh
2 2 2
2 (a _ a 2) + 2 (b _ b 2) + 2 (e _ _ e 2) ~a+b+e,
a +b b +e e +a
hay
2ab 2 2be 2 2ea 2
+ b 2 + 2 ~ a + b + e.
a+ b 2 +e e+a
Sa d\lng bfit d~ng thuc AM-GM, ta co

2ab 2 2ab 2 ~b2 ab +b.


------,--:-2 ~ = v aD" ~
a +b 2~ 2
Danh gia tUdng ttl, ta cung co

2be 2 . be +e
---~---
b + e2 ~ 2 '
2ea 2 ea +a
---~---
e +a 2
2
176 VO Qu6c Ba Clln -' Nguy€n M<;lnh Linh
1. CACDE THI TDYEN SINH CHINHTHUC

Do do, ta chi dn chung minh

ab + be + ea + a + b + e
2 ~a+b+e,

hay
ab + be + ea ~ a + b + e.
Bay gio, slidlJng bfit d~ng thuc AM-GM, ta co

a2 +1 b2 +1 e2 +1
a +b +e ~ 2 + 2 + 2 = 3.

Tit day va bfit d~ng thuc quen thuQc 3 ( a b + b e + e a) ~ (a + b + e) 2 (co th€


chung minh b~ng bi~n d5i to'dng dO'dng), ta dO'Qc

(a + b + e)2
ab+be+ea~ 3 ~a+b+e.

Dau d~ng thuc xay fa khi va chi khi a =b=e = 1.


b) Tit gia thi~t, ta co A > 2 va

(A - 2)2 = 4(12n2 + 1).


Suy fa A ch~n. D~t A = 2x vdi x E N*, khi do ta co

(x _1)2 = 12n2 + 1.
Do do x la s6 ch~n. D~t x = 2 Y vdi yEN * thi ta co A = 4 Y va

(2y - 1)2 = 12n2 + 1,


hay
y(y - 1) = 3n 2 • (2)
Suy fa y ho~c y - 1 chia h~t cho 3 .

• Tnfdng hQ'p 1: y ehia hit eha 3. D~ng thuc (2) co th6 dO'Qc vi~t l~i thanh

2
Y.(y-l)=n
3 .

Do (y, y - 1) = 1 nen (t, y - 1) = 1. K~t hQp vdi k~t qua tfen, ta


dO'QC y - 1 la s6 chinh phO'dng, mau thu§:n vi y - 1 chia 3 dO' 2 va khong
co s6 chinh phO'dng nao chia 3 dO' 2 .

Vo Qu6c Ba C§:n- Nguy~n M~nh Linh 177


.. ? ". ~ , .... ....

CHl1dNG 3. LOIGIAI CAe DE TOAN ON T~P

• TrliCtng hC1P 2: y- - I chia hit cho 3. D~ng thuc (2) co th€ vi~t l~i thanh
y - 1 2
y. = n
3

Do (y, y - 1) = 1 nen (y, Y;l) = 1. Til' do, k~t hQp v{ji k~t qUli tren, ta
suy ra y la s6 chinh phlidng. Keo theo A = 4 Y la s6 chinh phlidng. Ta co
diSu phiii chung minh. 0

Nh.n xet. Ya) con co th€ giiii b~ng cach khac nhli sau: Sa dl;lng bfit d~ng thuc
AM -GM cho hai s6 dlidng, ta co

a2 + 2b 2 + 1 .
Danh gia tlidng tlj, taciing co

2b 2 4b 2
---~-----
b + c :;.-- b + 2c + 1 '
2 2 2

2c 2 4c 2
---~-----
2
C + a c 2 + 2a 2 + 1

Tit day, sa dl;lng bfit d~ng thuc Cauchy-Schwarz d~ng c9ng mftu, ta co
2 2
VT ~ 4a 2 4b 4c
(1) :;.-- a2 + 2b2 + 1 + b 2 + 2c 2 + 1 + c 2 + 2a 2 + 1
4a 4 4b 4 4c 4
= + + ----.,-------=-
a 4 + 2a 2b 2 + a 2 b 4 + 2b 2c 2 + b 2 c 4 + 2c 2a 2 + c 2
4(a 2 + b 2 + C 2 )2
~ ~---------~---~-----~-----=-
a 4 + b 4 + c 4 + 2(a 2b 2 + b 2c 2 + c 2a 2 ) + a 2 + b 2 + c 2
4(a 2 + b 2 + C 2)2
~ ~~-~--~~--~.,-----~-~
(a 2 + b 2 + C 2 )2 + a 2 + b 2 + c 2
= 3.

M~t khac, ta cling co (da chung minh trong lbi giiii (j tren)

a +b +c ~ 3.

Dodo

178 Vo Qu6c Ba Cful- Nguyen M~nh Linh


1. cAcHE THI TUYEN SINHCHINH THUC

Blli 4 (3.0 dilm). Cho tam giac nhQn ABC co A B < A C va n9i ti~p dl1dng
tron (0). Cac dl1dng cao B B', C C' dt nhau t'ili di€m H. GQi M la trung
di€m clla Be. Tia M H c~t dl1dng tron ( 0) t'ili di€m P.

a) Chung minh r~ng hai tam giac B PC' va C P B' d6ng d'ilng.
b) Cac dl1dng phan giac Clla cac goc B PC', C P B' lftn 111<;1t dt A B, A C
.t'ili cac di€m E va F. GQi 0' la tam dl1dng tron ngo'ili ti~p tam giac
A E F, K la giao di€m clla H M va A 0' .
-
i) Chung minh r~ng tu giac P E K F n9i ti~p.
ii) Chung minh r~ng cac ti~p tuy~n t'ili E va F clla dl1dng tron ( 0 ')
dt nhau t'ili m9t di€m n~m tren dl1dng tron ( 0 ) .

Lbi giai. a) GQi L lit di€m d6i xung clla A qua O. Khi do, ta co
LAB' B = LACL = 90°
nen B H II C L . Tl1dng tt;l, ta cling co C H II B L nen tu giac B H C L la hinh binh
hanh, suy ra H L di qua trung di€m clla B C. Tli do, ta co b6n di€m P, H, M, L
th~ng hang va LAP H = 90°.
····················..........
A~.:::::::...
•• ~~s

//
,.-:::...
•••••••
......
, ......,
•••••

,I',l .... '''',


"" l
Il
.
'
\.
..
. ..
.'.
I. ..
II
II
,I
.11
.
.
.
. ."
I ". .
"

.
P f\'~~~-t~\r~~~~!! \\
---- -- : F
.:
/
.I
:
.
j
I
:

/
,..'
~\~------~=-~~~~----~~
...... C
"
'"
" ".

........................................ L

Do LAP H = LAC' H = LAB' H = 90° nennamdi€m A, P, C', H, B'


cling thu9c dl1dng tron dl1dng kinh A H. Suy ra LAB' P = LAC' P , do do
LPC' B = LPB'C.

Vo QU6c Ba Cifn - Nguy€n M'ilnh Linh 179


CHVdNG 3, LOI GIAI cAc HE .rOAN ON TAP

L~i c6 LP Be' = L PCB' nen b, P Be' ~ b, PCB' (g-g),


.b- i) Do ~ P Be' r-v b, PCB' va E, F la chan cac dttong phan giac tttdng ung
nen L PEe' = L P F B', suy ra tu giac APE F nQi ti€p, tuc P thu9c ( 0') ,
: Do P E Ul phan giac cua goc B PC' nen
EB PB
(1)
EC' PC,'
. Do tu giac A PC' H n9i ti€p nen
LPC' H == 180 0 - LPAH == 180 0 - LPAB - LHAB
== 180 0 - LPLB - LHCB = 180 0 - LPLB - LCBL
== LBML = LCMH.
L~i co L PH C' = L CH M nen b, PC' H r-v b, C MH (g-g), suy ra
HC' HM ML
PC' MC MB
M~t khac, do tu giac P B L C n9i ti€p nen b, PM B r-v b, C M L (g-g), suy ra
ML CL HB
MB PB PB
K€t h9P v(ji k€t qua (j tren, ta dtt9C
HB HC'
PB pc"
hay
HB' PB
- (2)
HC' PC,'
Til' (1) va (2), tasuy ra ffg, = :g,nen HE la phan giac cuagoc BH C',
Chung minh tttdng tt;i, ta cling co H F la phan giac cua goc C H B ' , rna L B H C '
va L C H B' la hai goc d6i dinh nen E, H, F th~rtg hang;
Ngoai ra, do ~ P B C' r-v b, PCB' va b"H C' B r-v 6 H CO' nen ta cling co
PE Be' HE
PF CB' HF
Do do P H la phan giaccua goc E P F, suy ra PH di qua digrn chinh giua cua
cung E F khong chua A cl\a dttong tron ( 0 ') ,
L~i co tam giac A E F can t~i A nen A 0'Ia phan giac cua goc E A F, suy ra A 0 '
di qua digrn chinh giUa cua cung E F khong chua A cua dudng tron (0'), 'lU'do,
giao digrn K cuaA 0' va PH thu9c duong tron ngo~i ti~p tam giac A E F, Suy ra
tu giac P E K F n9i ti€p,
180 Yo Qu6c BuCffn - Nguy~n M~nh Linh
1. ' cAe HE THI TUYEN SINH CHINH nrGc

/;/::.," ~,.::::::::::::::::~.:::..:.:.•..............

··..
" ,t' \
..
.../ll '\.
p~
'.
'. ..j F
::\\ C
.:. .. I ,--'
~".
. :'
\ '
~ \~

\~ .~--~~----~~~------~~

b-ii) GQi D 13. di~m ehinh giUa eua eung B C khong chua A eua driong tron (0),
Q Ul di~m d6i xung eua D qua B C. Khi do, ta co
LBQC = LBDC = 180 LBAC 0
-

= 180 LB' HC' = 180 LBHC


0
-
0
-

nen tu ghic B H Q C n9iti~p, suyraL H B.Q = L H CQ ya

LQHC = LQBC = LDBC = .!.LBAC


22.
='.!.LB'HC.

Do do H Q la phan ghic eua goe B' H C. Suy ra E, H, Q, F th&ng hang.


Ta co D Q la phan giae eua goe B D C nen
0
L.QDB = 180 - LBAC = L.AEF.
2
Suy ra tu giae B D QE n9i ti~p. Do do
0

LBED = LBQD = 180 - L.A = 90 0


- .!.L.A = 90 0
- LAEO'.
2 2
TU do, ta co 0' E .1 ED, tue ED la ti~p tuy~n tl;li E eua ( 0') .
Chung minh triong tv, ta eung co F D la ti~p tuy~n tl;li F eua (0'). Do do, cae
ti~p tuy~n eua ( 0 ') tl;li E va F e~t nhau tl;li di~m D n~m tren (0). 0
Vo QU6e Ba C~n - Nguy~n M~nh Linh 181
CHudNG 3. UJI GIAI cAc HE ToAN ON TAP

Bili 5 (1.0 diim). Cho 2017 s6 hUu ty du'dng du'Qc vi~t tn3n rn9t du'ong tron.
Chang minh r~ng t6n t~i hai s6 du'Qc vi~t c~nh nhau tren du'ong tron sao cho
khi b6 hai s6 do thi 2015 s6 can l~i khong th€ chi a thanh hai nhorn rna t6ng
cac s6 a rn6i nhorn b~ng nhau.

Liti giai. Gia sa ta khong thg tim du'Qc hai s6 nhu' th~. Khong rn~t tinh t6ng quat ta
cho la cae s6 d€u la s6 nguyen du'dng va khong cling chlin. G9i S la t6ng cua t~t ca.
cac s6 trong d€ bai. Xet hai tru'ong hQp:

• Tn/dng hc;1p 1: S lit s6 chdn. Ta co cac s6 khong th€ cling la s6 Ie, suy
ra eo it nh~t ffi9t s6 chlin, do do co hai s6 qnh nhau khac tinh chlin Ie, b6
di hai s6 nay ta th~y cac s6 can I~i khong th€ chia thanh hai nhorn rna t6ng
cae s6 a hai nhorn b~ng nhau .

• Trt/dng hc;1p 2: S lit s6 te. T6n t~i hai s6 qnh nhau co cling tinh chlin Ie (vi
n€u khong chung se phai du'Qc vi~t chlin-Ie luan phien, khong th€ xay ra di€u
nay vi 2017 Ia s6Ie), b6 di hai s6 nay ta th~y cac s6 can I~i khong th€ chia
thanh hai nhorn rna t6ng cac s6 a hai nhorn b~ng nhau.

V~y luon t6n t~ 2 s6 du'Qc vi~t c~nh nhau tren du'ong tron Mkhi b6 2 s6 do thi 20 1 5
s6 can I~i khong th€ chia thanh 2 nhorn rna t6ng cac s6 arn6i nhorn b~ng nhau. 0

1.15. D@ thi tuy@n sinh THPT chuyen, 56' GD-OT Ha NQi,


2016 (d~mh cho chuyen Toan-Tin)

Bili 1 (2.0 diim).

a) Giai phu'dng trinh:

(2x - 1)2 - 9 = 4.J x 2 - x.

b) Giai h~ phu'dng trinh:

X2 - y2 - 2x + 4y = 3,

1 x 2 +y2=5.

La. giai. a) Di€u ki~n: x ~ 1 ho~c x ~ O. Phu'dng trinh dfi cho tu'dng duong vdi

x2 - X - 2 = .J x 2 - x.
182 Vo QU6c Ba Cffn - Nguyt3n M,~mh Linh
1. cAe DE THI TUYEN SINH eHiNH THue

D~t t = y' X 2 - X (t ~ 0). Phuong trlnh tren co th6 dUQc viSt l<;ti thanh
t2-2=t,
hay
(t + l)(t - 2) = O.
Suy ra t = 2 (do t ~ 0). Tu day, ta co
x 2 -x=4.
Gild phuong trlnh b~c hai §:n x , ta dUQc x = 1± f!1. V~y phuong trinh da cho co
hai nghi~m la 1- [D va 1 + f!1.
b) H~ phuong trlnh da cho tuong duong voi

(x - 1)2 - (y - 2)2 = 0,

1x 2 + y2 = 5.
Tu phuong trlnh tha nh~t, ta co x = y - 1 ho~c x =3- y.
• Voi x = Y - 1 , thay vao phuong trlnh tha hai, ta dUQc y 2 - Y - 2 = o. Tu
do, ta co y = -1 (tuong ang, x = -2) ho~c Y = 2 (tuong ang, x = 1).

• Voi x = 3 - y, thay vao phuong trlnh tha hai, ta dUQc y 2 - 3 Y + 2 = O.


Tu do suy ra y = 1 (tuong ang, x = 2) ho~c Y = 2 (Wong ang, x = 1).
V~y h~ phuong trinh co ba nghi~m (x, y) la (1, 2), (2, 1) va ( - 2, - 1 ) . 0

Bai 2 (2.0 diim).

a) Cho cac s6 thvc a, b, e co t6ng khac 0 thoa man a 3 + b 3 + e 3 = 3 abc.


Tfnh gia tri cua bi6u thac

b) Tim tM ea cac c~p s6 nguyen (x, y) thoa man

x 2 + 2y2 + 3xy - 2x - 4y - 5 = o.

La. giai. a) Ta eo
a
3
+ b3 + e3 _ 3abe = (a + b + e)(a 2 + b 2 + e 2 - ab - be - ea).
Do a + b + e i= 0 nen tu gia thiSt, ta suy ra
a2 + b2 + e2 - ab - be - ea = 0,

Vo QU6c Ba Cffn - Nguy€n M<;tnh Linh 183


CHudNG 3. LOI GIAI cAc nE ToAN ON TAp
hay
(a _b)2 + (b - C)2 + (c _a)2 = 0.
TIt do, ta co a= b = c. Thay vao bi€u thac P, ta dl1Qc P = ~.
b) D€ Yr~ng x 2 + 2 y 2 + 3 x y = (x + y)( x + 2 y ), do do phl1dng trlnh dll
cho co th€ dUQc vi€t l~i thanh

(x + y)(x + 2y) - 2(x + 2y) - 5 = 0,

hay
(x + 2y)(x + Y - 2) = 5.
Do x + 2 y, x +y - 2 E Z nen co ca.c trl1dng hQp sau xiiy ra:

• TH 1: x + 2y = -5, x +y - 2 = -1. = 7, Y = -6.


Giiii h~ dl1Qc x

• TH 2: x + 2y = -1, x + y - 2 = -5. Giiii h~ dl1Qc x = -5, Y = 2.


• TH 3: x + 2y = 1, x + y - 2 = 5. GiM h~ dl1Qc x = 13, y = -6.
• TH 4: x + 2y ...:. 5, x +y - 2 = 1. GiM h~ dl1Qc x = 1, Y = 2.

V~ycacc~ps6(x, y)theamanyeucfiula(7, -6),(-5,2),(13, -6) va (1, 2). D

Bai 3 (2.0 diim).

a) Tim tfit cii cac c~p s6 nguyen dl1dng (m, n) sao cho 2m - 1 chia h€t cho
n va 2n - 1 chia h€t cho m.

b) Cho cac s6 dl1dng a, b, c thea man a + b + c = 1. Chang minh r~ng

- abc
-+
a + b2 b + c 2
+ C + a 2 ~-4 -+-+-
abc
. 1(1 1 1)
Ldi giai. a) Khong mfit dnh t6ng quat, ta co th€ giii sa m ;;:: n. Khi do, ta co
.°< 2n - 1 < 2n ~ 2m, rna 2n - 1 chia h€t cho m nen

m = 2n-1.

Do do 2m - 1 = 4n - 3. Ma 2m - 1 chia h€t cho n nen 3 chia h€t cho n, suy ra


n = 1 (tl1dng ling, m = 1) ho~c n = 3 (tl1dng ang, m = 5).
V~y cac c~p s6 (m, n) thea man yeu cfiu la (1, 1), (5, 3) va (3, 5).

184 Vo Qu6c Ba Cffn -NguySn M~nh Linh


1. cAc DE THI TUYEN SINH CHINH TRUC

b) Sit d1.).ng cac bfit d~ng thdc AM-GM va Cauchy-Schwarz dl;mg c(mg rnftu, ta co
a a a
a + b2 a(a + b + e) + b 2 a 2 + b2 + ab + ae
a
~ 3ab + ae = 3b + e ~
1 1(3b +;;1) .
16

Danh gia tuong tl!, ta cling co

b
b
+ e2 ~ 16
1(3 + -;;1) ,
;;
_e_
e + a2
~
-...::::
_1
16
(~+~)
a b .

C<;'mg cac bfit d~ng thdc tren l~i thee v€, ta thu dUQc k€t qua dn chdng rninh. Dftu
d~ng thdc xay ra khi va chi khi a = b = e = ~. 0

Nh~n xet. Ngoai each da trlnh bay d tren, ya) con c6 th€ giai b~ng phuong phap
ch~nki€ukhac nhu sau: Tu
gia thi€t, tasuyra (2m-l)(2n-l) = 4mn-2m-2n+ 1
chia h€t cho mn. Do d6 2m + 2n - 1 chia h€t cho mn, rna 2m + 2n - 1 > 0 nen

2m + 2n - 1 ~ mn.

Kh6ng rnftt tinh t6ng quat, gia sit m ~ n. Khi d6, ta c6

mn ~ 2m + 2n - 1 < 2m + 2n ~ 4m,

suy ra n < 4. Ma n la s6 nguyen duong Ie nen n = 1 ho~c n = 3.


• Vesi n = 1, ta c6 2n - 1 = 1 chia h€t cho m nen m = 1.

• Vesi n = 3, ta c6 2n - 1 = 5 chia h€t cho m va m ~ n = 3 nen m = 5.

Tu day, ta c6 th€ di d€n k€t lu~n cho bai toano


\Yb) cling c6 th€ dUQc giiii b~ng cach khac nhu sau: Bfit d~ng thdc dn chdng rninh
: co th€ dUQc vi€t l<;ti thanh

~ (~ + ~ + ~) + ( 1 - a : b2) + (1- b : e2) + (1- e : a2 ) ~ 3,


'hay
b2 e2 a2
;

ab + be + ea
------:--- + + + ~ 3.
4abe a + b2 b + e2 e + a 2
'Sit d1.).ng bfit d~ng thdc Cauchy-Schwarz d~ng c(>ng rnftu, ta c6

VO QUQcBa Cftn -Nguy~n M~nhLinh 185


CHtJdNG 3. LOIGIAICA.C DE TOA.N ON T~P

Sa dl;lng danh quen thu('>c 3abe(a + b + e) ~ (ab + be + ea? (co th€ chang minh
b~ng bi€n d6i tudng dUdng, ta cling co

ab + be + ea
-----~
3
.
4abe 4(ab + be + ea)
Nhl1 th€, ta chi dn chang minh
3 1
. + ~ 3. (1)
4(ab + be + ea) a2 + b2 + e2
+1
D~n day, sa dl.mg bfit d~ng thac quen thu9C 3(ab + be + ea) ~ (a + b + e)2 (co
th€ chang minh b~ng bi~n d6i tl1dng dUdng), ta co
1 3 3
------~ =-.
2(ab+be+ea) 2(a+b+e)2 2

Ngoai ra, sa dl,mg bfit d~ng thac Cauchy-Schwarz d,~mg c9ng m§:u, ta co
1 1
- - - - - - +a-
4(ab + be + ea)
-b-
2+ 2 +-e 2-+-1
4
~ --------------
(a + b + e)2 + 2(ab + be + ea) + 1
4
~-----~------
(a + b + e)2 + ~(a + b + e)2 + 2
3
-
2
C9ng cac bfit d~ng thac tren l<;ti thea v~, ta thu dUQc bfit d~ng thuc (1). Tit do, ta
co di€u phili chang minh.

Blii 4 (3.0 diim). Cho dUdng tron (0) va di€m M n~m ngoai dl1dng tron.
Tli M ke
cac ti~p tuy~n M A, M B v(ji dUdng tron (A, B Ia cac ti~p di~m).
Duong thttng qua M c~t dUdng tron (0) t<;ti C va D (M C < M D) sao
cho di€m 0 n~m trong tam giac BCD. GQi E la di€m d6i xung cua C qua
O. GQi S la giao di€m cua E A va Be.

a) Chang minh r~ng hai tam giac 0 A C va MAS d6ng d<;tng.


b) Dl1dng th~ng S D dt dl1dng tron ( 0) t<;ti di€m tha hai K. Chung minh
r~ng tam giac B K C din.

e) GQi N la giao di€m cua M 0 va A E . Chang minh r~ng N D 1- D A .

186 Vo Qu6c Ba Cffn - Nguy~n M~nh Linh


"* A A ",. "*
I. CA{: DE THI TUYEN SINH-CHINH THOC

LUi liar. a }Xet cae tam giac A 0 M va A C S , ta co

L.AOM = ~LAOB = LADB = LACS


2
va
LOAM = LCAS = 90
0

nen 6AOM rv 6AC S (g-g). Suy fa


OA AM
AC AS
L~i co L OAC = 90 0
- LCAM = LSAM nen 6 OAC rv 6M AS (c-g-c) .
.......--..
............•.........................
/
~
~ ~
,
~

;./.../...... ......... S
.- '\...
j// .•..•.•....•••••••••••••••••. ~ \\. .
.
:
.'
~.~.

DV ,

j
!

.:
/M
",.:
.........
.'
. '

...... ...•.........
.................. ......
······B
.....................................'
h) Do tam giac 0 A C din t~i 0 va tam giac 0 A C d6ng d'ilng vdi tam ghic
MAS nen tam giac MAS can t'ili M. Suy fa M S = M A = M B , tu do ta co
tam giac M B S din t'ili M. D§:n d~n
LMSB = LMBS = LMDB.
Do do, tu giac M B D S nQi ti~p. Tu day, ta suy ra

LKDC = LSDM = LSBM = LBDM = L.BDC,

VO QU6c Ba Can - Nguyen M~h Linh 181


CHUdNG J. LOI GIAI cAc DE ToAN ON TAP

tuc C Hl digm chinh gifia cua cung B K cua dl1dng tron (0). Va nhl1 th€, ta co
tam ghic BKC din tl;li C.
c) GQi H lit giao diem cua A B va 0 M. Ta co

LBNM = LANM = LSNM = 90° - LNAH


= 90° - LEAB = LBAC = LBEC
= LSBM.
Do do, tu ghic B M S N nQi ti€p, rna tu ghic M S D B nQi ti€p nen N n~m tren
dl1dng tron ngol;li ti€p tu giac B M S D .
Xet cac tam giac DEN vit DCA, ta co L DEN = L D E A = L DCA va
LDNE = 180° - LDNS = 180° - LDBS = LDAC
nen LNDE = LADC. Suyra

LNDA = LADC + LCDN = LNDE + LCDN = LEDC = 90°.

Tit do ta co N D ..1 D A . Day chinh la k€t qua c~n chung minh. o


Bili 5 (1.0 die'm). Cho 101 s6 nguyen dl1dng co t6ng b~ng 300 dl1<;1e vi€t
tren mQt dl1dng tron. Chung minh r~ng luon t6n tl;li mQt day cae s6 vi€t li€n
nhau co t6ng b~ng 100.

Uti giai. GQi cac s6 nguyen dl1dng theo thu ttj tren dl1dng tron la a 1, a 2, ... ,
a 101. Xet cae t6ng .

. . .,
ta thfiy eo hai trl1dng hQp sau co the xay ra:

• Tnldng hgp 1: Trong cac s6 S 1, S 2, . . . , S 1 00, t6n t(Ii m9t s6 chia hh


cho 1 00 . Gia sit lit S k (k EN, 1 :( k :( 1 00) , khi do do 0 < S k < 300
nen ta co Sk = 100 ho~c Sk = 200.

N€USk = 100thicacs6a1, a2, ... , akthoamand~ubai.

N€USk = 200thiak+1 +ak+2+ .. ·+a101 = 100nencacs6 a k+1,


ak+2, ... , a 101 thoa man d~u bai.

188 Vo Qu6c Ba Cffn - Nguyen Mzmh Linh


1. cAc HE THI TUYEN SINH CHfNH THUC

• TntCing hQ'p 2: Trang cdc s6 S 1, S 2, . . . , S 1 00, kh6ng t6n tfli s6 new


chia hit cho 100. Theo nguyen 19 Dirichlet, t6n t~i hai s6 co cung s6 du khi
chiacho 100. Giasa Ut Sk va Sm (k > m), suyra Sk - Sm chiah8tcho
100. Ma 0 < S k - S m < S k < 300 nen S k - S m E {I 00, 200}.

N8u Sk - Sm = 100 thl a m+l + a m+2 + ... + ak = 100 nencac s6


am + 1, am +2, . . . , a k thoa man d~u bai.

N8u Sk- Sm = 200 thl a m+l + a m+2 + ... + ak = 200, suy ra


a k + 1 + a k +2 + ... + a 101 + a 1 + a 2 + ... + am -1 = 100. Do do cac
s6 a k + 1, a k + 2, . . . , a 101, aI, a 2, . . . , am -1 thoa man d~u bai. 0

... ?

1.16. De thi tuyen sinh THPT chuyen OHSP Ha NQi, 2017

Bai 1 (1. 5 c1iim). Cho cac s6 du'dng a, b, c, d. Chung minh r~ng trong 4 s6

1 1
2
a +-b
1 1
+-,
·c
c
2
+ -d1 +-,
1
a
2
+ -+-
d..... a b

co it nhfit mQt s6 khong nho hdn 3 .

.La. giai. Ta thfiy t6ng cua b6n s6 tren b~ng

Sit d\lng bfit d&ng thuc AM-GM, ta co

x2 + ~ + ~ ~ 3 3/ x 2 • ~. ~ = 3.
x x Y x x
Tu do suy ra S ~ 1 2. Do do, s6 IOn nhfit trong b6n s6 phai khong nho hdn 3. Dfiu
biing xay ra khi va chi khi ca b6n s6 a, b, c, d d6u b~ng 1. 0

Bai 2 (1.5 diim). Giai phu'dng trlnh:

..j(x 2 + 2X)2 + 4(x + 1)2-vx2 + (x + 1)2 + (x 2 + x)2 = 2017.

Vo Qu6c Ba Cffn - Nguy~n M~nh Linh 189


CHl1dNG 3. LCn GIAI cAc DE ToAN ON T!P

Lbi giai. Ta co

(X2 + 2X)2 + 4(x + 1)2 = [eX + 1)2 _1]2 + 4(x + 1)2


=[(x+1)2+1]2

ya

x 2 + (X + 1)2 + (X 2 + X)2 = 2(x 2 + X) + 1 + (X 2 + X)2


= (X 2 + X + 1)2.
Do do, phudng trinh dii cho tuong duong ydi (chu y r~ng X2 + X + 1 > 0)

(X + 1) 2 + 1 - (x 2 + X + 1) = 20 1 7,

hay
x=2016.
V~y phuong trinh da cho co nghi~m duy nh~t X = 201 6. o
Bili 3 (3.0 diem).
a) Tim t~t ca. cac sf, nguyen duong a, b, c, d thoa man

b) Tim tftt eft cac sf, thvc X sao cho trong 4 sf,

1 1
X - -,
X
X +-X
co dung mQt sf, khong phiii la sf, nguyen.

Lm giai. a) Ta co a 2 = b3 nen a = b.Jb, suy ra .J7J Ut sf, huu ti. Ttl do, thea
b6 dE! lloi giiii diu 2b) cua d~ tuyen sinh THPT chuyen KHTN nam 2015 -
yong 2 (b~n dQc yui long xem l~i phftn trudc), ta co b 1ft sf, chinh phuong. D~t
b = x 2 ydi x E N * thl ta co a = X 3 .
Tuong tv, ta cfing co c .Jc = d2 nen .Jc Ut sf, hOO ti, suy ra c la sf, chinh phuong,
tile ta co c = Z 2 ydi ZEN * . TU do suy ra d 2 = Z 3 , hay d = z.Jz. Ta eo ,Jz
la sf, hi1u ti nen Z la sf, chinh phuong. D~t Z = Y 2 ydi yEN * thi ta co d = Y 3 •
190 Vo Quf,c Ba O{n - Nguyen M~nh Linh
1. cAc DE THI TUYEN SINH CHINH THUC

Bay gia, ta vi~t I'.li gia thi~t thu ba cua bai tmln duoi d~;mg

hay
(x - y)(x 2 + xy + y2) = 98.
Til day suy ra x - y > o. De y r~ng x 2 + XY + Y 2 >
(x - y) 2 nen ta co
(x - y) 3 < 98, suy ra x - y :::;; 4. Cac s6 nguyen duong khong vU<;lt qua 4 va
Ia uoc cua 98 chi co 1 va 2. Nhu v~y, co hai twang h<;lp xay ra nhu sau:

• TntCing hQ'p 1: x - y = 1, x 2 + xy + y2 = 98. DB thfty h~ nay khong co


nghi~m nguyen duong (chu y r~ng x, y khac tinh ch~n Ie) .

• TntCing hQ'p 2: x - y =
2, x 2 + xy + y2 =
49. Giai h~ phuong trinh
nay voi chu y x, Y E N*, ta dU<;lc x = 5 va y = 3.

Til k~t qua cua twang h<;lp thu hai, ta suy ra a = 125, b = 25, C = 81 va d = 27.
b) Dieu ki~n: x =/:. O. Gia sit ell hai s6 x - ~ va x + ~ deu nguyen, khi do ta co

1 1
x--+x+- =2x
x x
Ia s6 nguyen. Do do, x Ia s6 huu d. Til day, ta suy ra x - ,J'i va x 2 + 2.J2Ia hai s6
vo d, mau thu§:n voi yeu du cua de bai.
Nhu v~y, trong hai s6 x - ~ va x + ~ phai co mQt s6 khong nguyen. Til do, do
yeu c~u cua bai toan nen ell hai s6 x - ,J2 va x 2 + 2,J'i deu Ia s6 nguyen. Di;it
x - .J2 = a voi a E Z, khi do ta co

2
x + 2,J'i = (a + v'2)2 + 2.J2 = a 2 + 2 + 2(a + 1).J2.
N~eu a -/.. 2(a+ 1) -2 Ia, so~ h-tiu tl, mau th;::
2 2
"/ - 1 th'1 ta co'M2
'V L. = (x +2.J2)-a ? A
uan. D 0 d'0 a = - 1,
tuc x = .J2 - 1. Thit I'.li, ta thfty voi x = ,J2 - 1 thi

1 1
x-.J2=-1 , x- - = -2, x +- = 2.J2.
x x
Chi co dung mQt s6 khong nguyen nen gia tri nay thoa man yeu c~u de bai. 0

Vo Qu6c Ba Cffn - NguyBn M'.lnh Linh 191


CHVdNG 3. LOI GIAI cAe HE ToAN ON TAp

Bai 4 (3.0 c1iim). Cho duong tron (0), han kinh R va di€m M n~m phia
ngoai dUClng tron (0). Ke cac ti€p tuy€n M A, M B t(ji duong tron (0)
(A, B Iii cac ti€p di€m). Tren dOC;ln th~ng A B lay di€m C (C khac A, C
khac B). GQi I, K l~n IUQt Ia trung di€m cua M A, M C. Duong th~ng
K A c£t dUClng tron ( 0) tC;li di€m tha hai D.

a) Chang minh r~ng K 0 2 - KM 2 = R 2.


b) Chang minh r~ng ta giac BCD M Iii ta giac nQi ti€p.
c) GQi E Iii giao di€m tha hai cua duong th~ng M D v(ji duong tron ( 0) va
N Iii trung di€m cua K E. Duong th~ng K E c~t duong tron (0) tC;li di€m
tha hai F. Chang minh rkg I, A, N, F cung n~ tren mQt dUClng tron.

LCii giili. a) D8 thay AB ..1 OM. Ta co 1 KIa duong trung hlnh ang vdi cC;lnh
AC cua tam giac AM C nen 1 K II AC, tu do suy ra 1 K ..1 OM.
GQi J la giao di€m cua 1 K va 0 M. Sit dl;lng dinh 19 Pythagoras, ta co

K0 2 - KM2 = (OJ 2 + JK2) - (JM 2 + IK2) = OJ 2 - JM2


= (OJ 2 + 1 J2) - (M J2 + 1 J2) = 0/ 2 - M /2
= OA2+AI2_MI2 = OA 2
= R2.
A" ........................................................ ..
"
..............'"
'.. E
..
\\
M ~~~::==~~~~~~=t~;;~()---------/-!1) H
/
/

.../ /
, ......

B·········· ...........···········

192 Vo Quac Ba C§:n - Nguy~n M~mh Linh


1. cAc HE THI TUYEN SINH CHfNH THUC

b) GQi G va H lfin h1Qt la giao di€m cua tia K 0 v(ji (0). Khi do, ta d~ thfiy
ttl' giac A D G H nQi ti€p nen

KA· KD =KH· KG = (KO + R)(KO - R)


=K02_R2=KM2.

Tti day, ta co K M la ti€p tuy€n cua dUdng tron ngo~i ti€p tam giac ADM. Suy ra

LDMC = LDAM = LDBC,

do do ttl' giac BCD M nQi ti€p.


c) Theo cau b), ta co
LDMK = LDAM = LAED.

Ma hai goc nay d vi tri so Ie trong nen K M II A E . Tti day, ta suy ra AM K E la


hinh thang va I N la dUdng trung binh cua hinh thang nay. Do do,

LI NF = LAEN = LI AF.

Di€u nay chung to tu giac AN F I nQi ti€p. Ta co di€u phai chung minh. 0

Bai 5 (1.0 diim). Xet hinh ve sau:

B C
D

Ta vi€t cac s6 1, 2, 3, . . . , 9 vao vi tri cua 9 di€m trong hinh ve tren sao
cho m6i s6 chi xufit hi~n dung mQt Ifrn va t6ng ba s6 tren m6i qnh cua tam
giac b~ng 18. Hai each vi€t dUQc gQi la nhu nhau n€u bQ s6 vi€t a cac di€m
(A, B, C, D, E, F, G, H) cua m6i cach lit trimg nhau. Hoi co bao nhieu
cach vi€t phan bi~t? T~i sao?

Vo Qu6c Ba Can ~ Nguy~n M~nh Linh 193


CHudNG 3. LdI GIAI cAc HE ToAN ON TAP

18~9+8+1=9+7+2=9+6+3=9+5+4
~ 8 + 7 + 3 = 8 + 6 + 4 = 7 + 6 + 5.

Day la tAt ca eae eaeh viBt 1 8 thanh t6ng eua ba s6 phan bi~t thuQc { 1, 2, . . . , 9}.
Trong tAt ca eae vi tri A, B, C, D, E, F, G, H, K:

• A, B, C xuAt hi~n trong hai t6ng;

(9 D, E, F xuAt hi~n trong ba t6ng;

• G, H, K xuAt hi~n trong mQt t6ng.

Do do 1, 2 ~ {A, B, C, D, E, F} (1, 2 ehlxuAthi~n trong mQtt6ng). Co 6 cach


di~n 1, 2 vao cac vi tri G, H, K. Khong giiim t6ng quat, gia sli G = 1, H = 2.

5 6
7

Do 9 xuAt hi~n trong cae t6ng chua 1, 2 nen d6 thAy F = 9, suy fa E = 8,


D = 7. Til do, ta co K = 3. KBt qua nay cho thAy {A, B, C} = {4, 5, 6}.
L~ co A + F + B = 18, B + D + C = 18, C + E + A = 18 nen A = 4,
B = 5, C = 6. Nhu v~y, ung vdi m6i each di~n hai s6 1, 2, ta co duy nhftt mQt
cach di~n eho cae s6 con l~. Tom l~, eo 6 caeh di~n thoa man yeu cAu d~ bai. D

194 Vo Qu6e Ba Cftn - Nguy6n M~nh Linh


1. cAc HE THI TUYEN SINH CHINH THUC

1.17. D~ thi tuy@n sinh THPT chuyen KHTN, 2017 (vong 1)

Bai 1 (3.5 c1ie'm).

a) Gild h~ phl1dng trinh:

~ x2 +
y2 - xy = 1,
lx+x 2 Y=2 y 3.

b) Gild phl1dng trinh:

2(x + 1).Jx +1= (.Jx + 1+ .Jl - x) (2 - .Jl - X2) .

lO'i gifJi. a) Til' h~ phl1dng trinh da cho, ta co

x(x 2 + y2 _ xy) + x 2y = 2 y 3,

hay
(x - Y)(X2 + xy + 2y2) = o.
Tit day, ta suy fax = y (chu y f~ng x 2 + X Y + 2 y 2 = (x + f) 2 + t y 2 nen
n~u x 2 + X Y + 2 Y 2 = 0 thl x = Y = 0). Thay vao phl1dng trinh thli nhfit eua
h~, ta dl1Qc x 2 = 1. Suy fax = 1 (tl1dng ling, y = 1) ho~c x = -1 (tl1ong
ling, y = - 1). Cac c~p s6 nay d~u thoa man.
V~y h~ phl1dng trlnh da cho co hai nghi~m (x, y) la (1, 1) va ( - 1, - 1) .
b) Di~u ki~n: - 1 :::; x :::; 1. D~t a = .J 1 + x va b =.J 1 - x (a, b ~ 0)
thl ta co a 2 + b 2 = 2. Phl1dng trinh da cho co th€ dl1Qc vi8t l~i thanh

2a 3 = (a + b)(2 - ab),

hay
2a 3 = (a+ b)(a 2 + b 2 - ab).
Tit day, ta co 2 a 3 = a3 + b 3 , hay a = b. MQt cach tl1dng dl1dng, ta co
.Jl+x=.Jl-x,

hay x = o. V~y phl1dng trinh da cho co nghi~m duy nhfit x = o. 0

Vo QU6c Ba Can - NguySn M.,mh Linh 195


CHDdNG 3. LOI GIAI cAc HE ToAN ON TAp

Bai 2 (2.5 diim).


a) Chang minh r~ng kh6ng t6n t~i cac s6 nguyen x, y thoa man d~ng thuc

12x2 + 26xy + 15y2 = 4617.

b) Vdi a, b 1ft cac s6 thl;iC du'dng, tim gia tri ldn nhAt cua bieu thuc

Uti giai. a) Ghl sil' t6n t~i c~p s6 nguyen (x, y) thoa man phu'dng trinh
12x2 + 26xy + 15y2 = 4617,
hay
(x + 2y)2 + + y)2 = 4617.
II(x
Ta thAy 46 17 chia 11 du' 8, trong khi do (x + 2 y ) 2 chia 11 du' 0, 1, 3, 4, 5, 9.
Mau thu~n nh~n du'<;jc cho ta k€t qua dn chung minh.
b) Sil' dt;mg bfrt d~ng thuc Cauchy-Schwarz, ta co

(a +b)
3
(~ +b) ~ (a+b)2.
Tll do suy ra
1 + ab
a(a+b)'
Danh gia tudng tg, ta cling co
a +b 1 + ab
---~----
3
b +a""b(a+b)
Dodo

( a+ b) ( 3 1 1) 1 + ab 1 + ab 1 1
. a +b
+ b3 +a
~
a(a+b)
+ b(a+b) =-+
ab
.
Tll day, ta suy ra
1 1
M ~ ab + 1- ab = I,
Dfru d~ng thuc xay ra khi va chi khi a = b = 1. V~y max M = 1. o
Nh~n xet. Ngoai cach da trinh bay d tren, ya) con co the giai b~ng cach khac
nhu' sau: Phudng trinh da cho co the dU<;jc vi€t l~i thanh
12x2 + 26xy + 15y2 - 4617 = O.

196 Vo Qu6c Ba Cgn - NguySn M~nh Linh


1. cAc HE THI TUYEN SINH CHINH THUC

Xem day la phuong trinh b~c hai ffn x. Di6u ki~n d.n d€ phuong trinh nay co
nghi~m nguyen la bi~t thtic /::..' phiii la s6 chinh phuong. Ta co

/::..' = 169y2 - 12(15y2 - 4617) = 55404 - lly2.

Do 55404 chia 11 du 8 nen /::..' chia 11 du 8. Ma mQt s6 chinh phuong khi chia
11 thi s6 du chi co th€ la 0, 1, 3, 4, 5, 9. Do do, /::..' khong th€ la s6 chinh
phuong. Tit day, ta co di6u phiii chting minh.

Bai 3 (3.0 die'm). Cho hinh thoi AB CD v(ji LBAD < 90°. DUdng tron tam
I nQi ti~p tam giac A B D ti~p xuc v(ji B D, B A l~n IUQt tl;li J, L. Tren dUdng
th~ng L J, lAy di€m K sao cho B K song song vdi I D. Chung minh r~ng

a) LCBK = LABI.
b) KC ..1 KB.
c) B6n di€m C, K, I, L cimg n~m tren mQt dUdng tron.

lCii giai. a) Do B C II AD, BK II I D va Alia trung tn;tc cua BD nen


LCBK = LIDA = LIBA.
b) Tit cau a) v(ji chu y tu giac B J I L nQi ti~p, ta co
LCBK = LIBA = LIJL = LCJK.
Do do, tti giac Be K J nQi ti~p. Suy ra L C K B = L C J B = 90°.
B

·

.
A ~----~----~--~~--~----------~./C
......../ .
..........................

c) Theo chting minh d cau b), ta co tu giac B C K J nQi ti~p nen


LJCK = LJBK = LJBI = LJLI.
Tit do suy fa tu giac C K I L nQi ti~p. o
Vo QU6c Ba Cffn - Nguyen Ml;lnh Linh 197
CHl1dNG 3. UJI GIAI cAc DE ToAN ON TAp

Bai 4 (1.0 diim). 11m t&t ca cac hQp s6 nguyen dl1dng n saocho t6n t<;li mQt
cach s£p x€p cac s6 1, 2, . . . , n thanh aI, a 2, . . . , a n rna khi chia cac s6
aI, a 1 a 2, . . . , a 1a2 ••• an cho n , ta dl1Qc cac s6 dl1 doi mQt khac nhau.

lai giai. Ta se chung minh vdi hQp s6 n > 4, khong t6n t<;li cach s~p x€p thoa
man yeu duo Th~t v~y, ghi sit t6n t<;li mQt cach x€p nhl1 v~y. Khi do, d~ th&y an = n
(vi ngl1Qc 1~ se co it nh&t hai s6 chia h€t cho n). Suy ra

ala2'" an-l = (n - 1)1.

• N€u n co d<;lng p q vdi p, q EN, 1 < P < q thi hi~n nhien ta co


1 < P < q < n - 1 nen trong cac thua s6 cua (n - I)! co chua p va q.
Do do (n - I)! : p q = n, mau thu~n.

• Xet tntong hQp n = p 2 vdi pEN * . Khi do, do n > 4 nen p > 2. Do do

p2 _ 1 > 2p > P > 1,

suy ra trong cac thua s6 cua (n - 1) ! co chua p va 2 p. K€t qua nay chung
to (n - I)! : 2 p 2 = 2 n , mau thu~n.

Tit chung minh tren, ta suy ra n = 4. Vdi n = 4, ta co mQt cach s~p x€p cua
1, 2, 3, 4 la 1, 3, 2, 4. Khi do, cac s6 1, 1 . 3, 1 . 3 . 2, 1 . 3 . 2 . 4 co cac
s6 dl1 1, 3, 2, 0 khi chia cho 4.
V~y co duy nh&t mQt gia tri n thoa man yeu du la n = 4. D

1.18. D€ thi tuy~n sinh THPT chuyen KHTN, 2017 (vong 2)

Bai 1 (3.5 diim).

a) Giai h~ phl1dng trinh:

l x+ Y =JX+3 Y ,
X2 +y 2 +xy=3.

b) Vdi a , b Ia cac s6 dl1dng thoa man a b +a +b = 1, chung minh r~ng

a b 1 + ab
----::-+
2 2 = .
1+a 1+b J2(1 + a 2)(1 + b 2)

198 Vo Qu6c Ba Cftn - Nguy~n M<;lnh Linh


1. cAc DE THI TUYEN SINH CHfNH THUC

LO'i gia i. a) Di~u ki~n: x + 3 y ;? o. Ta phudng trinh tha nh~t, ta suy ra x +y ;?


O. Khi do, h~ phudng trinh da cho co th€ dUQc vi~t h,li thanh

(x + y)2 = X + 3y,

Ta day, ta suy ra
I (x+y)2=3+xy.

x+3y =3+xy,
hay
(x - 3)(y-1) = O.
Do do x = 3 ho~c Y = 1.

• Vdi x = 3, thay vao phudng trinh tha hai cua h~, ta dUQc y 2 + 3 y +6 = O.
Phudng trinh nay vo nghi~m.

• Vdi Y = 1, thay vao phudng trinh tha hai cua h~, ta dUQc x 2 +X - 2 = o.
Ta do, vdi chu Yx + y ;? 0, ta co x = 1.

V~y h~ phudng trinh da cho co nghi~m (x, y) duy nh~t la (1, 1).
b) Chu y r~ng, ta gia thi~t, ta co
2 = 1 + a + b + ab = (1 + a)(1 + b),
1 +a 2
= a + b + ab + a 2 = (a + b)(l + a),
1 + b2 = a + b + ab + b 2 = (a + b)(l + b).
Dodo
a b a b
1+a 2 + 1+b = (a+b)(l+a) + (a+b)(l+b)
2

a + b + 2ab
(a + b)(l + a)(l + b)
1 + ab
-2(a+b)'
M~t khac, ta cling co

1 + ab + ab
1
.j2(a + b)2(1 + a)(l + b)
1 + ab
-2(a+b)'
Ta hai k~t qua tren, ta suy ra di€u phlii chang minh. o
VO Qu6c Ba Can - Nguy~n M~nh Linh 199
CHl1dNG 3. LOI GIAI cAc DE TOAN ON TAp

Bai 2 (2.5 diim).

a) TIm tfit cit cac c~p sf:, nguyen tf:, (p, q) thoa man d~ng thac

pep - 1) = q(q2 - 1).

b) Vdi a, b, e HI. cac sf:, thgc dUdng thoa man a b + be + e a + abc = 2,


tim gia tri Wn nhfit cua bigu thac

a+l b+l e+l


M = a 2 + 2a + 2 + b 2 + 2b + 2
+
e 2 + 2e + 2
'

Loi gifli. a) Ttt gia thi6t, d€ thfiy P > q ;? 2, Ta co q (q - 1)( q + 1) chia h6t
cho p, rna 0 < q - 1 < q < p va p nguyen tf:, nen q va q - 1 kh6ng thg chia
h6t cho p, Th do, ta co q + 1 chia h6t cho p. L~i co 0 < q + 1 < 2 q < 2 P nen
q +1= p.
N6u q Iethl p = q + 1 chful va p > 2 nen p la h<;sp s6, mau thuftn. Do do
q = 2, ttt do ta co p = 3. ThU' l~i, ta thfiy thoa man.
V~y co duy nhfit mQt c~p sf:, nguyen t6 (p, q) thoa man yeu cftu la (3, 2).
b) Tntdc h6t, ta chang minh b6 d8 sau:
B6 d~ 1. Vdi m9i a, b, e > 0, ta co
8
(a + b)(b + e)(e + a) ;? "9(a + b + e)(ab + be + ea).
a
Chang minh. B6 d8 nay da du<;sc chang minh bai 3 d8 tuygn sinh THPT chuyen
Tmin cua sa GD-DT Ha NQi nam 2015. (B~n dQC vui iong xem l~i ph~n tntdC.) •
Trd l~i bai toan: D~t x = a + 1, Y = b + 1 va z = e + 1, Thay vao gilt thi6t
va rut gQn, ta co x + y + z = x y z . Bigu thac M co thg du<;sc vi6t l~i thanh
M= x + Y + Z
x2+ 1 y2 +1 Z2 +1
Dgyr~ng
x xyz x+y+z x+y+z
------
2
- - - - - - - - - ---------------
x +1 x2yZ+yz x(x+y+z)+yz (x+y)(x+z)'
Tltclng tl!, ta cling co
y x+y+z z x+y+z
y2+1-(y+z)(y+x)' z2+1 (z + x)(z + y)'

200 Vo Quf:,c Ba C~n - Nguy~n M~nh Linh


1. cAc HE THI TUYEN SINH CHfNH THUC

Dodo
x+y+z x+y+z x+y+z
M = + + ------=-----
(x+y)(x+z) (y+z)(y+x) (z+x)(z+y)
2(X+y+Z)2
(x + y)(y + z)(z + x) .

Ap d\mg b8 d~ tren, ta co

M
2(x+ y + Z)2 9(x + y + z)
~ ~--------~------------
~(x + y + z)(xy + yz + zx) 4(xy + yz + zx)'

M~t khac, su dt;mg b&t d~ng thuc 3 abc (a + b + c) ~ (a b + be + c a ) 2 (co


th€ chung minh b~ng bi~n d5i tudng dUdng), ta dUQc

+ yz + ZX)2 ~ 3xyz(x + y + z) =
(xy 3(x + y + Z)2.
Tudo, taco xy + yz + zx ~ J3(x + y + z) va

9(x+y+z) 3J3
M ~ --~-----=----~-
4J3(x+y+z) 4

D~ng thuc xay ra khi va chi khi x =Y =z = J3, hay a =b =c = J3 - 1 .


. max M = 3-.;'3
Yay 4 . D

Nh~n xet. 0 cau a), nguyen ban d~ g6c duQc phM bi€u nhu sau: Gid sit p, q lit
hai s6 nguyen t6 thoa man ddng thuc
pep - 1) = q(q2 - 1).
i) Chung minh rling t6n t(li s6 nguyen dU(}ng k sao cho
p-1=kq, q2-1=kp.

ii) TIm tdt cd cdc s6 nguyen t6 p, q thoa man ddng thuc (*).
Co th€ th&y yi) chinh la gQi ymQt huang giai cho yii). Loi giai d tren la mQt huang
di dQc l~p, kh6ng dn su dt;mg d~n kSt qua cua y i). Duai day, xin dUQc giOi thi~u
them mQt Wi giai khac cho nguyen ban cau a) nhu sau: Truoc h€t, ta se chung minh
y i). Ro rang p =I- q. Khi do, ttt gia thiSt vai chu y p, q nguyen t6, ta th&y p - 1
chia hSt cho q . D~t P - 1 = k q vai kEN * . Thay vao (*), ta dUQc
kpq = q(q2 - 1),
hay
q2 _ 1 = kp.

Vo QU6c Ba egn - Nguy~n M~nh Linh 201


CHVdNG 3. LOI GIAI cAc DE ToAN ON TAp

Bay gid, ta se giili y ii). Tu i), thay vao d~ng thac tha hai, ta dUQC
q2 _ 1 = k(1 + kq),
hay

Xem day la phuong trlnh b~c hai fin q. Di€u ki~n c~n dc3 phuong trlnh nay co
nghi~m q nguyen la bi~t thac b. phili la s6 chinh phuong. Ta co

+ 4k + 4.
b. = k4
Do kEN * nen d~ th~y (k 2 ) 2 < b. :::;; k 4 + 4 k 2 + 4 = (k 2 + 2) 2. Ma b. la
s6 chinh phudng nen b. E {(k 2 + 1)2, (k 2 + 2)2} .
• Vdi b. = (k 2 + 1)2, ta co 4k + 4 = 2k2 + 1. V~ tnii la s6 chan, trong
khi v~ phlti la s6 Ie, mau thu§:n .

• Vdib. = (k 2 +2)2,taco4k+4 = 4k 2 +4,suyrak = 1 (dok > 0).


Thay vao phudng trlnh (* * ), ta dUQc
q2 _ q _ 2 = O.

Tudo,tacoq = 2(doq > O).Dop = 1 +kqnenp = 3.


D6i vdi Y b), sau khi d~t fin phl.l x, y, z, ta co thc3 chang minh M ~ 3 f
bfu1g cach khac nhu sau: D~t x = J} m, y = J} n va z = J} p thi ta co
m, n, p > v:
va m + n + p = 3 m n p. Ta cftn chang minh
m n p 3
- --+
3m 2 + 1 3n 2 + 1
+ 3 p2 + 1 ~-
'" 4'
Sti dl.lng bftt d~ng thac AM-GM, ta co
m m m 1
-----,-------,- :::;; = ----
3m 2 +1 m 2 + 1 + 2m 2 2m + 2m 2 2(m + 1) .
Danh gia tuong tv, ta cling co
n
---~---
1 P 1
3n 2 + 1'" 2(n + 1)' 3 p 2+1:::;; 2(p+l)'
Do do, ta chi c~n chang minh
1 1 1 3
m +1+n+1+ p +1 :::;; 2'
hay
2[(m + l)(n + 1) + (n + l)(p + 1) + (p + l)(m + 1)]
:::;; 3(m + l)(n + l)(p + 1).

202 Vo Qu6c Ba C1ln - Nguy~n M<;lnh Linh


1. cAc HE THI TUYEN SINH CHINH THUC

Sau khi thu gQn, ta vi~t d119C bftt d~ng thuc tren dl10i d~ng

mn + np + pm + 3mnp ;? 3 + m + n + p,
hay
mn + np + pm ;? 3.
D~n day, sil' d\mg bftt d~ng thuc Cauchy-Schwarz d.;mg cQng mfiu, ta co

mn+np+pm=mnp
1 1 1)
(-+-+- ;?
9mnp
=3.
m n p m+n+p

Bai 3 (3.0 dim). Cho tam giac AB C nhQn voi AB < AC. E, F Ifin I119t
Ia trung di€m cua cac Climh C A, A B . Dl1dng trung trt!c cua do<;tn th~ng E F
c£t B C t~i D. Giii sil' co di€m P n~m trong goc E A F va n~m ngoai tam giac
A E F sao cho L P E C = L DE F va L P F B = L D FE. P A c£t dl1dng
tron ngo~i ti8p tam giac P E F t~i Q khac P.

a) Chungminhr~ng LEQF = LBAC + LEDF.


b) Ti~p tuy~n t~i P cua dl1dng tron ngo~i ti8p tam giac P E F dt cac dl1dng
th~ng C A, A B Ifin I119t t~i M, N. Chung minh r~ng b6n di€m C, M,
B, N cling n~m tren mQt dl1dng tron, gQi dl1dng tron nay Ht (K) .

c) Chung minh r~ng (K) ti~p xuc voi dl1dng tron ngo~i ti8p tam giac A E F .

Uti gicii. a) Do ttl giac P E QF nQi ti~p nen ta co


LEQF = 180 0 - LEP F
= LPEF + LPFE
= LDEC + LDFB
= (LEAD + LEDA) + (LFAD + LFDA)
= LBAC + LEDF.
V~y LEQF = LBAC + LEDF.
b) Khong mftt tinh t6ng quat, ta giii sil' M n~m giua A, C con N n~m tren tia d6i
tia B A nhl1 hlnh ve ben dl1oi, con cac trl1dng h9P khac Utm tl1dng tt!. Ta co
LMNB = 180 0 -LNPF-LPFN = 180 0 -LPEF-LDFE
0
= 180 0 - LDEC - LDEF = 180 - LCEF
= L.AEF = LACB.

Do do, tu giac N eM B nQi ti~p trong dl1dng tron (K).

Vo Qu6c Ba Cffn - NguySn M~nh Linh 203


CHVdNG 3. LOI GIAI cAc HE ToAN ON TAp

~~~--~~~~~----~,C
\"

N
.
I
\
K

c) Truoc h~t, ta se chung rninh

LPAB = LDAC. (1)

Th~t v~y, gQi X, Y, Z lfut hl'Qt Hl cac di6rn dbi xung cua P qua E F, A E, A F
(xern hinh ve ben dll'oi). Khi do, tit gift thi€t L PEe = L DE F, ta suy fa
LDEY = LDEX. Ma EX = EY nen DX = DY.
X

--'" Y

P
Tu'dng tt,t, ta cflllg co D X = D Z . Do do D X = D Y = D Z . M~t khac, ta l~i co
AY = AP = AZnenLDAZ = LDAY.K~thQptfnhdbixung,tasuYfa(l).

204 Vo Qu6c Ri Cfin - Nguy~n M~nh Linh


1. cAc HE THI TUYEN SINH CHfNH THUC

G<.>i R 1ft giao digm thu hai cua cac du'dng tron (P EM) va (A E F). Ta co

LRPN = LREM = LRFA,

do do tu giac P R F N n9i ti~p. L~i do ttl giac B N C M n9i ti~p nen

LABC = 180 0
- LNBC = 180 0
- LCM N = LAM N.
Til day, ta thu dl1<;lc

LARE = LAFE = LABC = LAMN = 180 0 - LPRE.

Suy ra ha digm A, R, P th~ng hang.


G<.>i I Ia giao digm cua E F va AD. Theo tinh chAt du'dng trung hinh thi I Ia trung
di~m eua AD. Do tu giac A E R F n9i ti~p nen LA E I = LA R F, k~t h<;lp voi
(1), ta suy ra l::"AE I '"" l::"ARF (g-g). Cling gAp doi hai c~nh AI va AF thi
!::"AED '" !::"ARB (c-g-c). Tit day, taco

LABR = LADE = LDEC - LDAE = LPEF - LPAB


= LPEF - LFER = LPER = LRMP.

Do do, ttl giac N M RB n9i ti~p, hay N, M, R, B thu9C (K). L~i co

LERM = LEPM = LEFP


= LEFR + LRFP
= LRAE + LRNM.

V~y du'dng tron ngo~i ti~p tam giac A E F va du'dng tron (K) tiBp xuc nhau t~i R .
Ta eo di6u phiii chung minh. 0

Bai 4 (1. a c1iim). Cho s6 nguyen n ;? 5. Xet m9t da giac 16i n qnh. Ngu'di
ta mu6n ke m9t s6 du'dng cheo cua da giac sao cho cac dl1dng cheo nay chia
da giac da cho thanh dung k mi6n, m6i mi6n la m9t ngii giac 16i (hai mi6n
hAt ky khong co di~m trong chung).

a) Chung minh r~ng ta co thg thl!C hi~n du'<;lc voi n = 20 1 8 va k = 672.


b) Voi n = 20 1 7, k = 672 thl co thg thl!c hi~n dl1<;lC khong? Hay giiii thich.

Uti giai. a) Ky hi~u da giac 2018 c~nh la A l A 2 ... A 2018 . Ke cac du'dng
cheo AlAs, A l A 8 , A l A l1 , . . . , AIA201S. Khi do, da giac nay du'<;lC ehia
thanh 672 ngii giac 16i:

AlA2 A 3 A 4 A S, AIAsA6 A 7 A 8, ... , AIA20lSA2016A2017A2018.


Vo Qu6c Ba Cgn - Nguy€n M~nh Linh 205
CHVdNG 3. LOI GIAI cAc HE ToAN ON TAp
b) Gift sit ta co th€ chia da ghic 16i 20 1 7 c~mh nay thanh 672 ngii giac 16i bdi cac
dl10ng eheo cua no. GQi p Ia s6 giao di€m cua cac dl10ng cheo n~m trong da giac. Do
m6i dinh cua ngii giac 16i Ia dinh eua da giac d5 cho ho~c Ia mQt trong p giao di€m
eua cae dl10ng cheo nen t6ng s6 do t~t eft cae goc d cac dinh cua cae ngii giae nay Ia

p. 360° + (2017 - 2).180° = (2p + 2015).180°. (1)


M~t khac, s6 ngii giac Ia 672, m6i ngii giac co t6ng s6 do eac goc d dinh Ia 3 . 1 80°
nen t6ng s6 do cac goc d cae dinh cua cac ngii giac nay Ia 672 . 3 . 180°. (2)
Tit (1) va (2), ta suy fa 2 p + 2015 = 672 . 3, tuc P =! (vo IY). V~y ta khong
th€ thl/e hi~n dl1Qc vdi n = 2017 va k = 672. 0

" ?
1.19. De thi tuyen sinh THPT chuyen, Sit GD-DT Ha Nc;li,
2017 (d~mh cho chuyen Toan)

Bai 1 (2.0 diim).

a) GiM phl1dng trinh:


~6x - x 2 + 2X2 - 12x + 15 = O.

b) GiM h~ phl1dng trinh:

4X
2
= Y + ~'
! 4y2 = X +-.
x

Lui giai. a) Di6u ki~n: 0 ~ x ~ 6. D~t t = ~ 6x - x 2 (t ;::: 0), khi do


phl1dng trinh d5 eho co th€ dl1Qc vi€t I~i thanh

t-2t 2 +15=0,
hay
(3 - t)(5 +2t) = O.
Tit day, taco t = 3 (do t ~ 0). Do do

6x - x 2 = 9,
hay
(x - 3)2 = O.
TIt do x = 3. V~y phl1dng trinh co nghi~m duy nh~t x = 3.

206 Vo Qu6c Ba Cgn - NguySn M~nh Linh


1. cAc HE THI TUYEN SINH CHINH THOC

b) Dieu ki~n: x, y =1= o. Tu h~, ta 8Uy ra x, y > o. H~ phudng trlnh da cho co


th€ dUQc vi€t l~i thanh
4x 2 y = y2 + 3,

14y 2
x = x2 + 3.
Tru tudng ung hai phudng trlnh cua h~ thea v€, ta dUQc

4xy(x-y) = (y-x)(y +x),

hay
(x - y)(4xy + x + y) = O.
Do x, Y > 0 nen ta co x = y. Thay vao phudng trlnh thu nhat cua h~, ta dUQc

4x 3 =x 2 +3,

hay
(x - 1)(4x 2 + 3x + 3) = O.
Tuday, taco x = 1 (do x > 0). Suyra y = 1.
V~y h~ phudng trlnh da cho co nghi~m (x, y) duy nhat 1a (1, 1). o
Bai 2 (2.5 c1Mm).
a) Cho 86 nguyen t6 p > 3. Chung minh r~ng 201 7 - P 2 chia h€t cho 24 .
b) TIm tat ca cac c~p 86 nguyen dUdng (x, y) thoa man
X 3 +y3_9xy=0.

c) Cho a, b, c 1a cac 86 nguyen dudng. Chung minh r~ng

a + b + 2 v' ab + c 2
khong phai 1a 86 nguyen t6.

U/i giai. a) Do p 1a 86 nguyen t610n hdn 3 nen p Ie va p khong chia h€t cho 3.
Do p Ie nen p2 chia 8 du 1, tu do vdi chu Y2017 cling chia 8 du 1, ta 8Uy ra
20 1 7 - P 2 chia h€t cho 8. (1)
Do p khong chia h€t cho 3 nen p 2 chia 3 du 1, tu do vdi chu Y20 1 7 chia 3 du
1, ta suy ra 2017 - p2 chia h€t cho 3. (2)
Tu (1) va (2), vdi chu y (8, 3) = 1, ta co 201 7 - P 2 chia h~t cho 8 . 3 = 24.

Vo QU6c Ba Cffn - Nguy€n M~nh Linh 207


cHl1dNG 3. UJI GIAI cAc DE ToAN ON TAP
b) Phlldng trinh da cho co thS dll<JC vi€t l~i dlldi d~ng

x 3 + y3 + 33 - 3 . x . y ·3 = 27.

Sit d\lng h~ng d&ng thuc

a3 + b 3 + c3 - 3abc = (a + b + c)(a 2 + b 2 + c 2 - ab - bc - ca),

ta co phlldng trinh tren tlldng dlldng vdi

(x + y + 3)(x 2 + y2 + 9 - xy - 3x - 3y) = 27,


hay
+ y + 3)[Cx + y)2 + 9 - 3(x + y) - 3xy] = 27.
(x
Do x, yEN * nen x + y + 3 ~ 5, suy ra x + y + 3 1ft lldc khong nho hdn 5
cua 27. Tit day, ta co cae trlldng h<Jp sau:

• Tnidng hQ'p 1: x + y + 3 = 9, (x + y)2 + 9 - 3(x + y) - 3xy = 3.


Trong trlldng h<Jp nay, ta co x + y = 6 va xy = 8. Tit do suy ra x = 4,
y = 2 ho~c x = 2, y = 4 .
• Tnl'dng hQ'p 2: x + y + 3 = 27, (x + y)2 + 9 - 3(x + y) - 3xy = 1.
Trong trlldng h<Jp nay, ta co x + y = 24 va xy = 5;2, vo lyvi x, yE N*.

V~y cae c~p s6 (x, y) thoa man yeu du la (4, 2) va (2, 4).

c) Gia sit p = a + b + 2Jab + c 2 la s6 nguyen t6, khi do ta co Jab + c 2 la s6


hftu d. Theo b8 d~ 1 d€ tuySn sinh THPT chuyen KHTN nam 2015 - vong 2 (b~n
dQc vui long xem l~i phfin trlldc), ta co ab + c 2 la s6 chfnh phlldng.
D~t ab + c 2 = d 2 vdi d E N* thi ta co d > c va p = a + b + 2d. Suy ra
a =-b - 2d (mod p)

va
d2 - c2 =_b 2
- 2bd (mod p).
Tit day, ta co d 2 + b 2 + 2bd - c 2 chia h€t cho p, hay (b + d - c)(b + d + c) chia
h€t cho p. Do do, mQt trong hai s6 b + d - c va b + d + c se chia h€t cho p. Chu
°
y r~ng b + d + c > b + d - c > nen trong ca hai trlldng h<Jp, ta d€u co
b +d +c ~ p = a + b + 2d,
hay a +d - c < 0, mau thuftn vi d > c. Do do, vdi mQi a, b, c nguyen dlidng thi
a + b + 2Jab + c 2 khong thS la s6 nguyen t6. 0

208 Vo Qu6c Ba Cgn - Nguyen M~nh Linh


1. cAc HE THI TUYEN SINH CHINH THUC

Nh~n xet. Y b) con co th~ giiii b~ng cach khac nhu sau: Sit dl;lng bfit d~ng thuc
AM -GM cho hai s6 dUdng, ta co

9xy == x 3 + y3 = (x + Y)(X2 + y2 - xy) ~ (x + y)xy,


suy ra x +y ~ 9. M~tkhac, do x 3 + y3 = (x + y)3 - 3xy(x + y) nen ta cling co
9xy = (x + y)3 - 3xy(x + y). (3)

Suy ra x + y chia h~t cho 3. K~t h<;sp vdi danh gia a tren va x, y E N*, ta suy ra
x + y nh~n mQt trong cac gia tri 3, 6, 9.
• Vdi x + y = 3, thay vao (3), ta du<;sc xy = ~, vo ly VI x, y E N*.

• Vdi x + y == 6, thay vao phudng tlnh (3), ta du<;sc xy = 8. Til do, dt3 dang
tIm du<;sc x = 4, y = 2 ho~c x = 2, Y = 4.
• Vdi x + y = 9, thay vao (3), ta du<;sc xy = 841, vo lyvl x, y E N*.

V~y (x, y) E {(4, 2), (2, 4)}.


D6i vdi Yc), sau khi da d~t ab + c 2 = d 2 vdi d > c, d E N*, ta cling co th~ xV ly
ti~p nhu sau: D~t x = d - eva y = d + c thl ta co x, y E N*, xy = ab va

a + b + 2 .jab + c 2 = a + b + 2d = a + b + x + y.
Theo k~t qua diu 2c) d~ tuy~n sinh THPT chuyen cua sa GD-DT Ha NQi nam
2015, d~ danh cho chuyen Toan-Tin (bl;ln dQc vui long xern ll;li phfin trt1dC) thl·
a + b + x + y la h<;sp s6. Mau thu§:n nh~n du<;sc cho ta di~u phiii chung minh.

Bai 3 (1.5 diim). Chocacs6thl!cdudngx, y, z thoamanx 2 + y2+Z2 = 3.


Chung minh r~ng

x y z 3
3-yz + 3-zx + 3-xy ~ 2'

LCii giai. Sit dl;lng bftt d~ng thuc AM-GM, taco

y2 + Z2 3 - x2 3 + x2
3 - yz >- 3 - 2
= 3 - ---
2
?' 2
va
x~---
x2 +1
2
Vo Qu6c Ba C§:n - Nguyt3n Ml;lnh Linh 209
CHudNG 3. LOI GIAI cAc HE ToAN ON TAp

Tit do suy ra
---
X x2
~
+1 = 1 - ----,----
2
3 - Yz " x2 +3 x 2 +3
Danh gia tl1dng tI;i, ta cling co

y 2 z 2
---:(1- , --- ~ 1----
3-zx y2+3 3-xy" z2+3

Do do, ta chi dn chang minh

3_ 2( 1 + 1 + ----::--1_)
x2 +3 y2 +3 Z2 + 3

hay
1 1 1 3
x2+ 3 + y2 +3 + Z2 + 3 ~ 4'
D~n day, sa d\}ng b~t d~ng thuc Cauchy-Schwarz dl;lng cQng mau, ta co
1 1 1 9 3
---+
x2+ 3 y2 + 3
+ Z2 + 3 ~
x2 + y2 + Z2 + 9 =-.
4

D~u d~ng thuc xiiy ra khi va chi ki x = y = Z = 1. o

Bai 4 (3.0 c1iim). Cho tam giac nh<.m ABC vdi A B < A C, nQi ti~p daong
tron ( 0). GQi 1 la tam dllong tron nQi ti~p tam giac ABC, D la hinh chi~u
cua diSm I tren dllong th~ng B C va G la giao diSm tha hai cua dllong th~ng
AD vdi dllong tron ( 0 ). GQi F la digm chinh giUa cung ldn B C cua daong
tron ( 0 ). Dlldng th~ng F G c£t dl1dng th~ng 1 D tl;li digm H.

a) Chang minh r~ng ta giac 1 B He Ia ta ghic nQi ti~p.


"

b) GQi J la giao diSm thu hai cua dlldng th~ng A 1 vdi dllong tron ngol;li
ti~p tam giac B 1 C. Chung minh r~ng B H = C J.

c) GQi N la giao digm tha hai cua dlldng th~ng F H vdi dlldng tron ngol;li
ti~p tam giac B 1 C. Chung minh r~ng N J di qua trung digm cua Be.

L~i giai. a) GQi KIa giao digm cua AI va (0), khi do ta d~ th~y F, 0, K
thang hang va F K ..1 Be. Do do F K II 1 D, suy ra

LAI H = 180 0 -LH 1 K = 180 0 -L1 KF = 0


180 -LAGF = LAGH.
210 Vo Qu6c Ba Oin - Nguy€n Ml;lnh Linh
1. cAc HE THI TUYEN SINH CHINH THUC

Til do, ta co tl1 giac A I G H n9i ti~p. Suy ra


DI·DH=DG·DA.
M~t khac, ta l'.li co tl1 giac A B G C n9i ti~p nen

DG·DA=DB·DC.
K~t hQp hai k~t qua tren l'.li, ta dUQC

DI·DH=DB·DC.
Til do suy ra ttl giac B I C H n9i ti~p.
F

b) Theo tinh chftt cua goc ngoai, ta co


LBIK = LBAI + LABI = LCAI + LIBC
= LKBC + LIBC = LIBK.
Do do tam ghic B I K can t'.li K, suy ra K B = K I. M~t khac, ta l'.li co
K
la digm chinh giUa cua cung nho B C cua (0) nen K B = K C. Til do, ta co
K B = K I = K C , hay K chinh Ia tam dUdng tron ngo'.li ti~p tam giac B Ie.

Vo Qu6c Ba Cffn - Nguy~n M'.lnh Linh 211


CHUONG 3. u~n GIAI cAc HE ToAN ON TAp

Tit day, ta suy ra I J chinh Ia dlidng kinh cua dlidng tron (B I C ), rna H
n~rn tren dUdng tron nay nen L. I H J = 90°, tdc ta co H J .1 I H. L:;ti co
B C .1 IHnen H J II B C. Ma tt1 giac B C J H nQi ti€p nen B C J H la hlnh
thang can. Tit do, ta co B H = C J.
c) Do F K la dudng kfnh cua ( 0) nen L. F C K = 90°, tit do ta co F C la ti€p
tuy€n t:;ti C cua dlidng tron (B I C ). Suy ra

FC 2 =FN·FH.

G<;>i M la trung di€m cua B C thl d6 thfty M thuQc F K. Ap d\mg h~ thdc lliQng
trong tam giac F C K vuong t:;ti C, ta co

FC 2 = FM· FK.

K€t hQp hai k€t qua tren l:;ti, ta dUQC

FN·FH=FM·FK.

Tit do, ta co td giac N M K H nQi ti€p. Suy ra

L.NMF = L.NHK = L.HNK = L.HMK. (1)

M~t khac, do M K la tn;1c d6i xdng cua hlnh thang can B C J H nen

L.HMK = L.KMJ. (2)

Tit (1) va (2), ta suy ra


L.NMF = L.KMJ.
Tit do, ta co

L.NMF + L.JMF = L.KMJ + L.JMF = 180°.

V~y ba di€rn N, M, J th~ng hang, hay N J di qua trung di€m M cua Be. D

Bai 5 (1.0 aim). Xet t~p hQp S g6m cac s6 nguyen dUdng co tinh chftt: V6i
hai phdn tit phan bi¢t bdt kY x, y thu9C S, ta luon co

301x-yl ~xy.

Hoi t~p hQp S co th€ co nhi€u nhftt bao nhieu ph~n tit?

212 Vo Qu6c Ba C5n - Nguy6n M~nh Linh


1. cAc HE THI TUYEN SINH CHfNH TRUC

lCii giai. Bai toan nay co hai cach ti€p c~n nhu sau:
each 1. R6 rang ta chi dn xet tntong h<jp S co nhi~u hdn 1 phfin ta.
Chia t~p h<jp
s6 nguyen dudng thanh 10 nhom s6 nhu sau: {I}, {2}, {3}, {4}, {5}, {6, 7},
{8, 9, 10}, {ll, 12, ... , 17}, {I8, 19, ... , 29}, {30, 31, ... }.

• N€u trong nhom {3 0, 3 1, ... } co hai s6 x, Y vdi x > y cimg thuQc vao
t~p h<jp S thl ta co x y ;;:: 30 x > 30 (x - y) = 30 Ix - y I, vo ly.

• N€u trong nhom {18, 19, ... , 29} co hai s6 e, f phan bi~t cimg thuQc
vao t~p h<jp S thl ta co ef ;;:: 1 8 . 1 9 > 30 . 11 ;;:: 30 Ie - f I, vo lY.

• N€u trong nhom {8, 9, 10} co hai s6 a, b phan bi~t cung thuQc vao t~p
h<jp S thl ta co a b ;;:: 8 . 9 > 60 = 30:2 ;;:: 30 Ia - b I, vo ly.

• Xet tntong h<jp trong nhom { 11, 12, . . . , 1 7} co hai s6 c, d phan bi~t
cung thuQc vao t~p h<jp S. N€u Ie - d I :::; 4 thl ta co

cd ;;:: 11 . 12 > 30 . 4 ;;:: 30 Ic - d I,


mau thufin vdi gilt thi€t. Do do Ic - d I ;;:: 5, tuc

(c, d) E {ell, 16), (11, 17), (12, 17)}.

Tuy nhien, khi tha tn!c ti€p, ta d~u co cd> 30 Ic - d I, VO ly.


Tom l~i, trong 1 0 nhom s6 tren chi chQn du<jc tU m6i nhom t6i da mQt s6 d€ l~p thanh
t~p h<jp S thoa man d~ bai, suy ra S co t6i da 10 phfin tlt. M~t khac, ta chi ra du<jc mQt
t~p h<jp co 1 0 phfin tlt thoa man yeu du la S = {I, 2, ... , 6, 8, 11, 1 8, 45}.
V~y t~p h<jp S co th€ co nhi~u nhfit la 10 phfin tlt. •
b) GQi cac phfin ta
cua S la aI, ... , an (n, aI, ... , an E N *, n ;;:: 2).
Khong mfit tinh t6ng quat, gilt sa
a 1 < . . . < an. Khi do, thea gilt thi€t, ta co

hay
1 1 1
- - - - ; ; : : - , 'v'i=I,2, ... ,n-1. (1)
ai ai+l 30
Ta se chling minh n :::; 10. Th~t v~y, gilt sa
ngu<jc l~i n ;;:: 11. Trong (1), cho
i lfin lu<jt nh~n cac gia tri 6, 7, ... , n - 1, sau do cQng n - 6 bfit d~ng thlic
thu dudc .. lai thea v€'ta .
dudc

lin - 6
---;;::--.
a6 an 30
Vo QU6c Ba Cffn - Nguy~n M~nh Linh 213
CHVdNO 3. LOI olAI cAc HE ToAN ON TAP

Doa n > an-I>'" > al vaal, a2, ... , an E N* nena6 ~ 6, tit do taco
1 5 n-6 1 1 1 1
- = - ~ -- ~ - - - < - ~ -.
6 30 30 a6 an a6 6
Mau thuftn nh~n dUQc chung to n ~ 10. M~t khac, ta chi fa dUQC m('>t t~p hQp S co
1 0 phfin til' thOa man yeu du d6 bai la S = {I, 2, 3, 4, 5, 6, 8, 11, 1 8, 45}.
V~y t~p hQp S co thg co nhi6u nhAt la 10 phfin til'. 0
, ?

1.20. De thi tuyen sinh THPT chuyen, Sit GD-OT Ha NQi,


2017 (dfmh cho chuyen Toan-Tin)

Bai 1 (2.0 c1iim).

a) Gild phudng trinh:

.J 5x - x 2 + 2X2 - lOx +6= O.

b) Gild h~ phudng trinh:

x + y + xy = 3,
1 -./x + h= 2.

lO'i giai. a) Di6u ki~n: 0 ~ x ~ 5. D~t t = .J 5x - x 2 (t ~ 0), khi do


phudng trinh da cho co thg dUQc vi8t If;li thanh
t - 2t2 + 6 = 0,
hay
(2-t)(3+2t)=0.
Tit day, ta co t = 2 (do t ~ 0). Do do
5x - x 2 = 4,
hay
(x - l)(x - 4) = O.
Tit do x = 1 ho~c x = 4. V~y phudng trlnh co hai nghi~m la x = 1 va x = 4.
b) Di~u ki~n: x, y ~ O. H~ phudng trinh da cho co thg dUQc vi8t If;li thanh

x + y + xy = 3,
1 x+y+2,JXY=4.
214 Vo Qu6c Ba Cfin - Nguy~n Mf;lnh Linh
1. cAc HE THI TUYEN SINH CHINH THUC

Tru tudng ung hai phudng tdnh cua h~ theo v@, ta dUQc

xy - 2,Jxy = -1,

hay
(,Jxy - 1)2 = o.
Suy ra x y = 1. Til' do, ta co x + y = 2. Til' day, d@ th~y x = y = 1.
V~y h~ phudng tdnh da cho co nghi~m (x, y) duy nh~t lit (1, 1). D

Bai 2 (2.5 diim).

a) 11m t~t ca cac s6 nguyen dUdng x, y, z thoa man


i x +y-z=2,
l3x 2 +2y2_z2 = 13.

b) Cho cac s6 nguyen dudng a, b, c thoa man a 2 + b2 = c 2 . Chung


minh r~ng a b chia h@t cho a + b + c.
c) TIm t~t ca cac s6 tu nhien n thoa man 2 n + 1, 3n + 1 Ul cac s6 chinh
phudng va 2 n + 9Ul s6 nguyen t6.

ldi giai. a) Tu phudng tdnh thu nh~t, ta co z = x + y - 2. Thay vao phuong


trlnh thu hai cua h~, ta dUQc
3x 2 + 2y2 - (x + y - 2)2 = 13,
hay
2X2 + y2 - 2xy + 4x + 4y - 17 = o.
Phudng tdnh tren co th€ vi@t dUQc thanh
(x - y)2 + 4(y - x) + 4 + x 2 + 8x + 16 - 37 = 0,
hay
(x - y - 2)2 + (x + 4)2 = 37. (1)
Tit day, ta suy ra (x + 4) 2 ~ 37. Ma x E N * nen x E {I, 2}.

• Voi x = 1, thay vao (1), ta dUQc (y + 1) 2 = 12. Khong t6n t<;li y nguyen
dUdng thoa man phudng tdnh nay.

• Voi x = 2, thay vao (1), ta dUQc y2 = 1. TiI'do, y -:- 1, z = 1.


V~y co duy nhfit mQt bQ s6 (x, y, z) thoa man yeu du Ia (2, 1, 1).

VO Qu6c Ba Cgn - Nguy@n M<;lnh Linh 215


CHUONG 3. UJI GIAI cAc HE ToAN ON TAp

b) Gill thi~t da cho co th6 dl1QC vi~t I~i thanh


2ab = (a +b)2_C 2 , (2)
hay
2ab = (a +b - c)(a+ b + c). (3)
Tli (2), ta suy ra a + b va c co cling tinh chan Ie, va a + b > c. Do do a + b - c
la s6 nguyen dl1dng chan. D~t a + b - c = 2k voi k E N*, khi do tli (3), ta co

ab=k(a+b+c).
V~y a b chia h~t cho a + b + c.
c) Do 2 n + 1 va 3 n + 1 la cac s6 chinh phl1dng dl1dng nen t6n t~i cac s6 nguyen
dl1dng a, b sao cho 2 n + 1 = a 2 va 3 n + 1 = b 2. Khi do ta co

2n+9 = 2S(2n+1)-16(3n+l) = 2Sa 2 -16b 2 = (Sa-4b)(Sa+4b).


Do 2 n + 9 la s6 nguyen t6, Sa + 4 b > 1 va Sa + 4 b > Sa - 4 b nen ta phai
co Sa - 4b = 1, tuc b = 5a4-1. Ta day, ta co h~ phl1dng trlnh:
2n+l=a 2 , (4)
(Sa - 1)2
{ 3n + 1 = 16 (5)

2
Tli (4) suy ra n = a 2
-1 va a > 1 (do n > 0). Thay vao (5), ta dl1QC

3(a 2 -1) +1- (Sa-l)2


2 - 16 '
hay
(a - 1) (a - 9) = O.
Suy ra a = 9. Tli do, ta co n = 40. Thi1l~i, ta thAy thoa man.
V~y co duy nhAt mQt gia tr! n thoa man yeu cfm la n= 40. o

Ba i 3 (1.5 dim). Cho cac s6 thl!C dl1dng a, b, c thay d6i luon thoa man
111
2"
a
+ b 2 + 2"
c = 3.
TIm gia tri ldn nhAt cua bi6u thuc

i l i
P - + + ------::-
- (2a + b + C)2 (2b + c + a)2 (2c + a + b)2 .

216 Vo Qu6c Ba Can - Nguyen M~nh Liilh


I, cAc DE THI TUYEN SINH CHfNH THUC

LCii gifli. Bai toan nay co ba each ti€p nh~n nhl1 sau:
Cach 1. Sit dl.mg bftt d~ng thuc Cauchy-Schwarz, ta co

1 1
4 ( -+-+-+-
1 1) (1 1 1 1)2
~ -+-+-+- ~ (
16 )2
a2 a2 b2 c2 a abc 2a +b+c '
suyra

(2a
1
+ b + C)2 ~
1
64
(2 a2 + b2 + c2
1 1)
'
(1)

Danh gia tl10ng tl!, ta ciing co

(2b
1
+ c + a)2
~ _1
"64
+_1 )
(~+
b2
~2
c a2 '
1 ~ _1 (~+ _1 +_1 )
(2c + a + b)2 "64 c 2 a2 b2 '

CQng cac bftt d~ng thuc tren l~i theo v€, ta dl1<;!c

1 (1 1 1) 3
P ~ 16 a2 + b2 + c2 = 16'
Dftu d~ng thuc xay ra khi va chi khi a =b= c = 1, V~y max P = 3
1 6' •

Cach 2. Y tl1dng eua cach nay ciing Ul thi€t l~p danh gia (1) nhl1ng b~ng cach sit
d\mg bftt d~ng thue AM-GM nhl1 sau:

1
------~ 1 1[
~- -1 + - -
1 -]
(2a+b+c)2"8a(b+c)"84a 2 (b+c)2

~ ~ (4~2 + 4~C) ~ ~ [4:2 + ~ (b12 + c12 )]


1
= 64
(2a + 2
1
b2 + c2
1)
' •

each 3. Sit dVng bftt d~ng thuc AM-GM, ta co


1 1
-----~ ,
(2a+b+c)2 4(a+b)(a+c)
Danh gia tl10ng tt;!, ta ciing co
1 1
------~ ------
(2b + c + a)2 " 4(b + c)(b + a)'
1 1
------~ ,
(2c+a+b)2 4(c+a)(c+b)
Vo QU6c Ba Cffn - Nguy~n M~nh Linh 217
CHVdNO 3. LOr orAr cAc HE ToAN ON TAp

CQng cac b~t d~ng thuc tren I?i thea v~, ta dUQc

I I I
P ~ + + -------
4(a + b)(a + e) 4(b + e)(b + a) 4(e + a)(e + b)
a+b+e
2(a + b)(b + e)(e + a)'
D~n day, sil' dl,mg b8 d~ 1 (j bai 2b) d~ tuy€n sinh THPT chuyen KHTN nam 2017
- vong 2 (b?n dQc vui long xem I?i phfin tr1idc), ta dUQc

p~ a+b+e 9
'" 19
6
(a + b + e)( a b + b e + ea ) 16(ab + be + ea)'
M~t khac, sit dl,mg b~t d~ng thuc AM-OM, ta I?i co

1 1
3(ab + be + ea) = (ab + be + ea) ( 7;2 + b2 + e12 )
~3Va2b2e2.331
y a 2b12e 2
= 9.

Suy ra a b + be + e a ~ 3. Tli do, ta co

p ~ 9 3
"" 16·3 16

D~u d~ng thuc xay ra khi va chi khi a = b = e = 1. V~y max P = ?6' D

Bai 4 (3.0 aim). Cho tarn giac nhQn ABC vdi A B < A C, nQi ti~p duong
tron ( 0). GQi D la trung di€m clla c?nh Be, E la hlnh chi~u clla di€m A
tren c?nh B C va H la tl1!C tarn clla tarn giac ABC. Duong th~ng A D c~t
duong tron (0) t?i di€m tha hai F.

a) Chung rninh B C 2 = 4· DA . DF.


b) Tia D H dt duong tron (0) t?i di€m G. Chang minh r~ng b6n diem
A, G, E va D cling thuQc mQt dUdng tron.

c) Duong th~ng F E dt dUdng tron ( 0) t?i di€m tha hai K. Chung minh
r~ng dUdng th~ng B C ti~p xuc vdi dUdng tron ngo?i ti~p tarn giac G K E .

218 Vo Qu6c Ba Cffn - Nguy~n M?nh Linh

You might also like